Download as pdf or txt
Download as pdf or txt
You are on page 1of 149

PRELIM BITS

UPSC / CSE Medieval History PYQs


[2023 – 1995]

P. Roman
8/10/2023

Do NOT Share This Document


1

1. Consider the following dynasties [2023]


1. Hoysala
2. Gahadavala
3. Kakatiya
4. Yadava
How many of the above dynasties established their kingdoms in early eighth century AD?

A. Only one
B. Only two
C. Only three
D. None
Ans. D

HOYSALA EMPIRE

 The Hoysala Empire ruled most of what is now Karnataka, India between the 10th and
the 14th centuries. The capital of the Hoysalas was initially located at Belur but was later
moved to Halebidu.

 The Hoysala rulers were originally from Malenadu, an elevated region in the Western Ghats.
In the 12th century, taking advantage of the internecine warfare between the Western
Chalukya Empire and Kalachuris of Kalyani, they annexed areas of present-day Karnataka
and the fertile areas north of the Kaveri delta in present-day Tamil Nadu. By the 13th
century, they governed most of Karnataka, minor parts of Tamil Nadu and parts of western
Andhra Pradesh and Telangana in the Deccan Plateau.

 The Hoysala era was an important period in the development of art, architecture, and
religion in South India. The empire is remembered today primarily for HOYSALA
ARCHITECTURE. Over a hundred surviving temples are scattered across Karnataka.
 Well known temples "which exhibit an amazing display of sculptural exuberance" include

Contact : https://t.me/EN123upsc || Telegram Link : https://t.me/prelimbits


2

the Chennakeshava Temple (Vishnu), Belur, the Hoysaleswara Temple (Shiva),

Halebidu, and the Chennakesava Temple or Keshava Temple (Krishna) ,


Somanathapura.

 The Hoysala rulers also patronised the fine arts, encouraging literature to flourish in
Kannada and Sanskrit.

THE GAHADAVALAS OF KANNAUJ

 Claimed to be Suryavanshi kshatriyas, ruled the kingdom of Kannauj


 Chandradeva founded the Gahadavala dynasty of Kannauj around 1090 CE.

 They gradually squeezed the Palas out of Bihar and made Banaras a second capital.

KAKATIYA DYNASTY

 The Kakatiya dynasty ruled most of eastern Deccan region comprising present day
Telangana and Andhra Pradesh, and parts of eastern Karnataka and southern Odisha
between 12th and 14th centuries.

 Their capital was Orugallu, now known as Warangal.

 Early Kakatiya rulers served as feudatories to Rashtrakutas and Western Chalukyas for
more than two centuries.

 They assumed sovereignty under Prataparudra I in 1163 CE by suppressing other


Chalukya subordinates in the Telangana region.

 Ganapati Deva (r. 1199–1262) significantly expanded Kakatiya lands during the
Contact : https://t.me/EN123upsc || Telegram Link : https://t.me/prelimbits
3

1230s and brought under Kakatiya control the Telugu-speaking lowland delta areas around
the Godavari and Krishna rivers.

 Motupalli, now in Krishna district, was an important seaport in Ganapati Deva reign.

 Ganapati Deva was succeeded by Rudrama Devi (r. 1262–1289) who is one of the few
queens in Indian history.

o Marco Polo, who visited India around 1289–


1293, made note of Rudrama Devi's rule and
nature in flattering terms. She successfully
repelled the attacks of Yadavas (Seuna) of
Devagiri into the Kakatiyan territory.

 In 1303, Alauddin Khilji, the emperor of the Delhi


Sultanate invaded the Kakatiya territory which ended
up as a disaster for the Turks.

 But after the successful siege of Warangal in 1310,


Prataparudra II was forced to pay annual tribute to
Delhi.

 Another attack by Ulugh Khan in 1323 saw stiff


resistance by the Kakatiyan army, but they were finally defeated.

 Kakatiya era the development of a distinct style of architecture which improved and
innovated upon the existing modes. Most notable examples are the Thousand Pillar
Temple in Hanamkonda, Ramappa Temple in Palampet, Warangal Fort, and Kota Gullu
in Ghanpur.

SEUNA (YADAVA) DYNASTY


 The Seuna, Sevuna, or Yadavas of Devagiri (1187–1317) was a Medieval Indian dynasty,
which at its peak ruled a kingdom stretching from the Narmada river in the north to the
Tungabhadra river in the south, in the western part of the Deccan region.
 Its territory included present-day Maharashtra, North Karnataka and parts of Madhya
Pradesh, from its capital at Devagiri (present-day Daulatabad in modern Aurangabad
district, Maharashtra).
 The Yadavas initially ruled as feudatories of the Western Chalukyas. Around the middle
of the 12th century, as the Chalukya power waned, the Yadava king Bhillama V declared
independence.
 The Yadava kingdom reached its peak under Simhana II, and flourished until the early
14th century, when it was annexed by the Khalji dynasty of the Delhi Sultanate in 1308
CE.

Contact : https://t.me/EN123upsc || Telegram Link : https://t.me/prelimbits


4

2. Who among the following rulers of Vijayanagara Empire constructed a large Dam
across Tungabhadra River and 'a canal-cum-aqueduct several kilometres long from
the river to the capital city? [2023]
A. Devaraya I
B. Mallikaijuna
C. Vira Vijaya
D. Virupaksha
Ans. A

SANGAMA DYNASTY (C.1336–1485 CE) → DEVA RAYA I (C.1406–1422 CE)


 Deva Raya I was very capable ruler noted for his military exploits and his support to
irrigation works in his kingdom.

 He constructed a dam across river Tungabhadra to bring canals into the city to relieve
the shortage of water. He also built a dam on the river Haridra for irrigation purposes.

 Continually at war with the Velamas of Telangana, the Bahmani Sultan of Gulbarga, the
Reddis of Kondavidu, and the traditional rivals of Vijayanagara, the Gajapatis of Kalinga.

 He modernised the Vijayanagar army by improving the cavalry, employing Turkic


archers, and procuring horses from Arabia and Persia.

 He maintained a secular attitude in administrative matters. He had 10,000 Muslims in his


army, being the first Vijaynagar king to do so.

 Italian traveller, Nicolo Conti (in c.1420), and Russian merchant Nikitin (author of Voyage
to India), visited during his reign.

 Deva Raya I was a patron of Kannada literature and architecture. There was a pearl hall
where he honoured men of eminence.

 Madhura, a noted Jain poet, was in his court (and also in the court of his father King
Harihara II).

Contact : https://t.me/EN123upsc || Telegram Link : https://t.me/prelimbits


5

o He wrote the Dharmanathapurana on the life of the 15th Jain Tirthankar in


Kannada, as well as a poem in eulogy of Gommateshvara of Shravanabelagola.

 The noted Hazare Rama temple was constructed during his rule.

3. In medieval India, the term “Fanam” referred to: [2022]


A. Clothing
B. Coins
C. Ornaments
D. Weapons

Ans. B

CHALUKYAS OF BADAMI → COINAGE


 The Badami Chalukyas minted coins that were of a different standard compared to the
coins of the northern kingdoms.

 The coins had Nagari and Kannada legends

 The coins of Mangalesha had the symbol of a temple on the obverse and a 'sceptre
between lamps' or a temple on the reverse.

 Pulakeshin II's coins had a caparisoned lion facing right on the obverse and a temple on
the reverse.

 The coins weighed 4 grams and were called, in old-Kannada, hun (or honnu) and had
fractions such as fana (or fanam) and the quarter fana (the modern day Kannada equivalent
being hana – which literally means "money").

 A gold coin called gadyana is mentioned in a record at the Vijayeshwara Temple at


Pattadakal, which later came to be known as varaha (their royal emblem).

4. With reference to Indian history, consider the following statements:


1. The first Mongol invasion of India happened during the reign of Jalal-ud-din Khalji.
2. During the reign of Ala-ud-din Khalji, one Mongol assault marched up to Delhi and
besieged the city.
3. Muhammad-bin-Tughlaq temporarily lost portions of north-west of his kingdom to
Mongols.

Which of the statements given above is/are correct?

A. 1 and 2
B. 2 only
C. 1 and 3
D. 3 only

Ans. B
Contact : https://t.me/EN123upsc || Telegram Link : https://t.me/prelimbits
6

FIRST MONGOL INVASION


 The first Mongol invasion of India took place in the reign of Sultan Shams-ud-din
lltutmish.
 In 1221, the Mongols, under Genghis Khan, appeared for the first time on the banks of
the river Indus.
o Genghis Khan made several incursions into the Indian subcontinent when he chased
the Jalal al Din, the last ruler of the Khwarezmian Empire, all the way to the
Indus river in 1221.
o The Khwarezmian Empire was destroyed by the Mongols the year before and
Genghis Khan had ordered anyone member of the ruling family to be tracked down
and killed.
o In their pursuit of Jalal, the Mongols sacked several cities in the Punjab region
but their incursions were limited towards the western banks of the Sindh river
at the time.

KHILJI DYNASTY
 Balban (Slave Dynasty) died in 1287 and he was succeeded by his grandson, Kayqubad.
 Meanwhile, the Mongol Empire had splintered into four different khanates following the
death of Genghis Khan in 1227. One of these, the Chagatai Khanate came under the
control of Duwa and his son Qutlug Khwaja. They made alliances with other Mongol
tribes in the region, building up their forces for a southward expansion.
 In 1290 Khalji nobles headed by Jalaldduin, overthrew the incompetent successors of
Balban in 1290 & Est. Khalji dynasty == > Dynastic Revolution of 1290
 Later Jalauddin Khalji was murdered by his son-in-law Alauddin Khalji in 1296 and
usurped throne

ALAUDDIN KHILJI → BATTLE OF KILI


 Alauddin started a number of reforms to consolidate power and to prepare against
future Mongol invasions, such as paying the army in cash and retaining personal
control instead of delegating it to his generals.
 The Chagatai Khanate (Mongol) invaded Punjab in 1292, but their advance guard
under Ulghu was defeated and taken prisoner by the founder of the Khilji
Dynasty, Jalaluddin.
 Chagatai armies were beaten by the Delhi Sultanate several times in 1296–1297.
 In 1299 on the orders of Duwa Khan‘s sons Qutlug Khwaja and Temur Buqa, a Mongol
army marched towards Delhi with a force of 50,000-60,000 horsemen. The numbers of the
Mongols prove this was a full-scale invasion instead of a punitive raid. Intending to strike at
Delhi while the sultanates main army was campaigning in Gujarat, the Mongols
bypassed several towns and villages in the process.
o They were met by a sultanate force led by Zafar Khan at the river Yamuna. The

Contact : https://t.me/EN123upsc || Telegram Link : https://t.me/prelimbits


7

Delhi forces were defeated and forced to retreat to Delhi.


 Upon the news of the defeat, Alauddin held a council on the course of action to be taken.
 Alauddin decided to give battle and marched out with whatever forces he could raise,
towards Kili, north of Delhi.
 Mongols were defeated in this battle

MUHAMMAD BIN TUGHLUQ → SIEGE OF DELHI


 In 1327 the Chagatai Mongols under Tarmashirin laid siege to Delhi and he only lifted it
when he was paid a huge ransom by the Tughlaqs.
 Tarmahirin led the invasion of India during the reign of Muhammad Bin Tughluq.
 No major invasions would take place following Tamashirin‘s siege of Delhi.

5. With reference to Indian history, who of the following were known as “Kulah-Daran” ?
A. Arab merchants
B. Qalandars
C. Persian calligraphists
D. Sayyids

Ans. D

 The religious class of Islam was composed of a number of important groups , namely, the
theologians , the ascetics, the sayyids, the Pirs , and their descendants.
 Of these, the most important were the Theologians. The theologians who occupied the
judicial and religious offices in kingdom were collectively known as Dastar-bandan or
turban wearers, because they wore their official head-dress, the turban.
 The Sayyids put on a pointed cap (kulah) and they were known as ‗Kulah Daran‘
during Delhi sultanate.
 The Sayyids were claimed to descent from the Prophet through his daughter Fatima. They
Commanded special respect in Muslim society.
 Even the Timur protected the life of Sayyids during his invasion in India. Although his policy
was one of general slaughter.

6. Consider the following pairs:


King Dynasty
1. Nannuka Chandela
2. Jayashakti Paramara
3. Nagabhata II Gurjara-Pratihara
4. Bhoja Rashtrakuta
How many pairs given above are correctly matched?

A. Only one pair


B. Only two pairs
C. Only three pairs
Contact : https://t.me/EN123upsc || Telegram Link : https://t.me/prelimbits
8

D. All four pairs

Ans. B

 Nannuka - Chandela
 Jayashakti - Chandela
 Nagabhata II - Gurjara-Pratihara
 Bhoja - Paramara dynasty of Malwa

CHANDELLAS OF BUNDELKHAND (OR JEJAKABHUKTI)


 Feudatories of the Gurjara−Pratiharas of Kannauj

 Involved in conflicts with the Pratiharas, the Palas, and the Kalachuris of Chedi (who
bordered the Chandellas on the south).

 Well-known for their art and architecture in Nagra style temples at their original
capital Khajuraho

 Lakshmana Temple (by ruler Yashovarman), Vishvanatha Temple (by ruler Dhanga) and
Kandariya Mahadeva Temple (by ruler Vidyadhara)

 They also commissioned a number of water bodies, palaces and forts at other places

 Dynasty was founded by Nannuka & his capital at Kharjjuravahaka (Khajuraho).

 Important forts: Jayapura-Durga (modern Ajaigarh), Kalanjara (modern Kalinjar), and


their later capital Mahotsava-Nagara (modern Mahoba).

Important Rulers

Early rulers

 The Chandelas were originally vassals of the Gurjara-Pratiharas.


 Nannuka (r. c. 831–845 CE), the founder of the dynasty, was the ruler of a small kingdom
centered around Khajuraho.
 According to the Chandela inscriptions, Nannuka's successor Vakpati defeated several
enemies.
 Vakpati's sons Jayashakti (Jeja) and Vijayashakti (Vija) consolidated the Chandela
power.
o According to a Mahoba inscription, the Chandela territory was named
"Jejakabhukti" after the Jayashakti.
 Vijayashakti's successor Rahila is credited with several military victories in eulogistic
inscriptions.
 Rahila's son Harsha played an important role in restoring the rule of the Pratihara king
Mahipala, possibly after a Rashtrakuta invasion or after Mahiapala's conflict with his step-
brother Bhoja I

Contact : https://t.me/EN123upsc || Telegram Link : https://t.me/prelimbits


9

Yashovarman (c. 925−950 CE)

 He continued to acknowledge the Pratihara suzerainty, but became practically


independent.
 His reign marked the beginning of the famous Chandella style art and architecture.

 He commissioned the famous Lakshmana Temple at Khajuraho (lord Vishnu)

 Vakpatiraja was famous court poet & he composed Gaudavaho. It mentions


achievements of Yasovarman.

Dhanga-Deva (c. 950−999 CE)

 First independent Chandella king who took the title of Maharajadhiraja== > Chandellas
become a sovereign power
 Undertake extensive military campaigns.

 Commissioned a magnificent Vishvanatha Temple (dedicated to Shiva) at Khajuraho.


Parshvanatha temple (jain) was also build during his reign.

o Architectural style is similar to that of the older Lakshmana Temple and the newer
Kandariya Mahadeva Temple.

Vidyadhara (c. 1003 – 1035 CE)

 The Ghaznavid ruler, Mahmud of Ghazni invaded the kingdom and Vidyadhara had to
pay tribute to him.

 The Muslim invasions weakened the Chandella empire and the Kalachuri king Gangeya
Deva taking advantage of the situation captured the eastern parts of the kingdom.

 During his reign, the Kandariya Mahadeva (Shiva) temple was built.

Contact : https://t.me/EN123upsc || Telegram Link : https://t.me/prelimbits


10

Paramardi-deva (c. 1165−1203 CE)

 last powerful Chandella king

 Around 1183 CE, the Chahamana ruler Prithviraj Chauhan invaded the Chandella
kingdom and sacked Mahoba (Capital) as per the Madanpur stone inscriptions.

 The Chandella force led by Alha, Udal, and other generals, was defeated in this battle.

Decline

 The Chandella power continued to decline because of the rising Islamic (Delhi Sul.)
influence, as well as the rise of other local dynasties.

GURJARA-PRATIHARA EMPIRE (WESTERN INDIA): 730-1036


 Primarily pastoralists and fighters.

 Dynasty was founded by Harichandra in and around Jodhpur, south western Rajasthan
 Pratihara were instrumental in containing Arab armies moving east of the Indus River.

 Gurjara-Pratihara are known for their sculptures, carved panels and open pavilion style
temples. The greatest development of their style of temple building was at Khajuraho, a
World Heritage Site.

 They came to prominence in the second quarter of the 8th century, during the reign of
Nagabhatta I.
 Sanskrit Poet & Dramatist Rajasekhara lived in the court of Mahendrapala I – son of
Mihir bhoja.

 Foreign traveller Al-Masudi had visited Pratihara Empire.

Important Rulers

Nagabhatta I (730-760)

 Most famous Pratiharas kings, known for checking Arab armies. He defeated the Arab
army under Junaid and Tamin during the Caliphate campaigns in India.

 He exercised control over the areas of Malwa, Rajputana, and Gujarat.

 Defeated by Rashtrakuta king Dhruva.

Contact : https://t.me/EN123upsc || Telegram Link : https://t.me/prelimbits


11

Vatsaraja (780-800)

 Made Kannauj in western U.P as his capital.

 His Expansionist policy brought him into conflict with Dharmapala, the Pala king and
Rashtrakuta king Dhruv, thus began ―Tripartite Struggle‖ and continued for about another
350 years.

 Vatsraja defeated the Pala ruler Dharmapala and Dantidurga, the Rashtrakuta
king, for control of Kannauj.

Tripartite struggle

Nagabhata II (805–833)

 He conquered Kannauj and the Indo-Gangetic Plain as far as Bihar from the Palas, and
again checked the Muslims in the west.

 He rebuilt the Shiva temple at Somnath in Gujarat, which had been demolished in an Arab
raid from Sindh.

Tripartite struggle

Bhoja I/ Mihir Bhoja (836 885)


 Greatest ruler of Pratiharas, got victories over Palas and Rashtrakutas.
 Capital at Kannauj, which also known as Mahodaya.

 Barah copper plate inscription, there is mention his military camp i.e., skandhavara at
Mahodaya.

 Called King Baura by Al-Masudi & Abu suleman called him king Juzr. According to Arab
travellers, the Pratihara rulers had the best cavalry in India.
 Expansion checked by Sankarvarmen of Kashmir and Rashtrakuta Krishna II and Devapala

 Devotee of Vishnu, adopted title Adivaraha.

 He made Teli mandir Gwalior Dedicated to Shiva, Vishnu and Matrikas,


Mahendrapala

 Empire reached its peak of prosperity and power.

Contact : https://t.me/EN123upsc || Telegram Link : https://t.me/prelimbits


12

 Fought a battle with the king of Kashmir but had to yield to him some of the territories in
the Punjab won by Bhoja

 Took the title Maharajadhiraja of Aryavarta (Great King of Kings of Northern India).

 His court was adorned by Rajashekhar, who was an eminent Sanskrit poet, dramatist
and critic who wrote:

o Karpuramanjari: A famous play written in Sauraseni Prakrit to please his wife,


Avantisundari, a woman of taste and accomplishment. He is perhaps the only
ancient Indian poet to acknowledge a woman for her contributions to his literary
career.

o Kavya Mimansa (around c.880−920 CE): A practical guide for poets that explains
the elements and composition of a good poem.

o Vidhasalabhanjika
o Bhrinjika

o Balaramayana

o Prapanch Pandav
o Balabharata

o Bhusan Kosh

Mahipala I c. 913–944 CE)

 Defeated by the Rashtrakuta King Indra III, who completely devastated the city of
Kannauj.
 Gujarat passed into the hands of the Rashtrakutas in this period, in all likelihood, as Al
Masudi in his accounts mentions that the Pratihara empire had no access to the sea.
Rajyapala (c. 960–1018 CE)

 The Rashtrakuta king Krishna III invaded north India and defeated the Pratihara ruler.

 Raid of Mahmud Ghazni on Kannauj; Rajyapala fled from battlefield.

Yashpala (c.1024–1036 CE)

 Last ruler of this dynasty

 By 1090 CE, the Gadhavalas conquered Kannauj

In 11th C. Pratiharas were wiped out from political scenario by Ghaznavids and this lead
to fragmentation of Rajput.
Their rule was succeeded by Chahamanas or Chauhans in Rajputana, Chalukyas or Solankis in
Gujarat and the Paramaras or Pawars in Malwa
Prithvirajraso

Contact : https://t.me/EN123upsc || Telegram Link : https://t.me/prelimbits


13

 Prithvirajraso written by Chandbadrai is the first major source for Rajput history.

 According to this book Rajputs were originated from a fire sacrifice performed at
Mt.Abu .

 Prithviraj Raso is considered as the one of first book in Hindi language.

Gurjara-Pratihara Art

 There are notable examples of architecture from the Gurjara-Pratihara era, including
sculptures and carved panels.

 Their temples, constructed in an open pavilion style.

 One of the most notable Gurjara-Pratihara style of architecture was Khajuraho, built by
their vassals, the Chandelas of Bundelkhand.

 Maru-Gurjara architecture.

 Tenples built during this period

o Mahavira Jain temple, Osian

o Bateshwar Hindu temples complex (MP) → Shiva, Vishnu and Shakti

o Baroli temples complex (Raj)

7. “Yogavasistha” was translated into Persian by Nizamuddin Panipati during the reign of:
A. Akbar
B. Humayun
C. Shahjahan
D. Aurangzeb

Ans. A

 Vasishta Yoga Samhita also known as Maha-Ramayana, Arsha Ramayana, Vasiṣṭha


Ramayana, Yogavasistha-Ramayana and Jnanavasistha is a historically popular and
influential syncretic philosophical text of Hinduism, dated to the 6th CE or 7th CE —
14th CE or 15th CE.
 It is attributed to Maharishi Valmiki, but the real author is unknown.
 The complete text contains over 29,000 verses. The short version of the text is called Laghu
yogavāsiṣṭham and contains 6,000 verses.
 The text is named after sage Vasistha who is mentioned and revered in the seventh book
of the Rigveda, and who was called as the first sage of the Vedanta school of Hindu
philosophy by Adi Shankara. The text is structured as a discourse of sage Vasistha to
Prince Rama.

Persian Trnaslation

Contact : https://t.me/EN123upsc || Telegram Link : https://t.me/prelimbits


14

 During the Mughal Dynasty the text was translated into Persian several times, as ordered
by Akbar, Jahangir and Darah Shikuh.
 One of these translations (Akbars reign) was undertaken by Nizam al-Din Panipati in
the late sixteenth century AD. This translation, known as the Jug-Basisht, which has since
become popular in Persia among intellectuals interested in Indo-Persian culture.
 The Safavid-era mystic Mir Findiriski (d. 1641) commented on selected passages of Jug-
Basisht.

8. The world‟s second tallest statue in sitting pose of Ramanuja was inaugurated by the
Prime Minister of India at Hyderabad recently. Which one of the following statements
correctly represents the teachings of Ramanuja? [2022]
A. The best means of salvation was devotion.
B. Vedas are eternal, self-existent and wholly authoritative.
C. Logical arguments were meant for the highest bliss.
D. Salvation was to be obtained through meditation.

Ans. A

RAMANUJACHARYA
 Born in 1017 in Sriperumbudur in Tamil Nadu, Ramanujacharya is revered as a Vedic
philosopher and social reformer.
 He was named Lakshmana at the time of his birth. He was also referred to as Ilaya Perumal
which means the radiant one.
 He traveled across India, advocating equality and social justice.
 He revived the Bhakti movement, and his preachings inspired other Bhakti schools of
thought. He is considered to be the inspiration for poets like Annamacharya, Bhakta
Ramdas, Thyagaraja, Kabir, and Meerabai.
 According to him the best means of attaining salvation was through intense devotion
to Vishnu. Vishnu in His grace helps the devotee to attain the bliss of union with Him.
 He is famous as the chief proponent of Vishishtadvaita subschool of Vedanta.
o VishishtAdvaita (literally "Advaita with uniqueness; qualifications") is a non-dualistic
school of Vedanta philosophy.
o It is non-dualism of the qualified whole, in which Brahman alone is seen as the
Supreme Reality, but is characterized by multiplicity.
 He went on to write nine scriptures known as the navaratnas, and composed numerous
commentaries on Vedic scriptures.
 Ramanuja‘s most important writings include his commentary on the Vedanta Sutras (the
Sri Bhasya, or "True Commentary"), and his commentary on the Bhagavad-Gita (the
Gitabhasya, or "Commentary on the Gita").
 His other writings include the Vedartha Samgraha ("Summary of the Meaning of the
Veda"), the Vedantasara ("Essence of Vedanta"), and Vedantadipa ("Lamp of Vedanta").
 Ramanuja was an advocate of social equality among all sections of people centuries

Contact : https://t.me/EN123upsc || Telegram Link : https://t.me/prelimbits


15

ago, and encouraged temples to open their doors to everyone irrespective of caste or
position in society at a time when people of many castes were forbidden from entering
them.
The Prime Minister has inaugurated the Statue of Equality, a statue of Ramanujacharya, on the
outskirts of Hyderabad, Telangana.

9. According to Portuguese writer Nuniz, the women in the Vijayanagara empire were
expert in which of the following areas? [2021]
1. Wrestling
2. Astrology
3. Accounting
4. Soothsaying

Select the correct answer using the code given below:

A. 1, 2 and 3 only
B. 1, 3 and 4 only
C. 2 and 4 only
D. 1, 2, 3 and 4

Ans : D

VIJAYANAGARA EMPIRE → SOCIAL LIFE


 Allasani Peddanna in his Manucharitam refers to the existence of four castes —
Brahmins, Kshatriyas, Vaishyas, and Sudras — in the Vijayanagar society.

 Domingo Paes mentions the beautiful houses of the rich and the large number of their
household servants. Nicolo Conti refers to the prevalence of slavery.

 Silk and cotton clothes were mainly used for dress. Dancing, music, wrestling, gambling,
and cock-fighting were some of the common forms of amusement.

 The Sangama rulers were chiefly Shaivaites and Virupaksha was their family deity while
other dynasties were Vaishnavites. Ramanuja‘s Srivaishnavism was very popular.

 It is pertinent to note that all kings were tolerant towards other religions.

 Barbosa (Portugal) referred to the religious freedom enjoyed by everyone.

o Regarding Krishna Deva Raya‘s tolerant attitude, Barbosa says, ―The king allows
such freedom that every man may come and go and live according to his own creed,
without suffering any annoyance, and without enquiry whether he is a Christian, Jew,
Moor or heathen‖.

 Muslims were employed in the administration and they were freely allowed to build
mosques and worship. Deva Raya II enrolled Muslims in his armed services, allotted them
jagirs, and erected a mosque for their use in the city.

Contact : https://t.me/EN123upsc || Telegram Link : https://t.me/prelimbits


16

 The position of women was not much improved. However, some of them were
learned such as:

o Gangadevi, wife of Kumarakampana, (famous work Maduravijayam)

o Hannamma and Thirumalamma were two other famous poets.

 According to Nuniz, a large number of women were employed in the royal palaces as
dancers, domestic servants, and palanquin bearers. They were also experts in Wrestling,
Astrology, Accounting, Soothsaying.

o Portuguese traveller Fernoa Nuniz, came to India during Achyuta Deva Raya
(c.1529 –1542 CE) reign

 Sahagaman, i.e., Sati was honoured. The practice of devadasi (attachment of dancing
girls to temples) was in place. Even Paes refers to the flourishing devadasi system of this
period.

10. Consider the following statements:


1. It was during the reign of Iltumish that Chengiz Khan reached the Indus in pursuit of the
fugitive Khwarezm prince.
2. It was during the reign of Muhammad bin Tughluq that Taimur occupied Multan and
crossed the Indus.
3. It was during the reign of Deva Raya II of Vijayanagara Empire that Vasco da Gama reached
the coast of Kerala.

Which of the statements given above is/are correct?

A. 1 only
B. 1 and 2
C. 3 only
D. 2 and 3

Ans. A

FIRST MONGOL INVASION


 The first Mongol invasion of India took place in the reign of Sultan Shams-ud-din
lltutmish.
 In 1221, the Mongols, under Genghis Khan, appeared for the first time on the banks of
the river Indus.
o Genghis Khan made several incursions into the Indian subcontinent when he chased
the Jalal al Din, the last ruler of the Khwarezmian Empire, all the way to the
Indus river in 1221.
o The Khwarezmian Empire was destroyed by the Mongols the year before and
Genghis Khan had ordered anyone member of the ruling family to be tracked down

Contact : https://t.me/EN123upsc || Telegram Link : https://t.me/prelimbits


17

and killed.
o In their pursuit of Jalal, the Mongols sacked several cities in the Punjab region
but their incursions were limited towards the western banks of the Sindh river
at the time.

TUGHLAQ DYNASTY → NASIRUDDIN MUHAMMAD (1390-1398) → TAIMUR‘S INVASION


 last ruler

 Taimur‘s invasion (1398) during his reign weakened the sultanate.

 When Timur entered Delhi there was no opposition. He withdrew from India in 1399.

 Delhi sultanate disintegrated towards the beginning of 15th century and no of


independent states set up. E.g. Malwa and Gujarat etc.

 Tughlaq empire came to end in 1412.

VASCO-DE-GAMA (1498)
Deva Raya II of the Vijayanagara empire ruled between 1425 and 1446 AD. Vasco Da Gama
reached the coast of Kerala in 1498.

He came to India during Saluva dynasty ruled by Narasimha Raya II (1491–1505 CE), last ruler

 First Portuguese to came india.


 Piloted by a Gujarati named Abdul Majid, Vasco da Gama sailed from the Cape of Good
Hope and continued to India, reaching Calicut (Kozhikode) in 1498.
 Welcomed by Zamorin
 He thus discovered a new sea route from Europe to India and became the first
European to reach India by sea.
 Vasco da Gama led two Portuguese India Armadas (or fleet of warships), the First and the
Fourth [1502], sent under the patronage of King Manuel J.
 Vasco 2nd visit led the establishment of :
o In 1503, the 1st Portuguese fort was set up at Cochin (Kochi).
o In 1505, the 2nd Fort was set up at Cannanore (or Kannur).
11. With reference to medieval India, which one of the following is the correct sequence in
ascending order in terms of size?
A. Paragana – Sarkar – Suba
B. Sarkar – Paragana – Suba
C. Suba – Sarkar – Paragana
D. Paragana – Suba – Sarkar

Ans. A

Contact : https://t.me/EN123upsc || Telegram Link : https://t.me/prelimbits


18

AKBAR (1556-1605) → PROVINCIAL ADMINISTRATION


Suba
 Each suba was placed under a Subedar (provincial governor) who was directly appointed
by the Emperor. The subedar was the head of the province and responsible for
maintenance of general law and order

 Diwan : Head of the revenue Dept [taccavi (advance loans) were given thru this office]

 Bakshi : same function as performed by Mir Bakshi at the centre

o Responsible for checking and inspecting the horses and soldiers maintained by
the mansabdars in the suba. He issued the pay bills of both the mansabdars and
the soldiers

 Sadar : Responsible for the welfare of those who were engaged in religious activities and
learning. He also looked after the judicial department and in that capacity supervised the
works of the Qazis

Sarkar
 At the level of Sarkar, there were two important functionaries, the Faujdar and the
Amalguzar.

 Faujdari : Administrative division → maintain law and order


Contact : https://t.me/EN123upsc || Telegram Link : https://t.me/prelimbits
19

 Sarkar : Territorial and revenue division → maintain all accounts and send the daily
receipt to the provincial Diwan.

Pargana
 At the level of Pargana, the Shiqdar was the executive officer. He assisted the Amil in the
task of revenue collection.

 The Quanungo kept all the records of land in the Pargana.

 The Kotwals were appointed mainly in towns by the imperial government and were in
charge of law and order.

Village
 Muqaddam : Head

 Patwari : looked after the village revenue records.

 Qiladar : Charge of Forts. He was in charge of the general administration of the fort and
the areas assigned in jagir to him.

 The port administration was independent of the provincial authority.

o The governor of the port was called Mutasaddi who was directly appointed by the
Emperor.

12. With reference to Indian history, which of the following statements is/are correct?
[2021]
1. The Nizamat of Arcot emerged out of Hyderabad state.
2. The Mysore Kingdom emerged out of the Vijayanagara empire.
3. Rohilkhand kingdom was formed out of the territories occupied by Ahmad Shah Durrani.

Select the correct answer using the codes given below:

A. 1 and 2
B. 2 only
C. 2 and 3
D. 3 only

Ans. B

 The Nizam of Arcot emerged as an independent state after the weakening of the
Mughal empire in the first half of the eighteenth century. It was a protectorate of the
Hyderabad state, but it did not emerge out of it.
 Mysore kingdom emerged as a powerful state under the Wodeyar dynasty, which initially
served as a vassal of the Vijaynagar empire.
 Rohilkhand state arose in the modern districts of Rampur, Bareilly etc. under the
declining Mughal empire in 1721
o Ahmad Shah Durrani started invading India in 1748.
Contact : https://t.me/EN123upsc || Telegram Link : https://t.me/prelimbits
20

CARNATIC SULTANATE (ARCOT)


 The Carnatic Sultanate was a kingdom in South India between about 1690 and 1855, and
was under the legal purview of the Nizam of Hyderabad, until their demise.
 They initially had their capital at Arcot in the present-day Indian state of Tamil Nadu.
 Their rule is an important period in the history of the Carnatic and Coromandel Coast
regions, in which the Mughal Empire gave way to the rising influence of the Maratha
Empire, and later the emergence of the British Raj.
 With the decline of Vijayanagara Empire in 1646, the Hindu viceroys Nayaks, established
in Madurai, Tanjore and Kanchi made themselves independent, only in their turn to
become tributary to the kings of Golconda and Bijapur, who divided the Carnatic between
them.
 Mughal Emperor Aurangzeb in 1692 appointed Zulfikhar Ali Khan as the first subahdar
of the Carnatic with his seat at Arcot as a reward for his victory over the Marathas led by
Rajaram.
 With the decline of the Mughal empire, Carnatic subah became an
independent sultanate.
 The Carnatic Sultanate controlled a vast territory south of the Krishna river. The
Nawab Saadatullah Khan I (1710–1732) moved his court from Gingee to Arcot.
 His successor Dost Ali (1732–1740) conquered and annexed Madurai in 1736.
 In 1740, the Maratha forces descended on Arcot. They attacked the Nawab, Dost Ali
Khan, in the pass of Damalcherry. In the war that followed, Dost Ali, one of his sons Hasan
Ali, and a number of prominent persons lost their lives. This initial success at once
enhanced Maratha prestige in the south.
 From Damalcherry the Marathas proceeded to Arcot, which surrendered to them
without much resistance.
 The growing influences of the English and the French and their colonial wars had a huge
impact on the Carnatic.
 Muhammad Ali Khan Wallajah (1749–1795) became the ruler in 1765.
 Wallajah supported the English against the French and Hyder Ali, placing him heavily in
debt.
 As a result, he had to surrender much of his territory to the East India Company
 The thirteenth Nawab, Ghulam Muhammad Ghouse Khan (1825–1855), died without
issue, and the British annexed the Carnatic Nawabdom, applying the doctrine of lapse.
 Ghouse Khan's uncle Azim Jah was created the first Prince of Arcot (Amir-e-Arcot) in 1867
by Queen Victoria, and was given a tax free-pension in perpetuity.

Contact : https://t.me/EN123upsc || Telegram Link : https://t.me/prelimbits


21

MYSORE

 Was located south of Hyderabad at the junction of Eastern and Western Ghats.
o Modern day Karnataka, some parts of Tamil Nadu and Kerala
 It was originally a viceroyalty under the Vijayanagara Empire in the 16th century.
 The Kingdom of Mysore had preserved its fragile independence ever since the end of
Vijayanagara Empire and had been only nominally a part of the Mughal Empire.
 In the early 18th century, Mysore was ruled by Wadiyar (or Wodeyar) King Chikka
Krishnaraja.
 Two of his ministers, Nanjaraja (the Sarvadhikari or prime minister) and Devaraja (the
Dulwai or commander in chief) seized power and reduced the king to a mere puppet.
 The foundation of Mysore power was laid by HYDER ALI, which was later consolidated
by his able son, TIPU SULTAN.
 From Wadiyars to Tipu Sultan, Mysore was sought after by various powers (Marathas,
Hyderabad, Carnatic and the English) who turned it into a constant battlefield.
THE AFGHANS (AT FARRUKHABAD AND ROHILKHAND)
The states of Farrukhabad and Rohilkhand were a fallout of the Afghan migration in the mid-
18th century because of political disruption in Afghanistan.
Farrukhabad

 Muhammad Khan Bangash (also known as Ahmad Khan Bangash), an Afghan adventurer,
established his control over Farrukhabad (territory between present day Aligarh
and Kanpur), during the reigns of Farrukhsiyar and Mohammad Shah.
Rohilkhand

 Similarly, an Afghan (Pathan or Pashtun) soldier Daud and his son Ali Muhammad Khan
carved out an independent principality for themselves.
 Ali Muhammad Khan took advantage of the collapse of authority in north India
following Nadir Shah‘s invasion to set up a petty kingdom—Rohilkhand.

 It was located at the Himalayan foothills (north-west of present state of UP,

Contact : https://t.me/EN123upsc || Telegram Link : https://t.me/prelimbits


22

between Kumaon in the north and the Ganga in the south) with its capital at
Aolan in Bareilly and later at Rampur.
Both the Rohillas and the Bangash helped Ahmad Shah Abdali during the Third Battle of
Panipat and accentuated the decline of the Mughals. The Rohillas themselves suffered heavily at
the hands of other powers in the area—the Jats and the Awadh rulers, and later the Marathas and
the British.

13. Consider the following events in the history of India: [2020]

1. Rise of Pratiharas under King Bhoja


2. Establishment of Pallava power under Mahendravarman - I
3. Establishment of Chola Parantaka - I
4. Pala dynasty founded by Gopala
What is the correct chronological order of the above events, starting from the earliest time?
A. 2-1-4-3
B. 3-1-4-2
C. 2-4-1-3
D. 3-4-1-2
Ans. C
 Mahendra Varrnan-1 (590- 630 CE)
 Gopala (750 CE)
 King Bhoja- Pratihara( 836-885 CE)
 Prantaka-1 Chola ( 907-953 CE)
MAHENDRAVARMAN I (590−630 CE)
 During his reign, conflict between the Chalukyas of Badami and the Pallavas heightened and
he was defeated by Pulekshin II at Pullalur (near Kanchi) who annexed the northern part
of the Pallava kingdom.
 Was a great patron of arts, was himself a poet and musician.
 He wrote the Mattavilasa Prahasanna and initiated the construction of the famous
cave temple at Mahabalipuram.
 He bore many titles, Vichitrachitta is one of them.
 Was earlier a Jaina but later took up Shaivism under the influence of Appar.
GOPALA (AROUND 750 CE)
 As per Khalimpur copper inscription of Dharampala, he founded Pala dynasty, replacing
later Guptas of Magadha and Khadga dynasty.

Contact : https://t.me/EN123upsc || Telegram Link : https://t.me/prelimbits


23

 Built famous Odantpuri monastery at Bihar, 2nd largest after Nalanda university
BHOJA I/ MIHIR BHOJA (836 885)
 Greatest ruler of Pratiharas, got victories over Palas and Rashtrakutas.
 Capital at Kannauj, which also known as Mahodaya.
 Barah copper plate inscription, there is mention his military camp i.e., skandhavara at
Mahodaya.
 Called King Baura by Al-Masudi & Abu suleman called him king Juzr. According to Arab
travellers, the Pratihara rulers had the best cavalry in India.
 Devotee of Vishnu, adopted title Adivaraha.
 He made Teli mandir Gwalior Dedicated to Shiva, Vishnu and Matrikas,
Parantaka Chola I
 Parantaka Chola I (873 CE–955 CE) was a Chola emperor who ruled for forty-eight years,
annexing Pandya by defeating Rajasimhan II
 The best part of his reign was marked by increasing success and prosperity.

14. With reference to the history of India, consider the following pairs: [2020]
1. Aurang: In-charge of the treasury of the State
2. Banian: Indian agent of the East India Company
3. Mirasidar: Designated revenue payer to the State

Which of the pairs given above is/are correctly matched?

A. 1 and 2 only
B. 2 and 3 only
C. 3 only
D. 1, 2 and 3

Ans. B

AURANG
 Was a Persian term referred to as a warehouse.
 It is a place where goods are collected before being sold.
 The term also refers to a workshop.

BANIAN
 The term banian means merchant.
 The banian was engaged by an individual western merchant or a firm to work for them as a
broker, interpreter, and agent.

MIRASIDARS
 Under the ryotwari settlement system, the government recognized mirasidars as the sole

Contact : https://t.me/EN123upsc || Telegram Link : https://t.me/prelimbits


24

proprietors of land.
 They acted as the Designated revenue payer to the State.

15. Who among the following Mughal Emperors shifted emphasis from illustrated
manuscripts to the album and individual portrait? [2019]
A. Humayun
B. Akbar
C. Jahangir
D. Shah Jahan

Ans. C

JAHANGIR
 Jahangir was fascinated with art and architecture. In his autobiography, the Jahangirnama,
Jahangir recorded events that occurred during his reign, descriptions of flora and fauna
that he encountered, and other aspects of daily life, and commissioned court painters
such as Ustad Mansur to paint detailed pieces that would accompany his vivid prose.
 For example, in 1619, he put pen to paper in awe of a royal falcon delivered to his court
from the ruler of Iran: ―What can I write of the beauty of this bird's colour? It had black
markings, and every feather on its wings, back, and sides was extremely beautiful,‖ and then
recorded his command that Ustad Mansur paint a portrait of it after it perished.
 Jahangir bound and displayed much of the art that he commissioned in
elaborate albums of hundreds of images, sometimes organized around a
theme such as zoology.
 Jahangir took his connoisseurship of art very seriously. He also preserved paintings from
Emperor Akbar's period.
 An excellent example of this is the painting done by Ustad Mansur of Musician Naubat
Khan, son in law of legendary Tansen.
 In addition to their aesthetic qualities, paintings created under his reign were closely
catalogued, dated and even signed, providing scholars with fairly accurate ideas as to
when and in what context many of the pieces were created.
 Jahangir himself was far from modest in his autobiography when he stated his prowess at
being able to determine the artist of any portrait by simply looking at a painting.

16. With reference to the Mian Tansen, which one of the following statements is not
correct [2019-I]
A. Tansen was the title given to him by Emperor Akbar.
B. Tansen composed Dhrupads on Hindu gods and goddesses.
C. Tansen composed songs for his patrons.
D. Tansen invented many Ragas.

Contact : https://t.me/EN123upsc || Telegram Link : https://t.me/prelimbits


25

Ans. A

AKBAR‘S NAVRATNAS
1. Abul Fazl - wrote Ain-i-Akbari and ―Akbar Nama‖.

2. Abdul Rahim Khan-I-Khana - Great poet, Translated Baburnama into Turki

3. Birbal - Was a Hindu advisor and main commander (mukhya senapati) of army in the
court. Akbar gave him title ‘Raja Birbal‘.

4. Mirza Aziz Koka - also known as Kotaltash, was the foster brother of Akbar built by Mirza
Aziz Koka as a mausoleum for himself, at the time of Jahangir‘s reign

5. Faizi - Translated Lilavati into Persian & under his supervision Mahabharata was
translated into Persian language.

6. Raja Man Singh - Was a mansabdar.

7. Raja Todar Mal - Finance Minister. Akbar bestowed on him title of Diwan-i- Ashraf.

8. Faqir Azio-Din - one of the Chief advisors of Akbar

9. Tansen - Musician, Hindu of Gwalior. He served to King Ramachandra. Tansen was the
title given to him by Raja Vikramjit of Gwalior. Akbar gave him title of ―Mian‖.

TANSEN
 He was born as Ramatanu & Also referred to and commonly known as Sangeet Samrat
(lit. 'Monarch of Music') , was a Hindustani classical musician.
 Born in a Hindu Gaur Brahmin family, he learnt and perfected his art in the northwest
region of modern Madhya Pradesh.
 He began his career and spent most of his adult life in the court and patronage of the
Hindu king of Rewa, Raja Ramchandra Singh (r.1555–1592), where Tansen's musical
abilities and studies gained widespread fame
 This reputation brought him to the attention of the Mughal Emperor Akbar, who sent
messengers to Raja Ramchandra Singh, requesting Tansen to join the musicians at the
Mughal court.
 Tansen did not want to go, but Raja Ramchandra Singh encouraged him to gain a wider
audience, and sent him along with gifts to Akbar.
 In 1562, about the age of 60, the Vaishnava musician Tansen joined the Akbar's court,
and his performances became a subject of many court historians.
 Akbar considered him one of the Navaratnas (the nine jewels), and gave him the title
Mian, an honorific, meaning learned man.
 Tansen was a composer, musician and vocalist, to whom many compositions have been
attributed in northern regions of the Indian subcontinent.
 He was also an instrumentalist who popularized and improved musical instruments. He

Contact : https://t.me/EN123upsc || Telegram Link : https://t.me/prelimbits


26

is among the most influential personalities in North Indian tradition of Indian classical
music, called Hindustani.
 Tansen is remembered for his epic Dhrupad compositions, creating several new ragas,
as well as for writing two classic books on music Sri Ganesh Stotra and Sangita Sara.
 He composed many dhrupads on Ganesha, Shiva, Parvati and Rama. He also composed
songs on his patrons. https://www.thehindu.com/entertainment/music/the-legend-of-mian-
tansen/article22893454.ece
 Kalpadruma is a compliation of 300 of his dhrupads that were in Gauhar Bani. Tansen
composed in his favourite ragas — Multani, Bhairavi and Todi .
 He invented the night raga Darbari Kanhra, morning raga Mian Ki Todi, mid-day raga, Mian
ki Sarang, seasonal raga Mian ki Malhar. His descendants and disciples are called Seniyas.

17. Consider the following statements : [2019]


1. Saint Nimbarka was a contemporary of Akbar.
2. Saint Kabir was greatly influenced by Shaikh Ahmad Sirhindi.

Which of the statements given above is/ are correct?

A. 1 only
B. 2 only
C. Both 1 and 2
D. Neither 1 nor 2

Ans. D

Saint Nimbarka & Akbar

 He was the preacher of Vaishnavite Bhakti in the Telangana region.


 Saint Nimbarka was not a contemporary of Akbar.
 Saint Nimbarka belongs to the 13th or 14th century while Akbar reigned during the
16th century.

Saint Kabir & Shaikh Ahmad Sirhindi

 Kabir was the disciple of Ramananda.


 Saint Kabir was the follower of Nirguna Bhakti who lived in the 15th century while Shaikh
Ahmad Sirhindi was the Islamic scholar of Naqshbandi Silsilah who lived during the
16th century.

NIMBARKA
 Nimbarka is thought to have lived in the 11th and 12th centuries CE, but this date has
been challenged, suggesting that he lived in the 6th or 7th century CE, a little earlier than
Shankaracharya.
 He established 'Dvaitadvaita,' or dualistic monism. He wrote a commentary on the Brahma
Sutras called Vedanta-Parijata-Saurabha.
Contact : https://t.me/EN123upsc || Telegram Link : https://t.me/prelimbits
27

 He was Ramanuja's younger contemporary who advocated the Dvaita


Advaita philosophy and the Bheda Abheda (difference/non-difference) philosophy.
 Like Vishista Advaita, the Bheda Abheda philosophy holds that the world and Brahman
are both equally real and that the universe is a component of Brahman. The only difference
is in emphasis.
 He was a Vaishnavite Bhakti preacher in the Telangana region.
 He was a Radha-Krishna devotee who lived in Mathura.
 He mentioned five ways to attain salvation: Karma, Vidya (not an independent
component), Dhyana (meditation), Prapatti (devotion), and Gurupasatti (devotion and
self-surrender to guru).
 Thus, Nimbarka emphasized bhakti-yoga, or devotional and faith-based yoga.
 Many publications were published about this once-popular religion, but most of them were
burned by Muslims during the reign of the Mughal emperor Aurangzeb (1659–1707),
therefore little information about Nimbarka and his followers has remained.
 Nimbarka denies the idea of materiality and souls as God's body. If they were, God would
be subject to flaws, pain, and misery, which would be in direct opposition to God's basic
eternal and flawless essence.
 Nimbarka, regards soul and matter as 'parts' or 'powers' of God.

Bheda Abheda philosophy

 Nimbarka's philosophical position is known as bhedabheda (dvaitadvaita), which translates


as simultaneous duality and non-duality.
 He, like Ramanuja, divides existence into three categories: God (Isvara), souls (cit),
and stuff (acit).
 God (Isvara) exists on His own, but the existence of souls and matter is dependent on God.
 Souls and matter have traits and capacities that are distinct from God (Isvara), but they are
not distinct from God because they cannot exist apart from Him.
 The terms "different" and "duality" relate to the separate yet dependent existence of soul
and matter (para-tantra-Satta-bhava), whereas "non-difference" and "non-
duality" signify that soul and matter cannot exist independently of God (svatantra-Satta-
bhava).
 Nimbarka sees the interaction between Brahman, souls (cit), and the universe (acit) as a
natural difference-non-difference (svabhavika-bhedabheda) relationship, similar to the
relationship between the sun and its rays or a snake and its coil.
 Brahman (God), souls (cit), and matter or the cosmos (acit), according to Nimbarka, are
three equally existent and co-eternal realities.
 The Controller (niyantra) is Brahman, the enjoyer (bhokte) is the soul, and the thing
enjoyed is the material cosmos (bhogya).
 Nimbarka Sampradaya

Nimbarka Sampradaya
Contact : https://t.me/EN123upsc || Telegram Link : https://t.me/prelimbits
28

 One of the four Vaishnava Sampradayas is the Nimbarka Sampradaya, also known as
the Hamsa Sampradaya and the Sanakadi Sampradaya.
 Nimbarka, a Telugu Brahman, yogi, and philosopher, created it and taught the Vaishnava
bhedabheda theology of Dvaitadvaita (dvaita-advaita), also known as dualistic non-
dualism.
 According to Dvaitadvaita, humanity is both distinct and indistinguishable from Isvara, God,
or Supreme Being.
 This sampradaya is specifically associated with Krishnaism - Krishna-centric traditions.

KABIR DAS (1440 - 1510 AD)


 Born in Varanasi. He was from a community of ‗lower caste‘ weavers of the Julaha caste, a
group that had recently converted to Islam.
 Disciple of Ramananda

 He was the first to reconcile Hinduism and Islam.

 He was a Nirguna saint and openly criticised the orthodox ideas of major religions like
Hinduism and Islam.

 Denounced idol-worship, fastings, pilgrimages and religious superstitious beliefs, bathing in


holy rivers, formal worship like nama.

o He did not only target the rituals and practices of both Hinduism and Islam, but
also dismissed the sacred authority of their religious books, the Vedas and the
Quran.

 His poems are called ―Banis‖ (utterances) or ‗Dohe‘. His works are compiled in the famous
book ―Bijak‖.

 His Verses are found in the Sikh holy scripture, Adi Granth

 Kabir says: ―Ram Rahim are same‖.

Kabir‘s own humble origins and his radical message of egalitarianism fostered a community of his
followers called the Kabir Panth.

It was established in India between 1600 and 1650, one or two centuries after his death.

All regard Kabir as their guru and treat the Bijak as their holy scripture.

THE BHAKTI SAINTS WERE DIVIDED INTO TWO SCHOOLS DEPENDING ON THE WAY THEY IMAGINED
GOD
 Believe invisible formless god, without attributes.
Nirgun
 Nirguna bhakta's poetry was Jnana-shrayi or had roots in knowledge.

 Nirgun Saints:

Contact : https://t.me/EN123upsc || Telegram Link : https://t.me/prelimbits


29

1. Guru Nanak
2. Ravidas

3. Kabir

 Believe God with form & attributes.


 Saguna bhakta's poetry was Prema-shrayi, or with roots in love.[
Sagun
 Saguna Saints:

1. Tulsidas
2. Surdas

3. Meerabai

18. Consider the following statements: [2019]


1. In the revenue administration of Delhi Sultanate, the in-charge of revenue collection was
known as 'Amil'.
2. The Iqta system of Sultans of Delhi was an ancient indigenous institution.
3. The office of Mir Bakshi' came into existence during the reign of Khalji Sultans of Delhi.

Which of the statements given above is/are correct?

A. 1 only
B. 1 and 2 only
C. 3 only
D. 1, 2 and 3

Ans. A

AMILS
 They were officers in charge of Collecting Revenue during the sultanate era in North India.
 Akbar had even directed the Amils to extend loans to the peasants in times of need and
encourage them to show high-quality seeds.
 Qanungo were the officials incharge of maintaining revenue records.

AKBAR (MUGHAL EMPIRE)→ CENTRAL ADMINISTRATION → MIR BAKSHI

 The head of the military administration was Mir Bakshi who was also considered as head
of the nobility.

 The Mir Bakshi was also the head of the intelligence and information agencies of the
empire. Barids (intelligence officers) and Waqia-navis (news reporters) were posted to
all the parts of the empire.

 Their reports were presented to the emperor at the court through Mir Bakshi.

Contact : https://t.me/EN123upsc || Telegram Link : https://t.me/prelimbits


30

IQTA SYSTEM

 The Iqtadari was a unique type of land distribution and the administrative system
that evolved during the sultanate of Iltutmish.
 Under this system, the entire empire was very evenly divided into several large and small
tracts of land, called the Iqtas.
 These plots of land were assigned to the various nobles, officers, and soldiers FOR the
purpose of easy and Flawless Administration and Revenue Collection.
 The Iqtas were transferable, i.e., the holders of Iqtas-Iqtadars-were transferred from one
region to another every three to four years.
 The holders of small Iqtas were individual troopers. They had no administrative
responsibilities.
 Muhammad of Ghuri in 1206 A.D. the able king was the first to introduce the Iqta
system in India, BUT it was lltutmish who gave it an institutional form.
 The Iqta system was abolished by Alauddin Khilji.
 However, during Firuz Shah Tughlaq's reign, in the year 1351 A,D, it became hereditary
 It was Originally of Central Asian and West Asian origin and was NOT an ancient
indigenous institution

19. With reference to Mughal India, what is/are the difference/ differences between
Jagirdar and Zamindar? [2019]
1. Jagirdars were holders of land assignments in lieu of judicial and police duties, whereas
Zamindars were holders of revenue rights without obligation to perform any duty other
than revenue collection.
2. Land assignments to Jagirdars were hereditary and revenue rights of Zamindars were not
hereditary.

Select the correct answer using the code given below.

A. 1 only
B. 2 only
C. Both 1 and 2
D. Neither 1 nor 2

Ans. D

AKBAR (1556-1605) → THE JAGIRDARI & MANSABDARI SYSTEM


Mansabdari

 Akbar introduced the Mansabdari system in his administration. Under this system
every officer was assigned a rank (Mansab).

 The word ―Mansabdar‖ used for all but it had 3 scale gradation –

o Mansabdar – (500 zat / below it)

Contact : https://t.me/EN123upsc || Telegram Link : https://t.me/prelimbits


31

o Amir (between - 500-2500 zat)

o Amir-i-Umda – (above 2500 zat)

 The ranks were divided into two – zat and sawar.

o Zat fixed the personal status & salary of a person.

o Sawar rank indicated the number of sawars had to maintained by person. Every
sawar had to maintain at least two horses.

 The mansab rank was not hereditary & mansabdar were paid by assigning Jagir.

 Most mansabdars did not reside in their own jagirs but used servants to collect revenues
there while they themselves served in another part of the empire.
The Delhi Sultanate (The Khalji and the Tughlaq monarchs) too appointed military commanders
as governers of territories. These land were called the Iqta and the landholders were called
Iqtadars or the muqti. Most of the Muqti stayed in their Iqta unlike Jagirdars.

Jagirdari System

 Iqta of the Sultanate period in a modified form became Jagir under the Mughals.

 The Jagirdari system was the system of assignment of revenue of a particular


territory to the nobles for their services to the state was an integral part of the
Mansabdari system.

 There were various types of jagirs:


o Tankha Jagirs: Which were given in lieu of salaries and they were transferable
every three to four years.
o Mashrut Jagirs: Which were given on certain conditions
o Watan Jagirs: Which were assigned to zamindar or rajas in their local dominions.
Watan Jagirs were hereditary and nontransferable.
o Altamgha Jagirs: Which were given to Muslim nobles in their family towns or
place of birth.

ZAMINDARS

 Along with Jagirdars, there were zamindars who were the people who had hereditary
rights over the produce of the land and claimed a direct share in the peasants produce.
 Zamindars also assisted the state and the jagirdar in the collection of land revenue. They
had their own armed forces, and generally lived in forts or grahis, which was both a place
of refuge and a status symbol.
 It should be noted that the zamindar was not the owner of all the lands
comprising his zamindari. The peasants who actually cultivated the land could not
be dispossessed as long as they paid the land revenue. Thus, the zamindars and the
peasants had their own hereditary rights in land.

Contact : https://t.me/EN123upsc || Telegram Link : https://t.me/prelimbits


32

20. The well-known painting "Bani Thani" belongs to the [2018-I]


A. Bundi school
B. Jaipur school
C. Kangra school
D. Kishangarh school

Ans. D

BANI THANI

 Bani Thani refers to an Indian miniature painting painted by Nihal Chand from the
Marwar school of Kishangarh.
 It portrays a woman who is elegant and graceful.
 The painting's subject, Bani Thani, was a singer and poet in Kishangarh in the time of
king Sawant Singh (1748–1764).

 Inspired by Radha, Bani Thani is characterized by idealized distinct features


such as arched eyebrows, lotus-like elongated eyes, and pointed chin.
 This painting was featured in an Indian stamp issued on 5 May 1973.
 The Kishangarh school of art is notable for its elongated style, with ―arched eyebrows,
lotus-like elongated eyes and pointed chin‖ a highly idealized facial form reminiscent
of Indian sculpture art.
 Kishangarh was influenced by Bundi painting in its use of lush vegetation, dramatic night
skies, vivid movement, and Mughal painting in its use of side-profile portraits.

BUNDI SCHOOL
 The Bundi School is an important school of the Rajasthani style of Indian miniature
painting that lasted from the 17th to the end of the 19th century in this princely state of
Bundi ruled by Hadoti Rajputs.
 One of the earliest examples of the Bundi Paintings is the Chunar Ragamala painted in 1561.
Bundi paintings emphasized on hunting, court scenes, festivals, processions, life of
nobles, lovers, animals, birds and scenes from Lord Krishna's life.
 The Bundi School had a close association with the Mughal style yet it was never
fundamental to the evolution and growth of Bundi paintings, however the delicacy of the

Contact : https://t.me/EN123upsc || Telegram Link : https://t.me/prelimbits


33

Mughal style was also not abandoned.


 The blend of Mughal and Deccani art elements in Bundi style are unique. Bundi paintings
excelled in its illustrations of Krishna-Lila and Rasikapriya. Besides these the other
themes that these paintings depict are Nayaka- Nayika Bheda, Baramasa, Harem or Zanana,
Hunting and Portraits.
 These miniature paintings are crafted with meticulous details and brushwork .These vibrant
works of art are painted with colors that are essentially made from minerals and vegetables,
precious stones, as well as pure silver and gold. They showcase the flora and fauna of the
beauty of the Bundi forests with Ragini as the central theme.

JAIPUR SCHOOL OF PAINTING


 The Jaipur School of painting originated in its former capital Amer, which was nearest of all
large Rajput states to Mughal capitals — Agra and Delhi.
 Rulers of Jaipur from the earliest times maintained cordial relations with the Mughal
emperors, who strongly influenced the aesthetics at Amer.
 Raja Bharmal (1548–1575) married his daughter to Akbar. His son Bhagwant Das (1575–
1592) was a close friend of Akbar and his son Man Singh, in turn, was Akbar‘s most trusted
general.
 Sawai Jai Singh (1699–1743), an influential ruler, established a new capital city Jaipur
named after him in 1727 and shifted from Amer.
o Jaipur School of paintings thrived under his reign and emerged as a well-defined
independent school.
o Court records reveal that some Mughal painters were brought from Delhi to
become a part of his atelier. He also invited eminent craftsmen and other artists to
settle down in Jaipur and reorganised the Suratkhana, the place where paintings
were made and stored.
o He was drawn to the Vaishnavite sect and commissioned numerous paintings on
the theme of Radha and Krishna.
o Artists during his reign painted sets based on Rasikapriya, Gita Govinda,
Baramasa and Ragamala, where the hero‘s figure is in striking resemblance with the
king.
o Portrait painting was also popular during his time and an accomplished portrait
painter, Sahibram, was part of his atelier. Muhammad Shah was another artist.
 Sawai Ishwari Singh (1743–1750) extended the same patronage to art. Apart from religious
and literary texts, he got scenes of his leisure pursuits painted, such as elephant rides, boar
and tiger hunts, elephant fights, and so on.
 Sawai Madho Singh (1750–1767) was attracted towards getting incidents of his court life
recorded.
 It was only in the eighteenth century, under the aspiration of Sawai Pratap Singh (1779–
1803) that the predominant Mughal influence receded and a Jaipur style with
Contact : https://t.me/EN123upsc || Telegram Link : https://t.me/prelimbits
34

reformulated aesthetics, which was a blend of Mughal and indigenous stylistic features
emerged.
o This was a second thriving period for Jaipur and Pratap Singh employed around 50
artists.
o He was a scholar, poet, prolific writer and an ardent follower of Krishna.
o During his time, apart from royal portraits and representations of courtly pomp and
splendour, literary and religious themes, such as Gita Govinda, Ragamala, Bhagvata
Purana, etc., got renewed stimulus.

KANGRA MINIATURE PAINTING


 By the early nineteenth century, there was a lavish use of gold. Jaipur preferred large size
formats and produced life-size portraits.
 Kangra Miniature painting is one of the schools of Pahari paintings along with Guler,
Basholi, Mandi, Chamba and Bilaspur.
 Miniatures are small-sized paintings, generally done in watercolor on cloth or paper.

 Nadir Shah‘s invasion and conquest of Delhi in 1739 resulted in the migration of
Mughal artists to the hills to escape the uncertainties of the plains.
o These artists found ready patrons in the hills which led to the founding of the Kangra
school of painting.
o By the mid eighteenth century the Kangra artists developed a new style wherein the
source of inspiration was the Vaishnavite traditions. Soft colours including cool blues
and greens, and a lyrical treatment of themes distinguished Kangra painting.
 The name Kangra style is given to this group of painting for the reason that they are
identical in style to the portraits of Raja Sansar Chand of Kangra.
 Salient features of this school of art →
o The delicacy of drawing and quality of naturalism are the stand-out features of
this school of art
o The Kangra style is by far the most poetic and lyrical of Indian styles marked with
serene beauty and delicacy of execution.
o Distinctive is the delineation of the female face, with a straight nose in line with
the forehead, which came in vogue around the 1790s, is the most distinctive feature
of this style.
o Most popular themes that were painted were the Bhagvata Purana, Gita Govinda,
Nala Damayanti, Bihari Satsai, Ragamala and Baramasa
Contact : https://t.me/EN123upsc || Telegram Link : https://t.me/prelimbits
35

21. With reference to the cultural history of India, consider the following statements
[2018]
1. White marble was used in making Buland Darwaza and Khankah at Fatehpur Sikri.
2. Red sandstone and marble were used in making Ilara Imambara and Rumi Darwaza at
Lucknow.

Which of the statements given above is/are correct?

A. 1 only
B. 2 only
C. Both 1 and 2
D. Neither 1 nor 2

Ans. D (According to official Ans. key) BUT →

 Buland Darwaza (lit. 'High Gate'), or the "Door of Victory", was built in 1602 by Mughal
emperor Akbar to commemorate his victory over Gujarat.
 It is the main entrance to the Jama Masjid at Fatehpur Sikri, which is 43 km from Agra,
India.
 Buland Darwaza is the highest gateway in the world and is an example of Mughal
architecture. It displays sophistication and heights of technology in Akbar's empire.
 The Buland Darwaza is made of red and buff sandstone, decorated by white and
black marble and is higher than the courtyard of the mosque.

BARA IMAMABARA
 It was built by Asaf-ud-Daula, Nawab of Awadh in 1784 and is located in Lucknow.
 There is no woodwork in the entire structure.
 The roof has been put together with interlocking bricks without using a beam or a girder.

RUMI DARWAZA
 It was built in the year 1784 by Nawab Asaf-ud-daula and is located in Lucknow.
 The material used for the darwaza is bricks and its then coated with lime.

Contact : https://t.me/EN123upsc || Telegram Link : https://t.me/prelimbits


36

22. With reference to the cultural history of India consider the following statements [2018]
1. Most of the Tyagaraja Kritis are devotional songs in praise of Lord Krishna
2. Tyagaraja created several new rages
3. Annamacharya and Tyagaraja are contemporaries
4. Annamacharya kirtanas are devotional songs in praise of Lord Venkateshwara

Which of the statements given above are correct?

A. 1 and 3 only
B. 2 and 4 only
C. 1, 2 and 3
D. 2, 3 and 4

Ans. B

 Tyagaraja was born on 14th May 1767 in the village of Thiruvayaru in the Thanjavur
district of Tamil Nadu whereas TaḷḷapākaAnnamācārya (or Annamayya) (22 May 1408 –
4 April 1503) was a 15th-century Hindu saint.
 Tyagaraja was an ardent devotee of lord Rama. In his praise and honour he wrote
numerous musical operas, and about twenty four thousand songs, a claim that has been
speculated among music historians.
 Tyagaraja‘s songs and compositions are full of devotion to his god Rama.
 Tyagaraja introduced several unusual ragas in Carnatic music.
 Annamacharya kirtanas are devotional songs in prais of Lord Venkateshwara.
 TaḷḷapākaAnnamācārya (or Annamayya) is the earliest known Indian musician to compose
songs called sankirtanas in praise of the god Venkateswara, a form of Vishnu.

23. Which one of the following was a very important seaport in the Kakatiya kingdom?
[2017]

A. Kakinada
B. Motupalli
C. Machilipatnam (Masulipatnam)
D. Nelluru
Ans. B

KAKATIYA DYNASTY
 The Kakatiya dynasty ruled most of eastern Deccan region comprising present day
Telangana and Andhra Pradesh, and parts of eastern Karnataka and southern Odisha
between 12th and 14th centuries.

 Their capital was Orugallu, now known as Warangal.

Contact : https://t.me/EN123upsc || Telegram Link : https://t.me/prelimbits


37

 Early Kakatiya rulers served as feudatories to Rashtrakutas and Western Chalukyas for
more than two centuries.
 They assumed sovereignty under Prataparudra I in 1163 CE by suppressing other
Chalukya subordinates in the Telangana region.

 Ganapati Deva (r. 1199–1262) significantly expanded Kakatiya lands during the
1230s and brought under Kakatiya control the Telugu-speaking lowland delta areas around
the Godavari and Krishna rivers.

 Motupalli, now in Krishna district, was an important seaport in Ganapati Deva reign.

 Ganapati Deva was succeeded by Rudrama Devi (r. 1262–1289)


who is one of the few queens in Indian history.

o Marco Polo, who visited India around 1289–1293, made


note of Rudrama Devi's rule and nature in flattering
terms. She successfully repelled the attacks of Yadavas
(Seuna) of Devagiri into the Kakatiyan territory.

 In 1303, Alauddin Khilji, the emperor of the Delhi Sultanate


invaded the Kakatiya territory which ended up as a disaster for the
Turks.
 But after the successful siege of Warangal in 1310, Prataparudra II was forced to pay
annual tribute to Delhi.
 Another attack by Ulugh Khan in 1323 saw stiff resistance by the Kakatiyan army, but they
were finally defeated.
 Kakatiya era the development of a distinct style of architecture which improved and
innovated upon the existing modes. Most notable examples are the Thousand Pillar
Temple in Hanamkonda, Ramappa Temple in Palampet, Warangal Fort, and Kota Gullu
in Ghanpur.
Prataparudra I
 The 1149 Sanigaram inscription of Prola II is the last known record of the Kakatiyas as
vassals.

 The 1163 Anumakonda inscription of Rudradeva alias Prataparudra I is the earliest


known record that describes the Kakatiyas as a sovereign power

24. Regarding the taxation system of Krishna Deva, the ruler of Vijayanagar, consider the
following statements: [2016]
1. The tax rate on land was fixed depending on the quality of the land.
2. Private owners of workshops paid an industries tax.

Which of the statements given above is/are correct?

A. 1 only
Contact : https://t.me/EN123upsc || Telegram Link : https://t.me/prelimbits
38

B. 2 only
C. Both 1 and 2
D. Neither 1 nor 2

Ans. C

VIJAYANAGAR EMPIRE → ECONOMY


 According to the accounts of the foreign travelers, the Vijayanagar kingdom was one of
the wealthiest kingdoms of the world at that time.

 Agriculture continued to be the chief occupation of the people.


o The Vijayanagar rulers provided a stimulus to its further growth by providing
irrigation facilities. New tanks were built and dams were constructed across the
rivers like Tungabhadra.

o Nuniz refers to the excavation of canals.

 There were numerous industries and they were organised into guilds. Metal workers and
other craftsmen flourished during this period. Diamond mines were located in Kurnool
and Anantapur district.

 The chief gold coin was the varaha but weights and measures varied from place to place.
Inland, coastal, and overseas trade led to the general prosperity.

 There were a number of seaports on the Malabar coast, the chief being
Cannanore.

 Commercial contacts with Arabia, Persia, South Africa, and Portugal on the west,
and with Burma, the Malay peninsulam and China on the east flourished.

o The chief items of Exports : cotton and silk clothes, spices, rice, iron,
saltpetre, and sugar.

o The Imports consisted : horses, pearls, copper, coral, mercury, China silk, and
velvet clothes.

 The art of shipbuilding had developed.

Taxation system → According to Abdur Razzaq

 Vijayanagara rulers followed an oppressive taxation policy.

 The prosperity of the empire can be explained through their taxation principles.

 The Land Tax was the major source of revenue during the period of the Vijayanagar
kingdom.

 During the reign of Krishna Devaraya the rate was fixed between one third and one-sixth
depending on the quality of the land.
Contact : https://t.me/EN123upsc || Telegram Link : https://t.me/prelimbits
39

 Besides land tax, there were also taxes imposed on shopkeepers, farm servants,
workmen, posters, shoemakers, musicians etc.

 Grazing and house taxes were also imposed.

 Commercial taxes consisting of levies, duties and customs on manufactured articles of


trade were also levied.

 Private owners of workshops also paid an industry tax.

25. With reference to the cultural history of medieval India, consider the following
statements : [2016]
1. Siddhas (Sitters) of the Tamil region were monotheistic and condemned idolatry.
2. Lingayats of the Kannada region questioned the theory of rebirth and rejected the caste
hierarchy.

Which of the statements given above is/are correct?

A. 1 only
B. 2 only
C. Both 1 and 2
D. Neither 1 nor 2

Ans. C

SIDDHAS / SIDDHAR / SITTAR / CITTAR


 Tamil siddha is an unorthodox religious philosophy in the traditions of India that is
associated with mysticism. The word siddha derives from the Sanskrit root sidh, meaning
―being accomplished or fulfilled.‖
o According to Ramakrishna Rao (2010), Siddhas, Siddhar, Sittar and Cittar, are the
words variously used in English by the Western and as well as Indian scholars but
each as specific connotation in Tamil.
 Siddhas means the refined monotheistic creed existed.
 Siddhar means perfected or realized saints.
 Sittar implies an expert in occultism, alchemy and so on with magic or
superstitious power
 Based in southern India, Tamil siddha is part of a larger siddha movement throughout South
Asia and tantric yoga. The siddha movement crosses religious lines with Hindu, Buddhist
and Jain siddhas sharing similar practices, beliefs about the universe and symbols.
 Tamil siddhas trace their origins to the 8th century. Siddhars in Tamil are saints, celestial
beings or yogis who have transcended time and space via meditation.
 The Tamil siddha philosophy is based on portions of the religious text "Tirumantiram,"
particularly the part called the "Tantrasastra," which describes Shakti worship, chakras
and magic spells. Shiva is the supreme god and Ultimate Reality, but to Tamil siddhas, the

Contact : https://t.me/EN123upsc || Telegram Link : https://t.me/prelimbits


40

goddess Shakti, the feminine aspect of Shiva, is preeminent. Shakti is the divine force
or life-force energy within the body and is, therefore, more accessible than Shiva.
 Through mysticism and yogic practices, Tamil siddhas manipulate the flow of the goddess
energy throughout the body to reach the ultimate goal of union with the Absolute and the
highest consciousness.
 Siddhas avoid temple worship and do not believe in the authority of the
Brahmin priestly class.
VIRASHAIVAS OR LINGAYATS
 The twelfth century witnessed the emergence of a new movement in Karnataka, led by a
Brahmana named Basavanna (1106-68) who was a minister in the court of a Kalachuri ruler.
His followers were known as Virashaivas (heroes of Shiva) or Lingayats (wearers of
the linga).
 Lingayats continue to be an important community in the region to date. They worship Shiva
in his manifestation as a linga, and men usually wear a small linga in a silver case on a loop
strung over the left shoulder. Those who are revered include the jangama or wandering
monks.
 Lingayats believe that on death the devotee will be united with Shiva and will not return
to this world. Therefore they do not practise funerary rites such as cremation, prescribed in
the Dharmashastras. Instead, they ceremonially bury their dead.
 The Lingayats challenged the idea of caste and the ―pollution‖ attributed to certain groups by
Brahmanas.

26. Consider the following pairs: [2016]


Famous place Region
1. Bodhgaya Baghelkhand
2. Khajuraho Bundelkhand
3. Shirdi Vidarbha
4. Nasik (Nashik) Malwa
5. Tirupati Rayalaseema

Which of the pairs given above are correctly matched?

A. 1, 2 and 4
B. 2, 3, 4 and 5
C. 2 and 5 only
D. 1, 3, 4 and 5

Ans. C

KHAJURAHO
 Khajuraho is famous for a Group Of Monuments, built between 850-1000 CE by the
Chandela dynasty.

Contact : https://t.me/EN123upsc || Telegram Link : https://t.me/prelimbits


41

 Khajuraho is a town located in the Bundelkhand region of the Indian state of Madhya
Pradesh.
 They are a UNESCO World Heritage Site. The temples are famous for their nagara-style
architectural symbolism and a few erotic sculptures.
 Of the surviving temples, the Kandariya Mahadeva Temple is decorated with a profusion
of sculptures with intricate details, symbolism and expressiveness of ancient Indian art.
 The temple complex was forgotten and overgrown by the jungle until 1838 when
Captain T.S. Burt, a British engineer, visited the complex and reported his findings in
the Journal of the Asiatic Society of Bengal

History

 The Khajuraho group of monuments was built during the rule of the Chandela dynasty.
The building activity started almost immediately after the rise of their power, throughout
their kingdom to be later known as Bundelkhand.
 Most temples were built during the reigns of the Hindu kings Yashovarman and Dhanga.
Yashovarman's legacy is best exhibited by the Lakshmana Temple.
 Vishvanatha temple best highlights King Dhanga's reign.
 The largest and currently most famous surviving temple is Kandariya Mahadeva built in
the reign of King Vidyadhara
 The Khajuraho temples were built about 35 miles from the medieval city of Mahoba, the
capital of the Chandela dynasty, in the Kalinjar region.
 In ancient and medieval literature, their kingdom has been referred to as Jijhoti, Jejahoti,
Chih-chi-to and Jejakabhukti.
 The first documented mention of Khajuraho was made in 641 by Xuanzang, a Chinese
pilgrim who described encountering several dozen inactive Buddhist monasteries and a
dozen Hindu temples with a thousand worshipping brahmins.
 In 1022 CE, Khajuraho was mentioned by Abu Rihan-al-Biruni, the Persian historian who
accompanied Mahmud of Ghazni in his raid of Kalinjar; he mentions Khajuraho as the
capital of Jajahuti. The raid was unsuccessful, and a peace accord was reached when the
Hindu king agreed to pay a ransom to Mahmud of Ghazni to end the attack and leave
 Khajuraho temples were in active use through the end of the 12th century. This changed
in the 13th century; after the army of Delhi Sultanate, under the command of the Muslim
Sultan Qutb-ud-din Aibak, attacked and seized the Chandela kingdom.
 About a century later, Ibn Battuta, the Moroccan traveller in his memoirs about his stay in
India from 1335 to 1342 AD, mentioned visiting Khajuraho temples, calling them "Kajarra"

Description

 The temple site is within Vindhya mountain range in central India. An ancient local legend
held that Hindu deity Shiva and other gods enjoyed visiting the dramatic hill formation in
Kalinjar area.

Contact : https://t.me/EN123upsc || Telegram Link : https://t.me/prelimbits


42

 The temples are clustered near water, another typical feature of Hindu temples. The
current water bodies include Sib Sagar, Khajur Sagar (also called Ninora Tal) and Khudar
Nadi (river).
 All temples, except one (Chaturbhuja) face the sunrise – another symbolic feature that is
predominant in Hindu temples.
 The relative layout of temples integrate masculine and feminine deities and symbols
highlight the interdependence.
 The artworks symbolically highlight the four goals of life considered necessary and
proper in Hinduism – dharma, kama, artha and moksha.
 Of the surviving temples, six are dedicated to Shiva, eight to Vishnu and his affinities,
one to Ganesha, one to Sun god, three to Jain Tirthankars.
 An overall examination of site suggests that the Hindu symbolic mandala design principle
of square and circles is present each temple plan and design.
 Further, the territory is laid out in three triangles that converge to form a pentagon.
Scholars suggest that this reflects the Hindu symbolism for three realms or trilokinatha,
and five cosmic substances or panchbhuteshvara.
 The temple site highlights Shiva, the one who destroys and recycles life, thereby controlling
the cosmic dance of time, evolution and dissolution.
 The temples have a rich display of intricately carved statues. While they are famous for
their erotic sculpture, sexual themes cover less than 10% of the temple
sculpture.
 The most visited temple, Kandariya Mahadev, has an area of about 6,500 square feet and
a shikhara (spire) that rises 116 feet.

Jain temples

 Parshvanath Temple
o 23rd of 24 Tirthankaras
o Chandella Dhanga as the reigning king.
o A well known early magic square is found in this temple. This is referred to as the
Chautisa (Thirty-four) Yantra, since each sub-square sums to 34.
 Adinath Temple
o First Tīrthaṅkara (Aka. Rishabhanatha, also Ṛṣabhadeva, Rishabhadeva, Ṛṣabha or
Ikshvaku)
o The Adinath Jain temple contains an idol with an inscription dated to year 1027
during the rule of Chandella king Madanavarman.
 Shantinath Temple
o Sixteenth Tirthaṅkara of Jainism
 Ghantai Temple
o The Ghantai Temple was built around 960 AD by Chandela kings of Khajuraho. This

Contact : https://t.me/EN123upsc || Telegram Link : https://t.me/prelimbits


43

temple was dedicated to lord Rishabha as Chakreshvari is seen in this temple along
with Nine planets and Gomukh ya ksha.

Architecture of the temples

 Khajuraho temples, almost all Hindu temple designs, follow a grid geometrical design
called vastu-purusha-mandala
 This design plan has three important components – Mandala means circle, Purusha is
universal essence at the core of Hindu tradition, while Vastu means the dwelling structure.
 The design lays out a Hindu temple in a symmetrical, concentrically layered, self-
repeating structure around the core of the temple called garbhagriya, where the
abstract principle Purusha and the primary deity of the temple dwell. The shikhara, or
spire, of the temple rises above the garbhagriya.

 The architecture is symbolic and reflects the central Hindu beliefs through its form,
structure, and arrangement of its parts.
 The mandapas, as well as the arts, are arranged in the Khajuraho temples in a
symmetric repeating patterns, even though each image or sculpture is distinctive in its
own way.
 All Khajuraho temples, except one, face sunrise, and the entrance for the devotee is this
east side.
 Above the vastu-purusha-mandala of each temple is a superstructure with a dome called

Contact : https://t.me/EN123upsc || Telegram Link : https://t.me/prelimbits


44

Shikhara / Kailash or Meru (or Vimana, Spire).


 In each temple, the central space typically is surrounded by an ambulatory for the pilgrim
to walk around and ritually circumambulate the Purusa and the main deity.
 The pillars, walls, and ceilings around the space, as well as outside have highly ornate
carvings or images of the four just and necessary pursuits of life – kama, artha, dharma,
and moksa. This clockwise walk around is called pradakshina.

Construction Material Used

 The Khajuraho temples are made of sandstone, with a granite foundation that is almost
concealed from view. The builders didn't use mortar
 The stones were put together with mortise and tenon joints and they were held in place
by gravity. This form of construction requires very precise joints. The columns and
architraves were built with megaliths that weighed up to 20 tons.
 The Khajuraho and Kalinjar region is home to superior quality of sandstone, which can
be carved precisely. The surviving sculpture reflect fine details such as strands of hair,
manicured nails, and intricate jewellery.

Arts and sculpture

 The Khajuraho temples feature a variety of artwork, of which 10% is sexual or erotic art
outside and inside the temples.
 Some of the temples that have two layers of walls have small erotic carvings on the outside
of the inner wall.
 Some scholars suggest these to be tantric sexual practices. Other scholars state that the
erotic arts are part of the Hindu tradition of treating kama as an essential and proper
Contact : https://t.me/EN123upsc || Telegram Link : https://t.me/prelimbits
45

part of human life, and its symbolic or explicit display is common in Hindu temples.[6][59
 The vast majority of arts depict various aspects the everyday life, mythical stories as well
as symbolic display of various secular and spiritual values important in Hindu tradition.
o For example, depictions show women putting on makeup, musicians making music,
potters, farmers, and other folks in their daily life during the medieval era.

BODH GAYA
 Bodh Gaya is a religious site and place of pilgrimage associated with the Mahabodhi
Temple Complex in Gaya district in the Indian state of Bihar.
 It is famous as it is the place where Gautama Buddha is said to have attained
Enlightenment under what became known as the Bodhi Tree.
 For Buddhists, Bodh Gaya is the most important of the main four pilgrimage sites related
to the life of Gautama Buddha, the other three being Kushinagar, Lumbini, and Sarnath.
 In 2002, Mahabodhi Temple, located in Bodh Gaya, became a UNESCO World Heritage
Site.
History

 Bodh Gaya is considered to be the holiest site in Buddhism. Known as Uruwela in the
Buddha's time, it is situated by the bank of Lilajan River. The first temple at the site was
built by King Ashoka.
 Traditionally, Buddha was born in 563 BC on the following auspicious Baisakhi purnima.
As Siddhartha, he renounced his family at the age of 29 in 534 BC, and travelled and
meditated in search of truth.
 After practicing self-mortification for six years at Urubela (Buddhagaya) in Gaya, he
gave up that practice because it did not give him Vimukthi.
 Then he discovered Noble Eight-fold path without help from anyone and practiced it,
then he attained Buddhatva or enlightenment.
 Enlightenment is a state of being completely free from lust (raga), hatred (dosa) and
delusion (moha). By gaining enlightenment, you enter Nirvana, in which the final stage is
Parinirvana.
 At this place, the Buddha was abandoned by the five men who had been his companions
of earlier austerities. Later these the five ascetics became the Buddha's first disciples in
Deer Park, Sarnath, 13 kms north east of Benares.
 The disciples of Gautama Siddhartha began to visit the place during the full moon in the
month of Vaisakh (April–May), as per the Hindu calendar. Over time, the place became
known as Bodh Gaya, the day of enlightenment as Buddha Purnima, and the tree as the
Bodhi Tree.
 The history of Bodh Gaya is documented by many inscriptions and pilgrimage accounts.
Foremost among these are the accounts of the Chinese pilgrims Faxian in the 5th century
and Xuanzang in the 7th century.

Contact : https://t.me/EN123upsc || Telegram Link : https://t.me/prelimbits


46

 The area was at the heart of a Buddhist civilization for centuries, until it was conquered by
Turkic armies in the 13th century. The place-name, Bodh Gaya, did not come into use until
the 18th century CE. Historically, it was known as Uruvela, Sambodhi (Sambodhi
Saṃ+bodhi, "Complete Enlightenment" in Ashoka's Major Rock Edict No.8), Vajrasana (the
"Diamond Throne" of the Buddha) or Mahabodhi ("Great Enlightenment").
 The main monastery of Bodh Gaya used to be called the Bodhimanda-vihāra (Pali). Now
it is called the Mahabodhi Temple.
 During the period from the 11th to 13th centuries, Bodh Gaya was under the control of
local chieftains known as the Pithipatis of Bodh Gaya who were responsible for the
management of the region. One of their rulers, Acarya
Buddhasena, was noted as making a grant to Sri Lankan
monks near the Mahabodhi temple.
Mahabodhi Temple

 The complex, located about 110 kilometres from Patna,


contains the Mahabodhi Temple with the Vajrasana or
"diamond throne" and the holy Bodhi tree.
 This tree was originally a sapling of the Sri Maha Bodhi
tree in Sri Lanka, itself grown from a what is claimed to be
a sapling of the original Bodhi tree.
 In approximately 250 BCE, about 200 years after the
Buddha attained Enlightenment, Emperor Asoka visited
Bodh Gaya in order to establish a monastery and shrine
on the holy site.
 Representations of this early temple are found at Sanchi, on the toraṇas of Stūpa I, dating
from around 25 BCE, and on a relief carving from the stupa railing at Bhārhut, from the early
Shunga period (c. 185–c. 73 BCE).
Sujata Stupa

 Across the Phalgu river is the Sujata Stupa, in the village of Bakraur. The stupa was
dedicated to the milkmaid Sujata, who is said to have fed Gautama Buddha milk and
rice as he was sitting under a Banyan tree, ending his seven years of fasting and asceticism,
and allowing him to attain illumination through the Middle Way
 The stupa was built in the 2nd century BCE as confirmed by finds of black polished wares
and punch-marked coins in the attending monastery

SHIRDI
 Shirdi is in the Ahmadnagar district of Maharashtra and is far from the Vidarbha region
and is famous for the religious personality of Sai Baba.
 Shirdi is famously known as the home of the late 19th century saint Shirdi Sai Baba.

Contact : https://t.me/EN123upsc || Telegram Link : https://t.me/prelimbits


47

NASHIK
 Nashik is in the northwest region of Maharashtra in India and is famous for Kumbh Mela.

TIRUPATI
 Tirupati is a city in the Rayalseema region of the Chittoor district of the Indian state of
Andhra Pradesh and is famous for Hindu Sri Venkateswara Temple.

27. With reference to the economic history of medieval India, the term Araghatta‟ refers to
[2016-I]
A. bonded labor
B. land grants made to military officers
C. waterwheel used in the irrigation of land
D. wasteland converted to cultivated land

Ans. C

CHOLA SOCIETY AND ECONOMY → AGRAHATTA


 Divided on caste lines: Periyar (Untouchbles), Vellallas (Farming groups).

 Emergence of Supra Caste dichotomy:

o Idangai (left hand caste groupings): Consisted mainly of artisanal and trading
groups.

o Valangai (right hand groupings): Consisted mainly of agricultural groups.

 Women were head of some villages, as per some inscriptions.

 Significant shift of royal patronage from gifts to Brahmans to gifts to Temples.

 Expansions in agrarian economy, use of Agrahatta (Persian wheel) for smooth irrigations.

Contact : https://t.me/EN123upsc || Telegram Link : https://t.me/prelimbits


48

 Emergence of various crafts centers, i.e., Kanchipuram as important weaving industry


centre, Kudamukku was center for betel nut & areca nut production.

 Emergence of trading caste: Garveras (Northern merchants migrated southwards),


Gaudas/ Gavundas (originally cultivators). Heggades who were initially revenue officials,
Kayasthas (scribes, they specialised in drafting and writing land grant documents), and
Karanas who also represented the occupational group of professional scribes.

 Shaivism & Vaishnavism flourished.

 Chola had trade relations with Java, Sumatra, Arabia & China.

 Inscriptions at Ennayiram, Thirumukkudal & Thirubuvanai gives details of education system


& colleges.

28. What is/are common to the two historical places known as Ajanta and
Mahabalipuram? [2016]
1. Both were built in the same period.
2. Both belong to the same religious denomination.
3. Both have rock-cut monuments.

Select the correct answer using the code given below.

A. 1 and 2 only
B. 3 only
C. 1 and 3 only
D. None of the statements given above is correct

Ans. B

GROUP OF MONUMENTS AT MAHABALIPURAM


 Collection of 7th- and 8th-century CE religious monuments in Tamil Nadu.
 The monuments were built during the Pallava dynasty.
 The group contains several categories of monuments:
 Ratha temples
o They are with monolithic temples carved in the shape of chariots.
o The best-known are the five monolithic structures known as the Five Rathas or the
Pandava Rathas: Dharmaraja Ratha, Bhima Ratha, Arjuna Ratha, Nakula-Sahadeva
Ratha and Draupadi Rathas.
o Other ratha monuments at Mahabalipuram include the late-7th-century Ganesha
Ratha. It is three storeyed and of better workmanship, it resembles the Bhima-ratha
in roof form.
 Structural temples
o The structural (free-standing) temples at Mamallapuram have been built with cut
stones as building blocks.
o Some of the major temples are:
 Shore Temple: It consists of a large temple, two smaller temples and many
Contact : https://t.me/EN123upsc || Telegram Link : https://t.me/prelimbits
49

minor shrines, open halls, gateways, and other elements, much of which is
buried by sand.
 Olakkanesvara temple: It was built in the early 8th century from grey granite
by King Rajasimha. The walls of the temple depict the Ravananugraha legend
from the Ramayana and a relief of Dakshinamurti (Shiva as a yoga teacher)
 Mukundanayanar temple: It has ratha-like architecture.
 Rock reliefs
o The best-known rock relief is the Descent of the Ganges also known as Arjuna's
Penance or Bhagiratha's Penance. (carved on two monolithic rock boulders)
o It has two primary interpretations: the effort needed to bring the Ganges from
the heavens to earth, and the Kirātārjunīya legend and the chapter from the
Mahabharata about Arjuna's efforts to gain the weapon he needed to help good
triumph over evil.

 Cave temples
o Varaha cave
o Kotikal cave
o Dharmaraja cave
o Ramanuja Cave

AJANTA CAVES (2ND C. – 5TH C.)


 Approx. 30 rock-cut Caves → Almost exclusively Buddhist. [both Theravada (Hinayana)
and Mahayana]
 Location : Aurangabad district of Maharashtra (UNESCO World Heritage Site). Caves are cut
into the side of a cliff that is on the south side of a U-shaped gorge on the small river
Waghora.
 The walls of the caves have both Murals and Fresco Paintings(painted on wet plaster) They
use tempera style, i.e. use of pigments.
 The paintings portray human values and social fabric along with styles, costumes and
ornaments of that period. The emotions are expressed through hand gestures.
 The unique feature of the paintings is that each female figure has a unique hairstyle. Even
animals and birds are shown with emotions.
 The Common Themes of these paintings range from Jataka stories to life of Buddha to
elaborate decorative patterns of flora and fauna.

Contact : https://t.me/EN123upsc || Telegram Link : https://t.me/prelimbits


50

 The outline of the figures is in red ochre, with contours of brown, black or deep red.
Some important paintings at Ajanta are:
 Scenes from the Jataka stories → Buddha‘s former lives as a Bodhisattva, the life of
Gautam Buddha, etc.
 Paintings of various Bodhisattvas in tribhanga pose in Cave No.
o Vajrapani (protector and guide, a symbol of Buddha‘s power)
o Manjusri (manifestation of Buddha‘s wisdom) and
o Padmapani (Avalokitesvara) (symbol of Buddha‘s compassion).
 The Dying Princess in Cave No. 16.
 Scene of Shibi Jataka, where King Shibi offered his own flesh to save the pigeon.
 Painting of Maya Devi, the mother of the Buddha
 Cave No. XVII representing Buddha‘s visit to the door of Yashodhara‘s abode in the city
of Kapilavastu
History
 The Ajanta Caves are generally agreed to have been made in two distinct phases
o 1st Phase → Satavahana period
o 2nd Phase (5th century) → During the reign of Hindu Emperor Harishena of the
Vaktaka dynasty. The second phase is attributed to the theistic Mahāyāna, or
Greater Vehicle tradition of Buddhist

29. Banjaras during the medieval period of Indian history were generally [2016-I]
A. agriculturists
B. warriors
C. weavers
D. traders

Ans. D

 There were many different tribal groups in Medieval India.


 Some were hunter-gatherer nomads. One such type of nomadic group was that of
the banjaras.
 Banjaras during the medieval period of Indian history were generally traders.
 Banjaras played a great role during the period of Alauddin Khalji.
o Alauddin Khalji used the Banjaras to transport grain to the city markets.
o Banjaras were transporters of goods from one place to another
 Emperor Jahangir wrote in his memoirs that the Banjaras carried grain on their bullocks

Contact : https://t.me/EN123upsc || Telegram Link : https://t.me/prelimbits


51

30. Who of the following founded a new city on the south bank of a tributary to river
Krishna and undertook to rule his new kingdom as the agent of a deity to whom all the
land south of the river Krishna was supposed to belong? [2015-I]
A. Amoghavarsha I
B. Ballala II
C. Harihara I
D. Prataparudra II

Ans. C

HARIHARA I
 Harihara I (&Bukka) was the founder of the Vijayanagara Empire.
 His initial military exploits and established his control over the valley of Tungabhadra
River (a tributary of Krishna), and gradually he expanded his control to certain
regions of Konkan and Malabar Coast.
 The death of the last ruler of Madurai, Veera Ballala III and the vacuum created facilitated
Harihara I to emerge as a sovereign power.
 The entire Hoysala territories came under his rule directly.

RASHTRAKUTA DYNASTY → AMOGHAVARSHA I (814-878)


 Called as ―Ashoka of the South‖ and also compared to Gupta king Vikramaditya.

 Built new capital city at Manyakheta (Modern Malkhed).

 Broach became the best port of the kingdom during his reign

 Defeated Eastern Chalukyas, maintained friendly relations with neighbours.

 He wrote Kavirajamarga (Kannada) and the Prashnottara Ratnamalika (Sanskrit).

 He was follower of Jainism (Digambara sec) [converted by Jinasena]

VEERA BALLAL III


 Veera Ballal III was Hoysala ruler of Dwarasamudra.
 He was defeated by Allauddin Khilji's army.

PRATAPARUDRA II
 Pratāparudra, also known as Rudradeva II.
 He was the last ruler of the Kakatiya dynasty of India.
 He ruled the eastern part of Deccan, having its capital at Warangal.
 He succeeded his grandmother Rudramadevi as the Kakatiya monarch.

31. Consider the following: [2015]


The arrival of Babur into India led to the

Contact : https://t.me/EN123upsc || Telegram Link : https://t.me/prelimbits


52

1. introduction of gunpowder in the subcontinent


2. introduction of the arch and dome in the region's architecture
3. establishment of the Timurid dynasty in the region

Select the correct answer using the code given below.

A. 1 and 2 only
B. 3 only
C. 1 and 3 only
D. 1, 2 and 3

Ans. B

MUGHAL PERIOD → BABUR (1526-1530 AD)


 First Battle of Panipat (1526) - Babur decisively defeated Ibrahim Lodhi. It Replaced the
Lodhi and established Mughal era in India.

 Reasons for Babur‘s victory: Alternatively resting one wing of Army, Services of two
ottoman master gunners - Ustad Ali & Mustafa, Use of Gunpowder, scientific use of cavalry
& artillery tactics - tulughma and the araba, effective use of Cannons.

 Battle of Khanwa (1527) - Babur defeated Rana Sangha and assumed title of Ghazi

 Battle of Chanderi (1528) - Medini Rai [Rajput ruler of Malwa] was defeated and with
this resistance across Rajputana was completely shattered.

 Battle of Ghagra (1529) - Babur defeated Mahmud Lodhi lodhi [lodhi was helped by
Nusrat Shah of Bengal] who aspired to the throne of Delhi.

Significance of Babur‘s Advent into India:


 Kabul and Gandhar became integral parts of Mughal Empire.

 Security from External invasions for almost 200 yrs.

 Babur Popularized the gun powder, cavalry and artillery in India. (Gunpowder used earlier
in India, but Babur popularized its use).

 Strengthened India‘s foreign trade.

 Proclaimed himself as "Emperor of Hindustan"

 Arrival of Babur into India led to the establishment of Timurid dynasty in the region.

 Babur introduced a new mode of warfare in India.

 Defeating Sangha & Lodi he smashed the balance of power and laid the foundation for all
India empire.

 He had assumed title of Badshah.

 Babur Introduced Char-Baghs and symmetrically laid out gardens.


Contact : https://t.me/EN123upsc || Telegram Link : https://t.me/prelimbits
53

 Built mosques at Panipath and Sambhat in Rohilkhand.

 Babur wrote Tuzuk-I-Baburi / Babarnama (tanslated into Persian by Abdur Rahim Khan-i-
Khana) & Masnavi.

 Babur was a great scholar in Arabic & Persian languages.

ARCH AND DOME


 Arches were introduced to India by Qutb al-Din Aibak during the year 1206. Domes were
introduced by the Turks during the 1500s by Persian architect. Minarets were introduced
by Qutb-Ud-Din Aibak.
o The architecture and art of Turks and Mughals were very different from that of
Indians.
o Under the reign of Qutb al-Din Aibak the Delhi Sultanate during the
year 1206 the Central-Asian styles and designs were introduced and introduced
the Islamic state to India. Domed tomb of Humayun is the earliest and major
building of Mughal reign during 1562 and 1571 by a Persian architect.
o Arches, domes, towers, minarets and decorations was introduced by the Turks
and used Arabic script.
o During the 1500s Akbar, third Mughal Empire expanded vastly during his reign and
brought several changes to his empire culturally, politically and economically.
o Qutb-Ud-Din-Aibak brought the minaret structure to India.
o The earliest mosque standing in India is in Qutb Minar in Delhi and the mosque is
known as ‗Quwwat-ul-Islam‘ or popularly known ‗Might of Islam‘ is in India
since 1193.

32. Consider the following pairs: [2015]

Medieval Indian State Present Region

1. Champaka Central India

2. Durgura Jammu

3. Kuluta Malabar

Which of the above pairs is/are correctly matched?

A. 1 and 2
B. 2 only
C. 1 and 3
D. 1 and 3 Only

Ans. B

Contact : https://t.me/EN123upsc || Telegram Link : https://t.me/prelimbits


54

MEDIEVAL INDIAN STATE


Medieval Indian State Present Region
Champaka Chamba
Durgara Jammu
Kuluta Kullu
Malwa Central India
Kamarupa Assam
Trigarta Jalandhar
Utkala Orissa
Twipra Tripura
OTHER MEDIEVAL CITY & FOUNDER
Medieval city Founder
Mehrauli Qutubbuddin Aibak
Siri Fort Alauddin Khilji
Tughlaqabad Giasuddin Tughlaq
Jahapanah Mohammad Bin Tughlaq
Feroz Shah Kotla Feroz Shah Tughlaq
Deen Panah Humayun
Shajahanabad Shahjahan

33. In medieval India, the designations „Mahattara‟ and „Pattakila‟ were used for [2014 ]
A. military officers
B. village headmen
C. specialists in Vedic rituals
D. chiefs of craft guilds

Ans. B

In medieval India, the designations 'Mahattara' and 'Pattakila' were used for village headmen.

RASHTRAKUTAS → ADMINISTRATION
 The system of administration in these empires was based upon the ideas and practices of
Gupta empire.

 King was head of administration and commander in chief of the armed forces. He also
dispensed justice.

 The king was assisted by number of ministers, generally form leading families and their
position was hereditary.

 The empires consisted of areas administered directly and areas ruled by vessels. The latter
had autonomy in internal affairs.

 The directly administered territories were divided into: Rashtra (province) and Visaya
(Districts) and Bhukti.

Contact : https://t.me/EN123upsc || Telegram Link : https://t.me/prelimbits


55

 The head of Rashtra was Rashtrapati, and had similar functions as the Uparaika
performed in Pala and Pratihara.

 Village was the smallest unit. The village headman carried out duties with the help of
Grama-Mahajana or Grama-Mahattara (village elders).

 Villages also had commitees to manage schools, tanks, temples etc.

 Law and order in the towns and in areas in their vicinity was the responsibility of
Koshtapala or Kotwal.

 Nad-gavundas or Desa-gramakutas were the hereditary revenue officers. [land grant


instead of paying]

34. Ibadat Khana at Fatehpur Sikri was [2014 ]


A. the mosque for the use of Royal Family
B. Akbar‘s private prayer chamber
C. the hall in which Akbar held discussions with scholars of various religions
D. the room in which the nobles belonging to different religions gathered to discuss religious
affairs

Ans. C

AKBAR (1556-1605) → RELIGIOUS POLICY


 Akbar rose to fame in the pages of history due to his religious policy. Various factors were
responsible for his religious ideas.

 The most important among them were his early contacts with the Sufi saints, the teachings
of his tutor Abdul Latif, his marriage with Rajput women, his association with
intellectual giants like Shaikh Mubarak and his two illustrious sons – Abul Faizi and Abul
Fazl

 He abolished the pilgrim tax and in 1562 CE, he abolished jiziya.

 He allowed his Hindu wives to worship their own gods.

 In 1575 CE, he constructed the Ibadat Khana (Hall of prayers) at his new capital
Fatehpur Sikri wherein Akbar invited learned scholars from all religions used to conduct
religious discussions with them. Some of the scholars were:

o Pursottam Das – Hindu

o Dastur Maharji Rana – Parsi (of Navsari)

o Hira Vijaya Suri – Jain saint of Kathiawar

o Aquaviva and Monserrate – Christian (sent by the Portuguese on Akbar‘s request)

 In 1579 CE, he issued the ―Infallibility Decree‖ by which he asserted his religious powers.

Contact : https://t.me/EN123upsc || Telegram Link : https://t.me/prelimbits


56

 In 1582 CE, he promulgated a new religion called Din-i-Ilahi/Tauhindi- Ilahi (Divine


Monotheism), which believes in one God and in Sul-i- Kul/peace to all. It contained the
good points of all religions and the basics were rational.

35. Consider the following Bhakti Saints: [2013]


1. Dadu Dayal
2. Guru Nanak
3. Tyagaraja

Who among the above was/were preaching when the Lodi dynasty fell and Babur took over?

A. 1 and 3
B. 2 only
C. 2 and 3
D. 1 and 2

Ans. B

 Guru Nanak Dev was born in 1469.


 Babur defeated Ibrahim Lodi in 1526.
 When Babur took over and established the Mughal dynasty Nanak was travelling to
different parts of India and preaching Sikhism.
 Babur once met Guru Nanak during one of his travel.

GURU NANAK DEV (1469-1538 AD)


 Birth : Talvandi Viilage & Death Kartarpur ( Both are in Pakistan)

 Founder of Sikh religion & the first Guru of ten Sikh Gurus. Initially worked as
Accountant.

 Preached middle path & need of Guru for liberation.

 He composed hymnes and sang along with the help of instrument called ‖Rabab‖ played
by his attendant ―Mardana‖.

 Travelled across the Asia including Mecca & Baghdad and spread the message of ―Ik
Onkar‖ - One God.

 Opposes idol worship, pilgrimage and caste system, and emphasized purity of character and
conduct.

 Called God as ―Waheguru‖ that is shapeless, timeless, omnipresent & invisible ( Nirgun
Bhakti)

 He started Guru-Ka-langer (Community Kitchen)

 He promoted ―Tauhid-e-wazidi

Contact : https://t.me/EN123upsc || Telegram Link : https://t.me/prelimbits


57

DADU DAYAL (C.1544–1603 CE)


 Belong to Nirguna Sant traditions in Northern India.

 He was a saint from Gujarat, who spent the best part of his spiritual life in Rajasthan. ‘
Dadu means brother, and Dayal means the compassionate one.

SAINT TYAGARAJA (1767−1847 CE)


 also known as Thyagayya and in full as Kakarla Thyagabrahmam, was a composer and
vocalist of Carnatic music, a form of Indian classical music.
 Tyagaraja and his contemporaries, Shyama Shastri and Muthuswami Dikshitar, are
regarded as the Trinity of Carnatic music.
 Thyagaraja composed thousands of devotional compositions, most in Telugu
and in praise of Lord Rama, many of which remain popular today, the most popular
being "Nagumomu"
 Of special mention are five of his compositions called the Pancharatna Kritis (transl. "five
gems"), which are often sung in programs in his honour, and Utsava Sampradaya Krithis
(transl. Festive ritual compositions), which are often sung to accompany temple rituals.
 Tyagaraja lived through the reigns of four kings of the Maratha dynasty – Tulaja II
(1763–1787), Amarasimha (1787–1798), Serfoji II (1798–1832) and Sivaji II (1832–1855),[4]
although he served none of them.

36. With reference to the religious history of medieval India, the Sufi mystics were known
to pursue which of the following practices? [2012]
1. Meditation and control of breath
2. Severe ascetic exercises in a lonely place
3. Recitation of holy songs to arouse a state of ecstasy in their audience

Select the correct answer using the codes given below:

A. 1 and 2 only
B. 2 and 3 only
C. 3 only
D. 1, 2 and 3

Ans. D

 Sufism is a mystical form of Islam that focuses on the spiritual search for God and shuns
materialism.
 The Sufi practices include →
o a lot of emphasis on love and devotion towards God.
o music and dance devotee expressions.
o Meditation and control of breath practices.
o The practice of Sama- a musical recital that intended to induce a mystical state
Contact : https://t.me/EN123upsc || Telegram Link : https://t.me/prelimbits
58

of ecstasy.
o Murid or disciple has to go through states and states by practising spiritual
exercises in a lonely place.

SUFI MOVEMENT
 In beginning of 12 AD, some religious people in Persia turned to asceticism due to the
increasing materialism of the Caliphate. They came to be called the ‗Sufis‘.

 In India, Sufi movement began in 1300 A.D & came to South India in 15th century.

 Sufism is the mystical arm of Islam. Sufis represent the inner side of the Islam creed
(tasawwuf).

 Sufi means wool: People who wear long woolen clothes were called as sufis.

 It was a liberal reform movement within Islam. It stressed the elements of love and
devotion as an effective means of the realization of God.

 It is based on Pir-Murid (Teacher- student) relationship.

 Sufism sprang from the doctrine of Wahadut-ul-wajud (unity of being) propounded by


Ibn-ul-Arabi [1165- 1240 AD].

 Early Sufi saints - Rabia, Mansur bin hallal.

 Sufi orders are broadly divided in two

o Ba-sara : Those followed Islamic law

o Be-shara : who did followed Islamic law.

 In Sufism, self-discipline was considered an essential condition to gain knowledge of


God. While orthodox Muslims emphasise external conduct, the Sufis lay stress on inner
purity.

 Sufism entered in India between 11th and 12th centuries.

o Al-Hujwari was first Sufis who settled in India and died in 1089 AD who is
popularly known as Data Ganj Bakhsh (Distributor of unlimited treasure).

 Multan and Punjab were the early centre and later on, it spread to Kashmir, Bihar, Bengal
and the Deccan.

The essential characteristics of Sufism

 Fana: Spiritual merger of devotee with Allah

 Insan-e-kamil: Perfect human with all good virtues,

 Zikr-tauba: remembrance of god all the time(zikr),

 Wahadatul-wazudi: one god for entire universe; unity of god and being.

Contact : https://t.me/EN123upsc || Telegram Link : https://t.me/prelimbits


59

 Sama: spiritual dance and music to promote their concepts, though music is un-Islamic.

Three stages in the history of Sufism

Stage Period Character

1st Stage: Khanqah 10th century Also called the age of Golden Mysticism

2nd Stage: Tariqa 11-14th century When Sufism was being institutionalised and
traditions and symbols started being attached to
it.

3rd Stage: Tarifa 15th century The stage when Sufism became a popular
onwards movement.

Sufism and music

 Sufism encourages music as a way of deepening one‘s relation with God, remembering
God by zikr (reciting name) sama or performance of mystical music

 Sufis too composed poems and a rich literature in prose, including anecdotes and fables.

 The most notable writer of this period was Amir Khusrau the follower of Nizamuddin
Auliya.

 Sufi saints are credited with invention of sitar and tabla.

 It contributed to evolution of classical music. Khayal style owes a lot to sufism.

 Amir Khusro was disciple of Nizamuddin Auliya. He give unique form to sama.

 Khusro is sometimes referred to as the ―Parrot of India‖. His songs are sung in several
dargahs across the country. Khusro is regarded as the ―Father of Qawwali‖

Important features

 The Sufis rejected the elaborate rituals and codes of behaviour demanded by Muslim
religious scholars.

 They believed that God is 'Mashuq' and Sufis are the 'Ashiqs'.

 Sufism was divided into 12 orders (Silsila) and each under a mystic Sufi saint of which
4 most popular ones were Chistis, Suhrawardis, Qadiriyahs and Naqshbandis.

 Silsila is continuous link between Murshid (teacher) and Murid (student). They lived in
Khanqah - hospice of worship.

 Names of Silsila were based on name of founding figure E.g. - Qadri Silsila based on name
of Shaikh Abdul Qadri & Place of origin E.g. - Chishti (place in Afghanistan).

 Sufism took roots in both rural and urban areas and exercised a deep social, political and
cultural influence on the masses.
Contact : https://t.me/EN123upsc || Telegram Link : https://t.me/prelimbits
60

 Sufism believes that devotion is more important than fast (Roza) or prayer (Namaz).

 Sufism discards caste system.

 Sufism has adapted extensively from the Vedanta school of the Hindu philosophy.

 The Cardinal Doctrines of the Sufism include: Complete surrender to the will of God;
Annihilation of the self; Becoming a perfect person

 These three cardinal principles altogether make the Doctrine of Fana which means
annihilation of human attributes through Union with God.

 In Sufism, a perfect being is also called a Wali (saint), a word that literally means ‗sincere
friend‘.

Important Silisila/Orders of Sufis

 Founder -Kwaja Abdul Chisti.


 Originated in Persia and Central Asia.
 Moinuddin Chishti introduced Chisti order in India.
Chisti  Baba Farid (Farīduddīn Ganjshakar) was disciple of Moinuddin & his
poems included in Guru granthsahib.
 Nizamuddin Auliya was called sidh/perfect due to yogi breathing exercises.
 The Chisti presence in Delhi was established by Qutbuddin Bakhtiyar
Kaki (the Qutub Minar is named after him).
 In the 13th century, the Chisti Order was established in the Deccan by
Shaikh Burhanuddin Gharib. The Deccan city of Bijapur emerged as an
important centre for Sufi activity, and one of the famous pirs of the region
was Muhammad Banda Nawaz.
 Sama was mostly associated with Chisti.
 Belong to Ba sharia.
Features
 A major feature of the Chishti tradition was austerity, including maintaining
a distance from worldly power. However, this was by no means a
situation of absolute isolation from political power.
 The sufis accepted unsolicited grants and donations from the political
elites. The Sultans in turn set up charitable trusts (auqaf) as
endowments for hospices and granted tax-free land (inam).
 The Chishtis accepted donations in cash and kind. Rather than
accumulate donations, they preferred to use these fully on immediate
requirements such as food, clothes, living quarters and ritual
necessities (such as sama).

Contact : https://t.me/EN123upsc || Telegram Link : https://t.me/prelimbits


61

 Founder – Shaikh Shihabuddin Surhavardi.


 Main centre was Multan.
Suhravardi  Saints of this order were also rich and held important government
positions.
 Associated with Delhi Sultanate.
 A famous saint is Bahauddin Zachariah
 Belong to Ba-sharia

Nakshabadi  Founder - Baha-ud-Din Naqshband Bukhari


 Khwaja pir mohammad introduced Nakshabadi order in India during
Akbar reign.
 Orthodox Sect.
 Mujaddid (an Islamic term for one who brings "renewal") opposed Shia, philosophy
of Wahadat-ul-Shahdud , wrote ‗Red-i-Khafid‘ arrested by Jhangir

Silsilah  The Quadiriyya Silsilah, which was popular in Punjab, was initiated
during the Mughal rule under the teachings of Sheikh Abdul Qadir and
his sons

37. With reference to Dhrupad, one of the major traditions of India that has been kept alive
for centuries, which of the following statements are correct? [2012 - I]
1. Dhrupad originated and developed in the Rajput kingdoms during the Mughal period.
2. Dhrupad is primarily devotional and spiritual music.
3. Dhrupad Alap uses Sanskrit syllables from Mantras.

Select the correct answer using the codes given below:

A. 1 and 2 only
B. 2 and 3 only
C. 1, 2, and 3
D. None of the above is correct

Ans. B

 Drupad is a genre in Hindustani Classical music.


 In ancient literature, Natyashastra is considered a source of Drupad.
 It is a form of devotional music and spiritual music that emphasis maintaining the purity
of the Raga.
 It traces its origin to the ancient text of Samveda.
 A Dhrupad has at least four stanzas, called Sthayi (or Asthayi), Antara, Sanchari and
Abhoga.
 Drupad music evolved with Sanskrit language scripts in itself.

38. Why did Buddhism start declining in India in the early medieval times? [2010]
1. Buddha was by that time considered as one of the incarnations of Vishnu and thus became
a part of Vaishnavism.
Contact : https://t.me/EN123upsc || Telegram Link : https://t.me/prelimbits
62

2. Invading tribes from Central Asia till the time of last Gupta king adopted Hinduism and
persecuted Buddhists.
3. Kings of the Gupta dynasty were strongly opposed to Buddhism.

Which of the statements given above is/are correct?

A. 1 only
B. 1 and 3 only
C. 2 and 3 only
D. 1, 2 and 3

Ans. A

 The Buddha was integrated into Vaishnavism through its mythology in the Vaishnava
Puranas, where the Buddha is adopted as the ninth avatar of Vishnu.
 According to the Agni Purana, Vishnu assumed this incarnation on earth due to the daityas
defeating the devas in their battles. In order to restore the natural order, he deluded the
asuras with his teachings.

39. Among the following, who was not a proponent of the Bhakti cult? [2010]
A. Nagarjuna
B. Tukaram
C. Tyagaraja
D. Vallabhacharya

Ans. A

NAGARJUNA (150-250 CE)


 Nagarjuna: He is often termed an Indian Einstein who proposed the Theory of Relativity in
his time in the form of a Sutra, the Prajna Parimata Sutra.
 He was also a great exponent of the Mahayana doctrine and propounded the
Madhyamaka school of Mahayana Buddhism (also known as Sunyavada School) which
focuses on sunyata or emptiness
 In some traditions he is known as an Indian metallurgist
 Nagarjun was a learned Buddhist of Nalanda University with profound knowledge in
chemistry. He wrote books like ‗Rasaratnakar‘ and ‗Arogyamanjari‘ on chemistry and
medicine respectively. Rasaratnakara describes methods for the extraction of metals like
gold, silver, tin and copper. https://www.myindiamyglory.com/2019/12/18/nagarjuna-
wizard-in-alchemy-metallurgy-introduced-gold-shine-mechanism/

TUKARAM (MARTHI POET)


 17th (1598 or 1608 - 1649 or 1650) century poet-saint

 Belong to Varkari tradition and was a Sudra by birth.

Contact : https://t.me/EN123upsc || Telegram Link : https://t.me/prelimbits


63

 Tukaram is known for his Avangas (dohas), which constitute the gatha devotional poetry,
and community oriented worship with spiritual songs known as kirtans.

 His poetry was devoted to Vitthala or Vithoba, an avatar of the Hindu God Vishnu.

 He was a contemporary of Shivaji and was responsible for creating a background for
Maratha nationalism, Parmaratha.

SAINT TYAGARAJA (1767−1847 CE)


 also known as Thyagayya and in full as Kakarla Thyagabrahmam, was a composer and
vocalist of Carnatic music, a form of Indian classical music.
 Tyagaraja and his contemporaries, Shyama Shastri and Muthuswami Dikshitar, are
regarded as the Trinity of Carnatic music.
 Thyagaraja composed thousands of devotional compositions, most in Telugu
and in praise of Lord Rama, many of which remain popular today, the most popular
being "Nagumomu"
 Of special mention are five of his compositions called the Pancharatna Kritis (transl. "five
gems"), which are often sung in programs in his honour, and Utsava Sampradaya Krithis
(transl. Festive ritual compositions), which are often sung to accompany temple rituals.
 Tyagaraja lived through the reigns of four kings of the Maratha dynasty – Tulaja II
(1763–1787), Amarasimha (1787–1798), Serfoji II (1798–1832) and Sivaji II (1832–1855),[4]
although he served none of them.

VALLABHACHARYA
 Born in Benaras in the 15th century and lived at the court of Krishnadeva Raya

 He propounded the Shudhadvaita (pure monism).

 In Shudhadvaita, as in Vishista Advaita, the world is taken to have a real existence, as also
Brahman. But →

 It is said that there is no change of Brahman into the world, the world exists as it were as
an aspect of Brahman without undergoing any change, it is a part of Brahman. We may
consider it like two sides of a coin.

 Its philosophy is Pushtimarga.

 He founded the Rudra Sampradya.

 According to him, God is omnipotent and omniscient and cause of all that is there in the
universe.

 Surdas was the disciple of Vallabhacharya who was blind but he was largely
instrumental in popularising the Krishna cult in north India.

Contact : https://t.me/EN123upsc || Telegram Link : https://t.me/prelimbits


64

40. With whose permission did the English set up their first factory in Surat? [2009]
A. Akbar
B. Jahangir
C. Shahjahan
D. Aurangzeb

Ans. B

 The East India Company was founded in India in 1600.


 The East India Company opened its first factory in "Surat".
 It was opened on 11 January 1613 after the permission of the Mughal emperor
"Jahangir".
 The Queen of Britain gave the company up to 21 years to do business with India.

JAHANGIR (1605-1627)
 When Akbar died, Prince Salim succeeded with the title Jahangir (Conqueror of World) in
1605.

 In c.1611 CE, he married Mehrunnisa (widow of Sher Afghani). He gave her title of Nur
Jahan (Light of the World) and her father Gyas Beg was given title of Itmaduddaulah.

o Nur Jahan had great influence on Jahangir‘s life and dominated the royal
household and set new fashions based on Persian traditions.

o She encouraged Persian art and culture in the court.

o She was given the status of Padshah Begum and was the only woman in Mughal
Darbar.

o The coins were struck in her name and all royal farmans also had her name.

o She was a constant companion of Jahangir and even joined him in his hunting.

 His son Khusrau revolted but was defeated and imprisoned and his supporter Guru
Arjun, the fifth Sikh Guru, was beheaded.

Mughal expansion under Jahangir

 In c.1615 CE, Amar Singh of Mewar (son of Maharana Pratap) submitted before him.
Rana‘s son Karan Singh was made Mansabdar in the Mughal court.
 In c.1616 CE, with the help of Maratha sardars , Khan-i- Khanan inflicted a crushing
defeat on the combined forces of Ahmadnagar, Bijapur, and Golconda.
 In c.1620 CE, Jahangir annexed Kangra.He was the first muslim ruler to do so.
 In c.1622 CE, Qandahara was occupied by Shah Abbas of Persia.
 Conquest of Deccan: Jahangir decided to follow Akbar‘s expansionist policy in the
Deccan. But Jahangir could achieve little success in it due to certain problems:
o He could not devote much attention in the crucial phase due to Khurram‘s revolt.

Contact : https://t.me/EN123upsc || Telegram Link : https://t.me/prelimbits


65

o The Mughal nobles were also involved in a number of intrigues and conflicts in
Deccan.
o Due to the rise of Malik Ambar in Deccan: Malik Ambar, with the help of
Marathas and the ruler of Bijapur Ibrahim Adil Shah, made it difficult for the
Mughals to consolidate their position in Berar, Ahmadnagar, and Balaghat.

 British Visited Machilipatnam during his reign. Captain Hawkins and Thomas Roe visited
his court.

o Emperor Jahangir granted a farman to Captain William Hawkins permitting the


English to erect a factory at Surat in 1613.

o In 1615, Thomas Roe, the Ambassador to James I, got an imperial farman


from Jahangir to trade and establish factories all across the Mughal empire.

 Jahangir erected Zanjr-i-adal at Agra fort (chain of justice) for the seekers of royal justice
 Mahtab Khan was his military general & had revolted against him
 Autobiography: Tuzuk-i- Jahangiri in Persian.
 He also patronised valuable dictionary Farhang-i-Jahangiri.
 During his reign, Abdul Hamid Lahori wrote Padshah Namah and Khafi Khan wrote
Muntakhab-i-Lubab.
 He also patronised many scholars like Ghiyas Beg, Naqib Khan, and Niamatullah.
 Introduction of the ―du-aspah-sih-aspah‖ system. It was modification to Mansabdari.
Nobles were allowed to maintain a large number of troops without raising their zat ranks.

Art and Architecture

 The practice of putting up buildings in marble and decorating the walls with floral
designs made of semi-precious stones (known as Pietra Durra) started during his reign.
 Noor Jahan built the tomb of her father Itmad-ud-dulah at Agra.
 Jahangir built Moti Masjid at Lahore and his own mausoleum at Lahore.
 He laid a number of gardens such as the Shalimar and Nishant gardens in Kashmir.
 Mughal painting reached its zenith under Jahangir. He employed a number of painters
like Abul Hasan, Bishan Das (regarded as master of portraits), Madhu, Anant, Manohar,
Govardhan and Ustad Mansur (specialist in animal painting).
 The use of ‗Halo‘ or ‗Divine lights‘ behind king‘s head started under him. Apart from
painting the scenes of hunting, battles and royal courts, progress was made in portrait
painting and paintings of animals.

Contact : https://t.me/EN123upsc || Telegram Link : https://t.me/prelimbits


66

41. Match List-I with List-II and select the correct answer using the code given below the
lists: [2009]

a) A – 2; B – 4; C – 3; D – 1
b) A – 2; B – 3; C – 4; D – 1
c) A – 1; B – 4; C – 3; D – 2
d) A – 1; B – 3; C – 4; D – 2

Ans. A)

BHIMESWARA SWAMY TEMPLE (DRAKSHARAMA)


 Draksharama is one of the five Pancharama Kshetras that are sacred to the Hindu god
Siva.
 The temple is located in Draksharamam town of Konaseema district in the Indian state of
Andhra Pradesh.
 Bhimeswara Swamy refers to Lord Siva in this temple.
 Inscriptions in the temple reveal that it was built between the 9th and 10th centuries CE
by the Eastern Chalukyan king, Bhima.
 The big Mandapam of the temple was built by Ganga Mahadevi ,daughter-In-Law of
Eastern Ganga Dynasty king Narasingha Deva I of Odisha.
 Architecturally and sculpturally, the temple reflects a blend of Chalukyan and Chola styles.

KANDARIYA MAHADEVA TEMPLE


 The Kandariya Mahadeva Temple, is the largest and most ornate Hindu temple in the
medieval temple group found at Khajuraho, Chhatarpur district in Madhya Pradesh.
 It is considered one of the best examples of temples preserved from the medieval period in
India
 Khajuraho was once the capital of the Chandela dynasty.
 The Kandariya Mahadeva temple was built during the reign of Vidyadhara (r. c. 1003-
1035 CE)

Contact : https://t.me/EN123upsc || Telegram Link : https://t.me/prelimbits


67

Various features of the temple marked on the Kandariya Mahadeva Temple.

Vidyadhara (c. 1003 – 1035 CE)

 The Ghaznavid ruler, Mahmud of Ghazni invaded the kingdom and Vidyadhara had to
pay tribute to him.

 The Muslim invasions weakened the Chandella empire and the Kalachuri king Gangeya
Deva taking advantage of the situation captured the eastern parts of the kingdom.

RAJARANI TEMPLE
 11th-century Hindu temple located in Bhubaneswar, the capital city of Odisha (Orissa
previously)
 The temple is believed to have been known originally as Indreswara. It is locally known as
a "love temple" because of the erotic carvings of women and couples in the temple.
 Rajarani Temple is built in the pancharatha style on a raised platform with two structures:
a central shrine called the vimana (sanctum) with a bada (curvilinear spire) over its roof
rising to a height of 18 m (59 ft), and a viewing hall called jagamohana with a pyramidal
roof.

Contact : https://t.me/EN123upsc || Telegram Link : https://t.me/prelimbits


68

 The temple was constructed of dull red and yellow sandstone locally called "Rajarani".
 There are no images inside the sanctum, and hence it is not associated with a specific sect
of Hinduism but broadly classified as Saivite based on the niches.

VIDYASHANKARA TEMPLE
 The Vidyashankara temple is located in the holy town of Sringeri, in the Chikmagalur
district of Karnataka. Apart from its religious significance, the temple is an eye open for
architecture enthusiasts as it combines the architectural style of the Hoysala and Dravidian
eras.
 Sringeri is the site of one of the Advaitha mathas established by Adi Shankara himself. It
has a continuing tradition and recorded history from the eighth century. Sri Adi Shankara‘s
disciple Sureshwarachrya was the first head of this matha.

42. Where is the famous Virupaksha temple located? [2009]


A. Bhadrachalam
B. Chidambaram
C. Hampi
D. Srikalahasti

Ans. C

VIRUPAKSHA TEMPLE
 Virupaksha Temple is located in Hampi in the Vijayanagara district of Karnataka.
 It is part of the Group of Monuments at Hampi, designated as a UNESCO World
Heritage Site.
 The temple is dedicated to Lord Virupaksha, a form of Shiva.
 The temple was built by Lakkan Dandesha, a nayaka (chieftain) under the ruler Deva
Raya II also known as Prauda Deva Raya of the Vijayanagar Empire.
 Hampi, capital of the Vijayanagar empire, sits on the banks of the Tungabhadra River
(Pampa hole/Pampa river).
 Virupaksha Temple is the main center of pilgrimage at Hampi, and had been considered the
most sacred sanctuary over the centuries. It is intact among the surrounding ruins and is still
used in worship .
 There is also a Virupakshini Amma temple (mother goddess) in a village called
Nalagamapalle, Chittoor district, Andhra Pradesh, approximately 100 km from Tirupati.

43. During the time of which Mughal Emperor did the English East India Company
establish its first factory in India? [2008]
A. Akbar
B. Jahangir
C. Shahjahan
D. Aurangzeb
Contact : https://t.me/EN123upsc || Telegram Link : https://t.me/prelimbits
69

Ans. B

Refer Q. 38

 The East India Company was founded in India in 1600.


 The East India Company opened its first factory in "Surat".
 It was opened on 11 January 1613 after the permission of the Mughal emperor
"Jahangir". William Hawkins visited the court of Jahangir and received a Farman which
allowed East India Company to set up their first factory.

44.Where is the famous Vijaya Vittala temple having its 56 carved pillars emitting musical
notes located? [2007]
A. Belur
B. Bhadrachalam
C. Hampi
D. Srirangam

Ans. C

VITTHALA TEMPLE

 The Vitthala temple and market complex is over 3 kilometres (1.9 mi) north-east of the
Virupaksha temple near the banks of the Tungabhadra River.
 It is the most artistically sophisticated Hindu temple in Hampi, and is part of the sacred
centre of Vijayanagara.
 It is unclear when the temple complex was built, and who built it; most scholars date it to a
period of construction in the early-to-mid-16th century
 Some of the books mention that its construction began during the time of Devaraya II and
continued during the reign of Krishnadevaraya, Achuytaraya, and probably Sadasivaraya
and it stopped probably due to the destruction of the city in 1565.
 The inscriptions include male and female names, suggesting that the complex was built by
multiple sponsors.
 The temple was dedicated to Vitthala, a form of Krishna also called Vithoba.
 The temple opens to the east, has a square plan and features an entrance gopuram with
two side gopurams. The main temple stands in the middle of a paved courtyard and several
subsidiary shrines, all aligned to the east.
 The temple is a unified structure in a courtyard measuring 500 by 300 feet which is
surrounded by a triple row of pillars. It is a low structure of one storey with an average
height of 25 height.
 The temple has three distinct compartments: a garbhagriha, an ardhamandapa and a
mahamandapa (or sabha mandapa).
 The Vitthala temple has a Garuda shrine in the form of a stone chariot in the courtyard;
it is an often-pictured symbol of Hampi
Contact : https://t.me/EN123upsc || Telegram Link : https://t.me/prelimbits
70

Garuda shrine in the form of stone chariot at Vitthala temple

 In the front of the stone chariot is a large, square, open-pillared, axial sabha mandapa,
or community hall.
 The mandapa has four sections, two of which are aligned with the temple sanctum. The
mandapa has 56 carved stone beams of different diameters, shape, length
and surface finish that produces musical sounds when struck
 The mandapa links to an enclosed pradakshina patha for walking around the sanctum.
Around this axial mandapa are (clockwise from east); the Garuda shrine, the Kalyana
mandapa (wedding ceremonies), the 100-columned mandapa, the Amman shrine and
the Utsav mandapa (festival hall).

45. In Indian history, who was Abdul Hamid Lahori? [2006]


A. An important military commander during Akbar's reign.
B. An official historian of the reign of Shahjahan.
C. An important noble and confidant of Aurangzeb.
D. A chronicler and poet during the reign of Muhammad Shah.

Ans. B

For more Refer Q. 38

 Abdul Hamid Lahori (d. 1654) was a traveler during the period of Shahjahan who later
became a court historian of Shahjahan.
 He wrote the book Padshah Nama also referred to as Badshah Nama, about the reign of
Shahjahan.

46. Bhakta Tukaram was a contemporary of which Mughal emperor? [2006]


A. Babar
B. Akbar
C. Jahangir
D. Aurangzeb

Ans. C

 Babur (1526-1530 AD)

Contact : https://t.me/EN123upsc || Telegram Link : https://t.me/prelimbits


71

 Akbar (1556-1605)
 Aurangzeb (1658 - 1707)
 Jahangir (1605-1627)
 Tukaram (1608 - 1650)

TUKARAM (MARTHI POET)


 17th (1598 or 1608 - 1649 or 1650) century poet-saint

 Belong to Varkari tradition and was a Sudra by birth.

 Tukaram is known for his Avangas (dohas), which constitute the gatha devotional poetry,
and community oriented worship with spiritual songs known as kirtans.

 His poetry was devoted to Vitthala or Vithoba, an avatar of the Hindu God Vishnu.

 He was a contemporary of Shivaji and was responsible for creating a background for
Maratha nationalism, Parmaratha.

47. Which one of the following is the correct chronological order of the Afghan rulers to
the throne of Delhi? [2006]
A. Sikandar Shah-Ibrahim Lodi-Bahlol Khan Lodi
B. Sikandar Shah-Bahlol Khan Lodi-Ibrahim Lodi
C. Bahlol Khan Lodi-Sikandar Shah-Ibrahim Lodi
D. Bahlol Khan Lodi-Ibrahim Lodi-Sikandar Shah

Ans. C

LODHI DYNASTY (1451-1526 AD)


 The Lodi dynasty was an Afghan dynasty

 It was the fifth and final dynasty of the Delhi Sultanate

 Founded by Bahlul Khan Lodi. Sikandar Lodi succeeded him.

Sikandar Lodi (1489-1517)

 He was contemporary of Mahmud Begarha of Gujrat and Rana Sanga of Mewar.

 He founded city Agra and transferred capital from Delhi to Agra.

 He abolished the octroi duty on grains and established a new measurement of the yard
called the Gaz-i- Sikandari.

 Sikandar was orthodox and a bigot king. He re-imposed the Jizya on Hindus.

 He was a poet of repute, composed under the pen-name of Gulruk.

 Sikandar Lodi was succeeded by his son Ibrahim Lodi.

Ibrahim Lodi (1517-1526)

Contact : https://t.me/EN123upsc || Telegram Link : https://t.me/prelimbits


72

 He was the last Lodi Sultan of Delhi.

 Ibrahim was defeated in 1526 at the Battle of Panipat.

 He was arrogant. Humiliated his nobles in open courts, this made Daulat Khan Lodi (the
governor of the Punjab) invite Babar to invade India/Delhi.

 This marked the end of the Lodi Dynasty and the RISE OF THE MUGHAL EMPIRE in India.

48. [2006]
Assertion (A): Muhammad bin Tughlaq issued a new gold coin which was called Dinar by
Ibn Battuta.
Reason (R): Muhammad bin Tughlaq wanted to issue token currency in gold coins to
promote trade with West Asian and North African countries.
A. Both A are R are true but R is the correct explanation of A
B. Both A and R are true but R is not a correct explanation of A
C. A is true but R is false
D. A is false but R is true

Ans. C

MOHAMMAD BIN TUGHLAQ / JAUNA KHAN (1325-1351)


 His reign marks the zenith of the Delhi Sultanate but also saw the beginning of
disintegration.

 He defeated Mongols.

 Appointed official on the basis of merit.

 Advanced secular policies. Applied justice to Ulema.

 During his reign, the famous traveller Ibn Batuta (author of Safarnama Rehla) came to
India in c.1334 CE and acted as a Qazi at Delhi for eight years.

 He was the only Delhi Sultan who had received a comprehensive literacy, religious and
philosophical education. (Most learned Man of D. Sul.)

 His ideas/decisions were ahead of his time hence he was not successful of a ruler.

 He Built fort of Adilabad and the city of Jahanpanah.

 During his reign 3 major kingdoms of South India emerged: Vijayanagar, Bahamani, and
Madurai.

 Imposed additional cesses called Abwabs on the peasants.

Muhamma Tughlaq‟s Experiments

 Transferred capital from Delhi to Devagiri (Daulatadab). Shifted back after 2 years
because of lack of water supply.
Contact : https://t.me/EN123upsc || Telegram Link : https://t.me/prelimbits
73

 Token Currency- Issued bronze coins at par with the value of the silver tanka coins. He
had also introduced the copper currency system. Later withdrew both.
o When famine-like conditions and frequent revolts began straining his coffers,
Tughlaq found it difficult to maintain the supply of gold (dinars) and silver (adlis)
coins on a large scale
o Muhammad Bin Tughluq introduced bronze and copper coins as he needed money
for various reasons:
 He launched Khorasan project to counter the threat of Chinses incursions – Failed.
 Quarachi expedition – launched in Kumaon hills to counter the threat of Chinses
incursions – failed.
 He set up Diwan -i- amir- kohi a separated department to extend cultivation by giving
loans to cultivators (takkavi loans) – Failed due to corrupt officials

49. In the year 1613, where was the English East India Company given permission to set up
a factory (trading post)? [2006]
A. Bangalore
B. Madras
C. Masulipattam
D. Surat

Ans. D

EIC → QUICK RECAP

1600  The East India Company was established.

1609  William Hawkins arrived at Jahangir‘s court but failed to secure permission due
to opposition by Portuguese.

1611  Captain Middleton obtained the permission of the Mughal governor of Surat
to trade there, also started trade in Masulipatnam.
 The English defeated the Portuguese in the Battle of Swally Hole near Surat

1613  A factory of East India Company was established at Surat. [west]

1615  Thomas Roe succeeded in obtaining two farman confirming free trade with
exemption from inland tolls and permission to establish factories at Agra,
Ahmedabad and Baroach.

1616  The Company established its first factory in the south in Masulipatnam.

1632  The Company got the Golden Farman from the Sultan of Golconda which
ensured safety and prosperity of their trade.

1633  The Company established its first factory in east India in Hariharpur, Balasore,
Odisha

Contact : https://t.me/EN123upsc || Telegram Link : https://t.me/prelimbits


74

1639  Madras was given by the Chandragiri chief to the English and soon Madras with
the Fort St. George replaced Masulipatnam as the English headquarters on the
east coast.

1651  The Company was given permission to trade at Hooghly (Bengal), Kasimbazar,
Patna and Rajmahal.

1658  All establishments of the English Company in Bengal, Bihar, Orissa and
Coromandel Coast were brought under the control of Fort St. George.

1662  The British King, Charles II, was given Bombay as dowry for marrying a
Portuguese princess (Catherine of Braganza)

1667  Aurangzeb gave the English a farman for trade in Bengal


In 1689 Sir John Child, the English Governor of East India Company in India, was
expelled by Aurangzeb. Child became involved in a war with the Mughal
emperor Aurangzeb, whose troops captured Surat and forced Child to make peace.
One of the peace terms required Child to leave India, but he died while the
matter was still pending.

1687  Bombay was made the headquarters by shifting the seat of the Western
Presidency from Surat to Bombay.

1690  August 1690 they able to get place name called Sutanuti (with the help of Job
Charnock , a company agent) from Mughal.
 Due to some conflict, Mughals raided Hoogly. Then Job Charnock, negotiated
with Mughals for return to Sutanuti in 1691.

1698  In 1698, the English succeeded in getting the permission to buy the zamindari of
the three villages of Sutanuti, Gobindapur and Kalikata (Kalighat).

 The city of Calcutta grew from the development of three villages Sutanuti,
Gobindapur and Kalikata secured from the Mughal governor of Bengal.
1700
 The fortified settlement was named Fort William (1700) and it became the seat
of British power in India till 1911 [with Sir Charles Eyre as its first president]
In 1772, when Calcutta was announced as the capital of British India

1717  The Mughal emperor Farrukh siyar issued a farman, called Magna Carta of the
Company, giving the Company a large number of trade concessions in Bengal,
Gujarat and Hyderabad. It included:
1. Company‘s import and export were exempted from duties.
2. Permission to issue Dastaks for transportation of goods
3. Duty free trade in Hyderabad.

Contact : https://t.me/EN123upsc || Telegram Link : https://t.me/prelimbits


75

4. Company minted coins to act as currency throughout Mughal Empire.

50. When Raja Wodeyar founded the kingdom of Mysore, who was the ruler of the
Vijayanagar Empire? [2006]
A. Sadasiva
B. Tirumala
C. Ranga II
D. Venkata II

Ans. A

Vijayanagara Empire disintegrated in 1565. With the fall of the empire, Mysore became an
Independent Kingdom. Sadasiva Raya (1542–1570 CE), last ruler of Tuluva dynasty.

 The Wadiyar dynasty (formerly spelt Wodeyer or Odeyer, also referred to as the Wadiyars
of Mysore), is a late-medieval/early-modern South Indian Hindu royal family of former
kings of Mysore from the Urs clan originally based in Mysore city.
 The Wadiyar dynasty started when Yaduraya, a garrison leader (poleygar), was made the
prefect of Mysore and the surrounding regions his overlord Harihara II of the
Vijayanagara Empire in 1399.
 With this, Yaduraya assumed the title Raja and the honorary surname Wadiyar. He and
his successors ruled the fiefdom of Mysore as rajas under the vassalage of the Vijayanagara
Empire until around 1553.

Independent kingdom

 The Vijayanagara Empire disintegrated in 1565. With the fall of the empire, Mysore
became an Independent Kingdom, the first independent king being Timmaraja Wodeyar
II, the great-great-great-grandson of the founding ruler Yaduraya.
 Thimmaraja's nephew Raja Wodeyar I expanded the borders of the kingdom. In 1610, he
moved the capital from Mysore to nearby island town of Srirangapattana on the river
Kaveri, which provided strategic protection against military attacks.
 The kingdom reached its peak under Devaraja Wodeyar II, who reformed the
administration of the kingdom by dividing it into 18 departments (called chavadis); he also
introduced a coherent system of taxation.
 From 1760 to 1799, the rule of the Wadiyar dynasty was essentially nominal, with real
power firmly in the hands of the Commander-in-chief and later self-proclaimed
sultan, Hyder Ali, and his son and successor Tipu. The two, ruling the sultanate from
Srirangapattana, expanded the kingdom aggressively.

SADA SIVA RAYA (C.1542–1570 CE)


 Last ruler of Tuluva dynasty.

Contact : https://t.me/EN123upsc || Telegram Link : https://t.me/prelimbits


76

 He was a puppet in the hands of his minister, Aliya Rama Raya, who was the de facto
king.

 Ram Raja completely defeated the Bijapur ruler .

 Then he allied himself with the Bijapur ruler to inflict humiliating defeats on Golconda
and Ahmednagar.

 However, his enemies (except Berar) ultimately combined to inflict a crushing defeat on
Vijayanagar at Bannihatti, in Battle of Talikota (Battle of Raksasa Tangadi), in 1565.

 The Battle of Bannihatti is generally considered to mark the end of the Vijayanagar
Empire.

 Caesar Fredrick (Portuguese traveller) visited Vijaynagar after battle of Talaikotta.

However, the Vijayanagar kingdom continued to exist under the Aravidu dynasty, which ruled
from Penukonda and later from Chandragiri (near Tirupati) for about another century. Thirumala,
Sri Ranga, and Venkata II were the important rulers of this dynasty.
The last ruler of Vijayanagar kingdom was Sri Ranga III (c.1642–1646 CE)

IN THE VIJAYANAGARA EMPIRE, FOUR DYNASTIES RULED THE WHOLE OF SOUTH INDIA FOR 310
YEARS.

Contact : https://t.me/EN123upsc || Telegram Link : https://t.me/prelimbits


77

51. The initial design and construction of which massive temple took place during the
reign of Suryavarman II? [2006]
A. Sri Mariamman Temple
B. Angkor Vat Temple
C. Batu Caves Temple
D. Kamakhya Temple

Ans. B

ANGKOR WAT TEMPLE

 Angkor Wat temple complex is located at Angkor, Cambodia.


 Angkor Wat is the world‘s largest religious structure, covering some 400 acres.
 The temple is dedicated to the Hindu god Vishnu.
 The temple was built in the 12th century by King Suryavarman II, one of the greatest
kings of the Khmer dynasty.
 Angkor Wat has unusual orientation of facing west, rather than the traditional east.
 Theravada Buddhist monks maintained Angkor Wat after it was abandoned in early
15th century.
 Angkor Complex including Angkor Wat is an UNESCO World Heritage Site since 1992.
Other Temples that India restores outside India
Contact : https://t.me/EN123upsc || Telegram Link : https://t.me/prelimbits
78

 Thiruketheeswaram Temple in Mannar, SriLanka.


 One of the five sacred Ishwarams dedicated to Lord Shiva.
 This temple was closed for 12 years.
 Shreenathji Temple in Manama, Bahrain.
 It is a 200-year-old Sri Krishna temple in Bahrain.

(RELATED ) UNAKOTI - ANGKOR WAT OF NORTH EAST


Unakoti, the chiselled ‗Angkor Wat of the North-East‘ is competing for a UNESCO world
heritage classification.

https://www.hindustantimes.com/lifestyle/travel/unakoti-the-angkor-wat-of-the-north-east-is-
vying-for-world-heritage-tag-101670680601782.html

https://indianexpress.com/article/lifestyle/destination-of-the-week/tripura-unakoti-angkor-wat-
north-east-unesco-world-heritage-tag-8319590/

 Unakoti means one less than a crore in Bengali


and in the local Kokborok language, it is called
Subrai Khung.
 Unakoti is a Saivite pilgrimage site with
marvellous rock carvings, murals with their
primitive beauty and waterfalls.
 The site is nestled in the Raghunandan hills in Unakoti District of Tripura.
 The site dating back to the 7th-9th centuries is now under Archaeological Society of India
(ASI).
Rock Sculptures

 There are two types of images found at Unakoti


o Rock-carved figures
o Stone images
 Among the rock-cut carvings, the CENTRAL SHIVA head and GIGANTIC GANESHA
FIGURES are prominent.
 The central Shiva head - Also known as Unakotiswara Kal Bhairava.
o The head is about 30 feet high, including an embroidered head-dress which itself
is 10 feet high.
o On each side of the head-dress of the central Shiva, there are two full-size female
figures - one of Durga standing on a lion and another female figure on the other
side.
o In addition, 3 enormous images of Nandi Bull are found half-buried in the ground.
o Hindu mythology says that Lord Shiva once spent a night here on way to Kashi.
Unakoti or one less than a crore gods and goddesses followed him.
 Ashokastami Mela
o Thousands of pilgrims visit this site every year during ‗Ashokastami Mela‘ which
takes place in the month of April.

Contact : https://t.me/EN123upsc || Telegram Link : https://t.me/prelimbits


79

 Angkor Wat of North-East


o The structures of the rock-cut sculptures are gigantic and have distinct mongoloid
features.
o They display almost the same mystical charm as in the Angkor Wat temple of
Cambodia and hence got the name.

SRI MARIAMMAN TEMPLE, SINGAPORE


 Singapore's oldest Hindu temple.
 The Sri Mariamman Temple was founded in 1827 by Naraina Pillai, eight years after the East
India Company established a trading settlement in Singapore. Pillai was a government clerk

BATU CAVES
 Batu Caves is a mogote (a type of karst landform) that has a series of caves and cave
temples in Gombak, Selangor, Malaysia.
 The cave is one of the most popular Hindu shrines outside India, and is dedicated to Lord
Murugan. It is the focal point of the Tamil festival of Thaipusam in Malaysia.

KAMAKHYA TEMPLE
 The Kamakhya Temple at Nilachal hills in Guwahati, Assam is one of the oldest and most
revered centres of Tantric practices, dedicated to the goddess Kamakhya

52. Consider the following statements about Sikh Gurus: [2004]


1. Banda Bahadur was appointed as the military leader of the Sikhs by Guru Tegh Bahadur.
2. Guru Arjun Dev became the Sikh Guru after Guru Ram Das.
3. Guru Arjun Dev gave to Sikhs their own script Guru Mukhi.

Which of the statements given above is/are correct?

A. 1 only
B. 2 only
C. 1 and 3
D. 1 and 2

Ans. B
Contact : https://t.me/EN123upsc || Telegram Link : https://t.me/prelimbits
80

 Banda Bahadur was appointed as the military leader of the Sikhs by Guru Gobind Singh.
 Guru Arjan Dev became the Sikh Guru after Guru Ram Das.

THE SIKHS
 Sikhism was founded by Guru Nanak in the Punjab region in the 15th century. The Sikh
Panth was as old as the Mughal Empire.
 When Guru Nanak began to preach his message of devotion and equality in Punjab in the
15th century, Babur was founding the Mughal Empire.
 ‗Sikh‘ literally means learner or disciple. In the course of time, the new cult took the form
of a religious movement and spread quickly among the Jat peasantry and other lower
castes in Punjab.
 Guru Nanak‘s religious movement was peaceful and secular.
 Nanak was succeeded by nine Gurus; all the ten Sikh Gurus emphasised on simplicity of
religion and freedom from bigotry.
1. Guru Nanak : founder of Sikhism.
2. Guru Angad : developed the Gurumukhi script, introduced the langar (free kitchen)
and tradition of mall akhara for the youth.
3. Guru Amar Das : composed the Anand Sahib, introduced the simple Anand Karaj
marriage and abolished sati among the Sikhs.
4. Guru Ram Das : laid the foundation of the holy city of Amritsar (earlier known as
Ramdaspur and Guru-ka-Chak) and initiated the construction of the Golden
Temple.
5. Guru Arjan Dev : compiled the Adi Granth and completed the construction of the
Golden Temple (It was during Ranjit Singh‘s reign that the shrine of the temple was
richly decorated with marble and gold plates and came to be known as the Golden
Temple).
6. Guru Hargobind : created the Sikh marital art called Gatka, trained the Sikhs in
military art and war tactics, built the Akai Takhat, and was known as the ‗soldier
saint‘.
7. Guru Har Rai : he maintained the large army of Sikh soldiers that the sixth Sikh Guru
had amassed, yet avoided military conflict, and was known as the ‗tender-hearted
guru‘.
8. Guru Har Krishan : was the youngest of all gurus, installed as guru at the age of
five.
9. Guru Teg Bahadur : he refused to convert to Islam and was beheaded in Delhi on
the orders of Mughal Emperor Aurangzeb.
10. Guru Gobind Singh : transformed the Sikhs into a military force by establishing
the body of the Khalsa in 1699.
Relationship with the Mughals

Contact : https://t.me/EN123upsc || Telegram Link : https://t.me/prelimbits


81

 The last four gurus were persecuted by the Mughals, Guru Arjan and Guru Teg Bahadur
were tortured and brutally killed for refusing to convert to Islam. Subsequently, Sikhism
militarised to oppose Mughal dominance.
 Aurangzeb was initially not very hostile to the Sikhs; but as the community grew in size
and challenged the central authority of the Mughals, the emperor turned against them.
 Guru Tegh Bahadur was executed in Delhi in 1675 (however, the emergence of Sikh
empire under Ranjit Singh was characterised by religious tolerance and pluralism).
Guru Gobind Singh(1666-1708)

 10th and last guru of the Sikhs


 Transformed the Sikhs into a military force by establishing the body of Khalsa in 1699.
 Members were were obliged to show their allegiance by five signs [5Ks] including kesh
(uncut hair) and kirpan (short sword).
 Guru Gobind conflict with Aurangzeb is well known.
o From about 1696, Guru Gobind had tried to carve
out an autonomous domain in and around
Anandpur.
o In 1704, Anandpur was captured by the Mughals
and the guru was compelled to leave. Guru Gobind
was murdered in a conspiracy in 1708.
Banda Bahadur (1708-15)

 After Guru Gobind Singh death the institution of Guruship came to an end and the
leadership of the Sikhs passed to his trusted disciple Banda Singh, who is more widely
known as Banda Bahadur.
 Banda rallied together the peasants and the lower castes of the Punjab from Delhi to Lahore
and carried on a vigorus though unequal struggle against the Mughal army for eight
years. He was captured m 1715 and put to death.
PERIOD OF SOVEREIGN SIKH RULE
 With the withdrawal of Abdali from the Punjab, the Sikh confederacies began to fill the
political vacuum. Between 1765 and 1800, they brought the Punjab and Jammu under their
control.
 In this way, the period from 1765 onwards saw steady development of Sikh political
power which culminated in the establishment of an autonomous state in the early 19th
century.
Sikh Confederacies

 In the face of severe Mughal repression, the Sikhs had organised themselves into several
small mobile bands called Jathas, each headed by a Jathedar.
 After 1765, numerous small Jathas regrouped themselves into 12 larger regional
confederacies or Misls led by local chiefs.

Contact : https://t.me/EN123upsc || Telegram Link : https://t.me/prelimbits


82

 The misls did occasionally unite, but largely the political authority in Punjab remained
decentralised and more horizontally dispersed during this whole period until Ranjit Singh,
the chief of the Sukerchakia misl, tried to raise a more centralised Sikh state at the
end of the 18th century. Thus, Punjab‘s rise to prominence had to wait till the end of the
century for Ranjit Singh.
Rakhi (Tax Protection) Framework
 Under a number of able leaders in the eighteenth century, the Sikhs organized themselves
into a number of bands called jathas, and later on misls.
 Their combined forces were known as the grand army (dal khalsa). The entire body used
to meet at Amritsar at the time of Baisakhi and Diwali to take collective decisions known as
―resolutions of the Guru (gurmatas)‖.
 A system called Rakhi was introduced, offering protection to cultivators on the payment
of a tax of 20 per cent of the produce.
The Punjab under Ranjit Singh

 He was the founder of the Sikh Empire (1799) by overthrowing Misls.


 He ruled the northwest Indian subcontinent in the early half of the 19th century.
 He was given the title Lion of Punjab (Sher-e-Punjab) for his success in freeing Lahore
(his capital) from the Afghan invaders.
Modernization of Army

 He combined the strong points of the traditional Khalsa army with western advances in
warfare to raise Asia‘s most powerful indigenous army of that time.
 He also employed a large number of European officers, especially French, to train his
troops.
 He appointed a French General to modernize his army.
 He set up modem foundries to manufacture cannon at Lahore
 He adopted the system of ‗Mahadari‘ or monthly payment of salaries to soldiers and
officers, and gave care to the equipment and mobilisation aspects of the army
Wide Empire

Contact : https://t.me/EN123upsc || Telegram Link : https://t.me/prelimbits


83

 Ranjit Singh‘s trans-regional empire (spread over several states) included the former
Mughal provinces of Lahore and Multan besides part of Kabul and the entire Peshawar.
 The boundaries of his state went up to Ladakh — in the northeast, Khyber pass (route the
foreign rulers took to invade India) in the northwest, and up to Panjnad in the south
where the five rivers of Punjab fell into the Indus.
Legacy

 The Maharaja was known for his just and secular rule. Both Hindus and Muslims were
given powerful positions in his darbar.
 He turned Harimandir Sahib at Amritsar into the Golden Temple by covering it with gold.
 He is also credited with funding Hazoor Sahib gurudwara at the final resting place
of Guru Gobind Singh in Nanded, Maharashtra.
The Conquest of Punjab & Anglo-Sikh Wars

 Period after the death of Maharaj Ranjit Singh (1839) saw great instability in Punjab.
 Ultimately, Power fell into the hands of the brave and patriotic but utterly undisciplined
army called Khalsa, which interfered in the affairs of state.
 Maharaja Ranjit Singh‘s son Dalip Singh was on the throne but state was ruled by his
mother Rani Jindan.
 British had signed a treaty of perpetual friendship with Ranjit Singh in 1809 but they
were looking for every opportunity to conquer Punjab.
1st Anglo-Sikh War (1845-46)

 Major Broad was placed in Amritsar as the East India Company‘s agent in 1843.
 The Sikh forces crossed the Sutlej in December 1845 and took offensive positions against
the English forces.
 Punjab army was defeated of battle of Sobraon
 Sign the humiliating Treaty of Lahore in March 1846.
o Mother Jindan Kaur as regent.
o Had to cede the Jalandhar Doab to the British.=
o Huge war indemnity [that they could not pay]
o British Resident, Sir Henry Lawrence was appointed to the Sikh court.
2nd Anglo-Sikh War

 Punjab army fought bravely symbolized by famous Battle of Chillianwala but it was defeat
after the final battle of Gujarat (a place in Punjab) in 1849.
 Lord Dalhousie the new Governor-General annexed Punjab in 1849
 Sir John Lawrence was appointed as the first Chief Commissioner of Punjab to take care
of the administration.
 The famous Koh-i-Noor diamond went into British hands. It was in possession of Maharaja

Contact : https://t.me/EN123upsc || Telegram Link : https://t.me/prelimbits


84

Ranjit Singh who had willed it to the Puri Jagannath Temple of Odisha but his will was
not executed by the British. They say it was acquired as part of the Treaty of Lahore
after the second Anglo-Sikh war.
53. How did the dynasty of Nizam Shahis of Ahmadnagar come to an end? [2004]
A. Ahmadnagar was annexed into the Mughal empire and Hussain Shah was given life
imprisonment
B. Mughal troops destroyed Daulatabad fort and killed Nizam-ul Mulk of Ahmadnagar
C. Fateh Khan usurped the throne from Nizam-ul-Mulk
D. Malik Ambar was defeated in a battle with Mughals in 1631 and the entire royal family was
killed by the Mughal troops

Ans. A

AHMADNAGAR SULTANATE (NIZAM SHAHI DYNASTY)


 The Ahmadnagar Sultanate was a late medieval Indian Muslim kingdom located in the
northwestern Deccan, between the sultanates of Gujarat and Bijapur.
 Malik Ahmed, the Bahmani governor of Junnar after defeating the Bahmani army led by
general Jahangir Khan on 28 May 1490 declared independence and established the Nizam
Shahi dynasty rule over the sultanate of Ahmednagar
 Initially his capital was in the town of Junnar with its fort, later renamed Shivneri. In
1494, the foundation was laid for the new capital Ahmadnagar.
 After the death of Malik Ambar in May 1626, his son Fath Khan surrendered to the
Mughals in 1633 and handed over the young Nizam Shahi ruler Hussain Shah, who was
sent as a prisoner to the fort of Gwalior.
 But soon, Shahaji with the assistance of Bijapur, placed an infant scion of the Nizam
Shahi dynasty, Murtaza Nizam Shah III on the throne and he became the regent.
 In 1636 Aurangzeb, then Mughal viceroy of Deccan finally annexed the sultanate to the
Mughal empire after defeating Shahaji.

 Ahmednagar sultanate was dependent on Koli chieftains for military or soldiers. Koli
chieftains of provided the cavalry and infantry for Sultans of Ahmednagar during
wartimes

Contact : https://t.me/EN123upsc || Telegram Link : https://t.me/prelimbits


85

54. Who was the last ruler of the Tughlaq dynasty of the Delhi Sultanate? [2004]
A. Firoz Shah Tughlaq
B. Ghiyas-ud-din Tughlaq Shah II
C. Nasir-ud-din Mahmud
D. Nasrat Shah

Ans. C

TUGHLAQ DYNASTY (1320-1414 AD)


The Tughlaqs were also known as the Quaraunah Turks as the father of Ghazi Malik was a
Qaraunah Turk in origin

Ghiyasuddin Tughlaq (1320-1325)

 Took title: Ghazi (Founder of the Tughlaq dynasty)

 1st Sultan to start Irrigation.

 Built a strong fort called Tughlaqabad near Delhi

 Dispatched his son, Jauna Khan /Mohammad Bin Tughlaq, to re-establish authority in
Warangal (Kakatiyas) and Madurai (Pandayas).

 Troublesome relationship with the Sufi saint, Sheikh Nizamuddin Aulia.

 Amir Khusrau famous work ―Tughlaq Nama‖ deals with the rise of Ghiyasuddin Tughlaq

 Died due to the collapse of a wooden structure; some scholars like Ibn Batuta believe
that his death was an act of sabotage orchestrated by his son Jauna Khan.

Amir Khusrau (c.1252–1325 CE) was the most famous Persian writer of this period. He wrote a
number of poems. He experimented with several poetical forms and created a new style of
Persian poetry called Sabaq- i- Hind or the Indian style. He also wrote some Hindi verses. Amir
Khusrau‘s Khazain-ul-Futuh speaks about Alauddin‘s conquests

Mohammad Bin Tughlaq / Jauna Khan (1325-1351)

 His reign marks the zenith of the Delhi Sultanate but also saw the beginning of
disintegration.

 He defeated Mongols.

 Appointed official on the basis of merit.

 Advanced secular policies. Applied justice to Ulema.

 During his reign, the famous traveller Ibn Batuta (author of Safarnama Rehla) came to
India in c.1334 CE and acted as a Qazi at Delhi for eight years.

 He was the only Delhi Sultan who had received a comprehensive literacy,

Contact : https://t.me/EN123upsc || Telegram Link : https://t.me/prelimbits


86

religious and philosophical education. (Most learned Man of D. Sul.)


 His ideas/decisions were ahead of his time hence he was not successful of a ruler.

 He Built fort of Adilabad and the city of Jahanpanah.

 During his reign 3 major kingdoms of South India emerged: Vijayanagar, Bahamani, and
Madurai.

 Imposed additional cesses called Abwabs on the peasants.

Muhamma Tughlaq‘s Experiments

 Transferred capital from Delhi to Devagiri (Daulatadab). Shifted back after 2 years
because of lack of water supply.
 Token Currency- Issued bronze coins at par with the value of the silver tanka coins. He
had also introduced the copper currency system. Later withdrew both.
 He launched Khorasan project to counter the threat of Chinses incursions – Failed.
 Quarachi expedition – launched in Kumaon hills to counter the threat of Chinses
incursions – failed.
 He set up Diwan -i- amir- kohi a separated department to extend cultivation by giving
loans to cultivators (takkavi loans) – Failed due to corrupt officials
Firoz Shah Tughlaq (1351-1388)

 Adopted policy of trying to appease the nobles, army, theologians and of asserting his
authority over only such areas which could be easily administered from the center.

 Appointed Khan-i-Jahan Maqbal, a Telugu Brahmin as Wazir or prime minister.

 He extended the principle of heredity to the army & nobility.

 Thus, the iqta system was not only revived, but also it was made hereditary.

 Malik Sarwar was prominent noble and had been wazir for some time. He asserted
independence and assumed title of Malik-us-Sharq (lord of the east).

 Malik ruled from Jaunpur, it was called Shiraz of the east. Malik Muhammad Jaisi author
of ―Padmavat‖ lived in Jaunpur.

 To appease theologians, Firoz took following decisions:

o Prohibited practice of Muslim women going out to worship.

o Gave concessions to theologians

o Made jizya a separate tax. Earlier it was part of land revenue. Only children,
women, disabled exempted.

o Erased wall paintings in his palace

 He constructed and improved several canals.

Contact : https://t.me/EN123upsc || Telegram Link : https://t.me/prelimbits


87

 He set up hospitals for poor called – Dar-ul-shifa.

 Established town of Hissar , Firozabad (Firoz Shah Kotla) & Jaunpur


 Set up new departments:

o Diwan -i-Khairat - Care of orphans and widows. [Department of Charity]

o Department for public work.

o Diwan -i-Bandagan – Department for slaves

 Introduced 2 new coins: Adha (50% Jital) and Bitch (23% Jital).

 He led two unsuccessful expeditions to Bengal. Bengal became free from the control of
Delhi Sultanate.

 He developed royal factories called karkhanas in which thousands of slaves were employed.

 Imposed four taxes sanctioned by Islamic kharaj (land tax), khams (1/5 of the looted
property during wars), Jizya (religious tax on the Hindus), and Zakat (2½per cent of the
income of the Muslims which was spent for the welfare of Muslim subjects and their
religion).

 He was first Sultan to impose Sharb (irrigation tax).

 Added last two storeys to Qutub Minar

 Ashokan pillars from Topara and Meerut were brought to Delhi.

 Patronised scholars like Barani (famous historian who wrote Tarikh-i-Firoz Shah and
Futwa-i-Jahandari) and Khwajja Abdul Malik Islami (who wrote Futah-Us- Sulatin).
He himself wrote the Futuhat-e-firozshahi.

Nasiruddin Muhammad (1390-1398)

 last ruler

 Taimur‘s invasion (1398) during his reign weakened the sultanate.

 When Timur entered Delhi there was no opposition. He withdrew from India in 1399.

 Delhi sultanate disintegrated towards the beginning of 15th century and no of


independent states set up. E.g. Malwa and Gujarat etc.

 Tughlaq empire came to end in 1412.

55. Consider the following statements: [2004]


1. Narasimha Saluva ended the Sangama dynasty and seized the throne for himself and
started the Saluva dynasty.
2. Vira Narasimha deposed the last Saluva ruler and seized the throne for himself.

Contact : https://t.me/EN123upsc || Telegram Link : https://t.me/prelimbits


88

3. Vira Narasimha was succeeded by his younger brother, Krishnadeva Raya.


4. Krishnadeva Raya was succeeded by his half-brother, Achyuta Raya.

Which of the statements given above are correct?

A. 1, 2 and 3
B. 2, 3 and 4
C. 1 and 4
D. 1, 2, 3 and 4

Ans. D

TULUVA DYNASTY (1491 – 1570 CE)


 Tuluva Narasa Nayaka was the founder of the Tuluva dynasty of the Vijayanagara Empire.
He was the father of Emperor Krishnadevaraya.
 After the death of king Saluva Narasimha in 1491, crown prince Thimma Bhupala was
murdered by an army commander. The faithful Narasa Nayaka then crowned the other
prince, Narasimha Raya II but retained all administrative powers in order to bring
stability to the kingdom.
 Narasimha Raya II was a teenager when he became Emperor of the Vijayanagara Empire,
and real power lay in the hands of his guardian, Tuluva Narasa Nayaka.
 In 1494, Narsa Captured Narasimha II in Fortress of Penukonda. Narsa ruled Vijayanagar
in disguise of Narasimha raya II.
 After the death of the Tuluva Narasa Nayaka resulted in feudatories rising in rebellion
everywhere. In his writings, Fernão Nunes noted that the whole world had risen in rebellion.
 At first, Immadi Narasa Nayaka, the eldest son of saluva Narasa Nayaka became king and
lasted at the throne for two years before being assassinated. Vira Narasimha Raya was
next crowned in 1505 and spent all his years fighting rebel warlords.

IN THE VIJAYANAGARA EMPIRE, FOUR DYNASTIES RULED THE WHOLE OF SOUTH INDIA FOR 310
YEARS.

Contact : https://t.me/EN123upsc || Telegram Link : https://t.me/prelimbits


89

56. Which one of the following sequences indicates the correct chronological order?
[2004]
A. Shankaracharya–Ramanuja–Chaitanya
B. Ramanuja–Shankaracharya–Chaitanya
C. Ramanuja–Chaitanya–Shankaracharya
D. Shankaracharya–Chaitanya–Ramanuja

Ans. A

 Shankaracharya (8th Century CE)


 Ramanuja (1017-1137 CE)
 Chaitanya (1486-1533 CE)

ADI SHANKARACHARYA
 Known as Adi Shankara, born 11th May 788 AD, at Kaladi near Kochi, Kerala.
 Took Samadhi at the age of 33, at Kedar tirth.
 He was a devotee of Shiva.
 Propounded the Doctrine of Advaita (Monism) and wrote many commentaries on the
Vedic canon (Upanishads, Brahma Sutras and Bhagavad Gita) in Sanskrit.

Contact : https://t.me/EN123upsc || Telegram Link : https://t.me/prelimbits


90

 He was opposed to Buddhist philosophers.


 Major Work:
o Brahmasutrabhasya (Bhashya or commentary on the Brahma Sutra).
o Bhajagovinda Stotra.
o Nirvana Shatakam.
o Prakaran Granths.
 Other Contributions:
o Was responsible for reviving Hinduism in India to a great extent when Buddhism
was gaining popularity.
o Established four Mathas in the four corners of India at Shingeri, Puri, Dwaraka and
Badrinath– for propagation of Sanathana Dharma.

Advaita Vedanta

 It articulates a philosophical position of radical nondualism, a revisionary worldview which it


derives from the ancient Upanishadic texts.
 According to Advaita Vedantins, the Upanishads reveal a fundamental principle of
nonduality termed ‗brahman‘, which is the reality of all things.
 Advaitins understand brahman as transcending individuality and empirical
plurality. They seek to establish that the essential core of one‘s self (atman) is brahman.
 The fundamental thrust of Advaita Vedanta is that the atman is pure non-intentional
consciousness.
 It is one without a second, nondual, infinite existence, and numerically identical with
brahman.

RAMANUJARCHARYA (1017-1137 A.D)


 Birth - Tamilandu

 Guru – yadavapreksha

 Propagator of Vishishtadvaita Vedanta or qualified monism → There exists a plurality


and distinction between Ātman (soul) & Brahman / Vishnu (metaphysical, ultimate
reality). According to him, God is Saguna Brahman.

o Brahman as defined by Ramanuja is an entirely personal God. Ramanuja


considered Brahman to be Vishnu or one of his avatars.

 The practice of religion in Vishista Advaita is similar to Dualism, and the only difference is
that mankind enjoys a higher status than in pure dualistic worship and is
nearer to God.

o Thus in Vishista Advaita, although both the world and Brahman are considered
equally real, they are not considered two separate entities as in Dualism.

 He also advocated prabattimarga or the path of selfsurrender to God. He invited


Contact : https://t.me/EN123upsc || Telegram Link : https://t.me/prelimbits
91

the downtrodden to Vaishnavism and advocated salvation by bhakti.

 Exponents of the Sri Vaishnavism tradition within Hinduism

 Literary works: Vedartha Sangraham, Sri Bhashyam, Gita Bhashyam

CHAITANYA MAHAPRABHU (1486-1533 AD)


 Founder of modern Vaishnavism (Krishna cult) in Bengal.

 He was a Saguna and popularized ―Kirtans‖ (religious songs) as a form of worshipping God.

 Popularized the chanting of ―Hare Rama, Hare Krishna‖.

 He made famous the practice of worshipping Radha and Krishna together.

 He gave the ―Achintaya Bheda-Abheda‖ philosophy.

 Didn‘t reject scriptures or idol worship called god as Hari.

 He wrote ―Siksastakam‖, a text in Sanskrit, where he elaborated his philosophy.

 He is the inspiration behind the world renowned ISKCON (International Society for
Krishna Consciousness) that was founded in the 20th century.

57. How did Sultan Qutb-ud-din Aibak die? [2003]


A. He was treacherously stabbed to death by one of his ambitious nobles
B. He was killed in a battle with Taj-u-din Yildiz, the ruler of Ghazni who entered into a contest
with him over the capture of Punjab
C. He sustained injuries while besieging the fortress of Kalinjar in Bundelkhand and succumbed
to them later
D. He died after a fall from his horse while playing Chaugan

Ans. D

Qutbud-din Aibak died after a fall from his horse while playing chaugan (polo) in 1210. He was
succeeded by Aram Shah. Qutbud-din Aibak was the founder of the first independent Turkish
kingdom in Northern India in 1206.

SLAVE DYNASTY (1206-1290 AD) → QUTUB-UD-DIN AIBAK (1206-1210)


 Qutub-ud-din Aibak Founded Slave dynasty. It is also known as Mamluk Dynasty. Aibak
is considered as real founder of Muslim rule in India
 In fact, three dynasties were established during this period. They were:
o Qutbi dynasty (c.1206–1211 CE ) founded by Qutbuddin Aibak.
o First Ilbari dynasty (c.1211–1266 CE) founded by Iltutmish.
o Second Ilbari dynasty (c.1266–1290 CE) founded by Balban.

Qutub-ud-din Aibak (c.1206–10 CE)

 Founder of the Slave dynasty; considered by many scholars as the real founder of Muslim
Contact : https://t.me/EN123upsc || Telegram Link : https://t.me/prelimbits
92

rule in India.
 He was a Turkish slave of Muhammad Ghori, who played an important role in the
expansion of the Turkish Sultanate in India after the Battle of Tarain and was made the
governor of Ghori‘s Indian possessions.
 He raised a standing army and established his hold over north India even during the
lifetime of Ghori.
 After the death of Ghori in c. 1206 CE, he became independent

 He Made Lahore capital.

 He is Known as ‗lakh Baksh‘ or giver of lakhs as he gave a lot of liberal donations.

 Aibak constructed →

o Quwwat-ul-Islam (Delhi)

o Arhai din ka Jhonpara (Ajmer)

 He Started construction of the Qutub Minar dedicated to Sufi Saint, Khwaja


Qutubuddin Bakthiyar Kaki - later completed by Iltutmish.

 He patronized Fakhar-ud-Din author of Tarikh-i-Mubarak Shahi & Hasan Nizami


who wrote Tajul-Ma'asir, the first official history of the Delhi Sultanate.

 Qutb-ud-din Aibak imposed jizya on non-Muslims for the first time.

The story of jizya


 Historically, the jizya tax has been understood in Islam as a fee for protection provided by
the Muslim ruler to non-Muslims, for the exemption from military service for non-
Muslims, for the permission to practice a non-Muslim faith with some communal
autonomy in a Muslim state, and as material proof of the non-Muslims‘ submission to
the Muslim state and its laws.

 In India, Islamic rulers imposed jizya on non-Muslims starting with the 11th century.

 The taxation practice included jizya and kharaj taxes. These terms were sometimes used
interchangeably to mean poll tax and collective tribute, or just called kharaj-o-jizya.

 Qutb-ud-din Aibak imposed jizya on non-Muslims for the first time.

 Alauddin Khilji, legalized the enslavement of the jizya and kharaj defaulters.

 Ahmad Shah (1411-1442), a ruler of Gujarat, introduced the Jizyah in 1414 and collected
it with such strictness that many people converted to Islam to evade it.

 Zizya was later abolished by the third Mughal emperor Akbar, in 1579. However, in 1679,
Aurangzeb chose to re-impose jizya on non-Muslim subjects

Arch And Dome


 Arches were introduced to India by Qutb al-Din Aibak during the year 1206. Domes were

Contact : https://t.me/EN123upsc || Telegram Link : https://t.me/prelimbits


93

introduced by the Turks during the 1500s by Persian architect. Minarets were introduced
by Qutb-Ud-Din Aibak.
o The architecture and art of Turks and Mughals were very different from that of
Indians.
o Under the reign of Qutb al-Din Aibak the Delhi Sultanate during the
year 1206 the Central-Asian styles and designs were introduced and introduced
the Islamic state to India. Domed tomb of Humayun is the earliest and major
building of Mughal reign during 1562 and 1571 by a Persian architect.
o Arches, domes, towers, minarets and decorations was introduced by the Turks
and used Arabic script.
o During the 1500s Akbar, third Mughal Empire expanded vastly during his reign and
brought several changes to his empire culturally, politically and economically.
o Qutb-Ud-Din-Aibak brought the minaret structure to India.
o The earliest mosque standing in India is in Qutb Minar in Delhi and the mosque is
known as ‗Quwwat-ul-Islam‘ or popularly known ‗Might of Islam‘ is in India
since 1193.

58. Consider the following statements: [2003]


1. Kitab-i-Nauras, a collection of songs in praise of Hindu deities and Muslim saints, was
written by Ibrahim Adil Shah II
2. Amir Khusrau was the originator in India of the early form of the musical style known as
Qawali

Which of these statements is/are correct?

A. Only 1
B. Only 2
C. Both 1 and 2
D. Neither 1 nor 2

Ans. C

ADIL SHAHIS OF BIJAPUR (1490-1686 AD)

 Founder - Yusuf Adil Shah.

 Mohd. Adil Shah built the world‘s second largest dome (Gol Gumbaz of Bijapur). [1st is St
Peter's Basilica in Rome]

Contact : https://t.me/EN123upsc || Telegram Link : https://t.me/prelimbits


94

 Bijapur was conquered & annexed by Aurangzeb in 1687

Ibrahim Adil Shah II (1580 – 1627)


 Known for religious tolerance and was given title of Jagadguru Budshah owing to his
belief in secularism.

 He tried to bring in cultural harmony between the Shias and the Sunnis and between
Hindus and Muslims through music.

 He was a great lover of music. He wrote the book Kitab-E-Navras (Book of Nine
Rasas) in Dakhani.

o The fifty-nine songs and seventeen couplets of the book are written in Dakhni Urdu
and indicate the ragas in which they were to be sung. Ibrahim was deeply devoted
to music and was himself a dhrupad-singer and the player of a stringed lute-like
instrument called tambur.

o The first verse of the Kitab is an invocation to Saraswati, and the second verse
invokes Prophet Muhammad and the Sufi saint Gesu Daraz.

o Subsequent verses extol the quest for knowledge as the most important pursuit in
life.

o Several verses explore traditions of love-poetry, finding similes and metaphors to


describe the beloved; others speak of the beauty of music or describe ragas as
personifications.

 He maintained band of musicians was known as Lashkar-e-Nauras (Army of Nauras) as


they were paid by the government regularly.

AMIR KHUSRAU (C.1252–1325 CE)


 Was the most famous Persian writer of this period.
 Patronized by Alauddin Khalji
 He created a new style of Persian poetry called Sabaq- i- Hind or the Indian style.
 He also wrote some Hindi verses.
 Amir‘s Khazain-ul-Futuh speaks about Alauddin‘s conquests.
 His famous work, the Tughlaq Nama, deals with the rise of Ghiyasuddin Tughlaq.
 Amir Khusrau introduced many new ragas → Ghora and Sanam.
 He also introduces:
o Qawwalis → Father of Qawali
o Sitar
o Ghazal
 Khusrau is sometimes referred to as the "voice of India" or "Parrot of India" (Tuti-e-
Hind), and has been called the "father of Urdu literature.

59. How did the Mughal Emperor Jahandarshah's reign, come to an early end? [2003]
A. He was deposed by his Wazir
B. He died due to a slip while climbing down the steps
Contact : https://t.me/EN123upsc || Telegram Link : https://t.me/prelimbits
95

C. He was defeated by his nephew in a battle


D. He died of sickness due to drinking

Ans. C

JAHANDAR SHAH [FEB 1712-FEB 1713]


 Jahandar Shah was a degenerate prince wholly devoted to pleasure. His administration
was virtually in the hands of the extremely capable Zulfiqar Khan, who had become his
wazir or prime minister.
 Zulfiqar believed that it was important to establish friendly relations with the Rajput
rajas and the Maratha sardars. Therefore, he rapidly reversed the policies of Aurangzeb.
o Marathas : Zulfiqar Khan confirmed the agreement reached between his deputy of
Deccan, Daud Khan Panni, and Shahu in 1711, whereby the Marathas were
granted the chauth and sardeshmukhi of the Deccan on the condition that these
collections would be made by Mughal officials and then handed over to the
Marathas.
o Rajputs : Jai Singh of Amber was appointed Governor of Malwa while Ajit Singh
of Marwar was appointed Governor of Gujarat.
o Jats, Bundelas, Sikhs : Zulfiqar Khan continued to pacify Churaman Jat and
Chhatrasal Bundela. He also continued to suppress the Sikhs.
 He also abolished the hated jeziah / Jizya
 He introduced the evil practice of revenue farming or Ijarah / Izara whereby the
government established contact with the revenue farmers and middlemen who paid
the government a fixed amount while they were free to extract whatever they could
from the peasant, leading to increased oppression of the peasantry.
 Jahandar Shah was soon defeated by his nephew Farrukhsiyar at Agra, thus bringing an
end to the reign of Jahandar Shah. Zulfiqar Khan was also executed.

FARRUK SIYAR [1713-1719]

 Killed Jahandar Shah with the help of Sayyid Brothers, The King Makers (Abdulllah
Khan and Hussain Ali).
o Farrukhsiyar appointed Abdullah Khan as his wazir and Hussain Ali Khan as the
Mir Bakshi.
 Brothers adopted the policy of Religious tolerance
 They continued to conciliate the Marathas, the Rajputs and the Jats while suppressing the
Sikhs. Farrukhsiyar‘s reign saw the victory of the Mughals over the Sikhs as the Sikh
leader Banda Bahadur was taken prisoner at Gurdaspur and later executed
 Abolished Jizya and Pilgrimage tax.
 First emperor to be killed by Nobles with the help of Peshwa Balaji Vishwanath,
 Issues three Farmans in 1717 to English (Magna Carta of the Company).

Contact : https://t.me/EN123upsc || Telegram Link : https://t.me/prelimbits


96

60. Battle of Dharmat was fought between [2003]


A. Muhammad Ghori and Jai Chand
B. Babur and Afghans
C. Aurangzeb and Dara Shikoh
D. Ahmad Shah Durrani and the Marathas

Ans. C

BATTLE OF DHARMAT
 The battle of Dharmat was fought during the Mughal War of Succession (1658–1659) by
Aurangzeb against Jaswant Singh Rathore who was allied with the Mughal prince Dara
Shikoh [son of Shahjahan].
 The battle was fought on the open plain of Dharmat on the hot Summer day of 15th April
1658 in which Aurangzeb won a decisive victory due to advantage in artillery and tactics.

61. Alam Khan, one of those who invited Babur to invade India was: [2003]
A. an uncle of Ibrahim Lodi and a pretender to the throne of Delhi
B. a cousin of Ibrahim Lodi who was ill-treated and expelled from the country
C. the father of Dilawar Khan to whom cruel treatment was meted out by Ibrahim Lodi
D. a high official in Punjab province who was discontented with Ibrahim Lodi's treatment of his
tribe

Ans. A

FIRST BATTLE OF PANIPAT


 The first Battle of Panipat, on 20 April 1526, was fought between the invading forces of
Babur and the Lodi dynasty.
 It took place in North India and marked the beginning of the Mughal Empire and the
end of the Delhi Sultanate.
 This was one of the earliest battles involving gunpowder firearms and field artillery in
the Indian subcontinent which were introduced by Mughals in this battle.

Background

 After losing Samarkand for the second time, Babur gave attention to conquer
Hindustan as he reached the banks of the Chenab in 1519.
 Until 1524, his aim was to only expand his rule to Punjab, mainly to fulfil his ancestor
Timur's legacy, since it used to be part of his empire.
 At that time, most of North India was under the rule of Ibrahim Lodi of the Lodi dynasty,
but the empire was crumbling and there were many defectors.
 He received invitations from Daulat Khan Lodi, Governor of Punjab and Ala-ud-Din,
uncle of Ibrahim.

Contact : https://t.me/EN123upsc || Telegram Link : https://t.me/prelimbits


97

 He sent an ambassador to Ibrahim, claiming himself the rightful heir to the throne of the
country, however the ambassador was detained at Lahore and released months later.
 Babur started for Lahore, Punjab, in 1524 but found that Daulat Khan Lodi had been
driven out by forces sent by Ibrahim Lodi.
 When Babur arrived at Lahore, the Lodi army marched out and was routed. In response,
Babur burned Lahore for two days, then marched to Dipalpur, placing Alam Khan, another
rebel uncle of Lodi's, as governor.
 Alam Khan was quickly overthrown and fled to Kabul. In response, Babur supplied Alam
Khan with troops who later joined up with Daulat Khan Lodi and together with about 30,000
troops, they besieged Ibrahim Lodi at Delhi. He defeated them and drove Alam's army off;
and Babur realised Lodi would not allow him to occupy the Punjab. == > Battle of Panipat

62. [2003]
The following item consists of two statements: one labeled as the Assertion (A) and the
other as Reason (R). You are to examine these two statements carefully and select the
answers to these items using the codes given below:
Assertion (A): Emperor Akbar marched towards Afghanistan in 1581 with a huge army.
Reason (R): He was on his way to reclaim his ancestral country of Ferghana in Central
Asia.
A. Both A and R are individually true but R is the correct explanation of A
B. Both A and R are individually true but R is not the correct explanation of A
C. A is true but R is false
D. A is false but R is true

Ans. C

AKBAR → CAMPAIGNS IN AFGHANISTAN AND CENTRAL ASIA


 Following his conquests of Gujarat and Bengal, Akbar was preoccupied with domestic
concerns. He did not leave Fatehpur Sikri on a military campaign until 1581, when Punjab
was again invaded by his brother, Mirza Muhammad Hakim. Akbar expelled his
brother to Kabul and this time pressed on, determined to end the threat from
Muhammad Hakim once and for all.
 In August 1581, Akbar seized Kabul and took up residence at Babur's old citadel. He
stayed there for three weeks, in the absence of his brother, who had fled into the
mountains. Akbar left Kabul in the hands of his sister, Bakht-un-Nissa Begum, and
returned to India.
 He pardoned his brother, who took up de facto charge of the Mughal administration in
Kabul; Bakht-un-Nissa continued to be the official governor.
 A few years later, in 1585, Muhammad Hakim died and Kabul passed into the hands of
Akbar once again. It was officially incorporated as a province of the Mughal Empire

Contact : https://t.me/EN123upsc || Telegram Link : https://t.me/prelimbits


98

63. [2003]
Assertion (A): Shah Alam II spent the initial years as an emperor far away from his
capital.
Reason (R): There was always a lurking danger of foreign invasion from the northwest
frontier.
Select the answers

A. Both A and R are individually true but R is the correct explanation of A


B. Both A and R are individually true but R is not the correct explanation of A
C. A is true but R is false
D. A is false but R is true

Ans. C

SHAH ALAM II OR ALI GAUHAR (1759-1806)


 Third Battle of Panipat (1761).
o In 1761, after the battle of Panipat and before leaving Delhi (20th March 1761),
Ahmed Shah Abdali named Shah Alam II as emperor and Najib-ud-Daulah as Mir
Bakshi.
o Abdali last invaded India in 1767. His invasions hastened the downfall of the
Mughal Empire, creating anarchy and confusion all around.
o The Emperor Shah Alam II was not allowed to enter Delhi for 12 years (1760-72,
until he was escorted to his throne by the Marathas in 1772). Thus, he remained in
exile for 12 years and came to be known as the ‗Fugitive Mughal Emperor‘.
 During his absence, the Rohilla leaders Najib-ud-Daulah and later his son
Zabita Khan and grandson Ghulam Qadir exercised undisputed power at
Delhi.
 Battle of Buxar (1765): Treaty of Allahabad.
o According to the treaty, he was forced to grant the Diwani (right to collect revenue)
of Bengal, Bihar and Orissa to the British.
o In return, the emperor was given a subsidy of 26 lakh and the districts of Kora
and Allahabad. The Emperor resided in the fort of Allahabad for the next six years as
a virtual prisoner of the English.
 Ghulam Qadir (Zabita Khan‘s Son) deposed Shah Alam and later ruthlessly blinded him
(10 August 1788). Shah Alam was also referred to as the ‗blind Mughal Emperor‘.
o Mahadji Scindia, however, hunted him down and the blind Shah Alam was restored
as the king (October 1788).
 In 1803, Delhi was recaptured by the English after defeating Daulat Rao Scindia and the
blind Mughal emperor once again accepted the protection of the British.
o From 1803 till 1857, the Mughal dynasty merely served as a political front for the
Contact : https://t.me/EN123upsc || Telegram Link : https://t.me/prelimbits
99

English (the Company‘s coins continued to carry the stamp of Emperor Shah
Alam till 1835).
o Shah Alam‘s power was so depleted during his reign that it led to a saying in
Persian, ‗Sultanate-Shah Alam, Az Dilli ta Palam‘, meaning, ‗The kingdom of
Shah Alam is from Delhi to Palam‘, Palam being a suburb of Delhi.
 Shah Alam II also authored his own Diwan of poems and was known by the pen-name
Aftab
64. [2003]
Assertion (A): Saluva Narasimha put an end to the old dynasty and assumed the royal
title.
Reason (R): He wanted to save the kingdom from further degeneration and disintegration.
A. Both A and R are individually true but R is the correct explanation of A
B. Both A and R are individually true but R is not the correct explanation of A
C. A is true but R is false
D. A is false but R is true

Ans. A

SALUVA DYNASTY → SALUVA NARASIMHA DEVA RAYA


 Saluva Narasimha Deva Raya (or Saluva Narasimha, Saluva Narasimha I; 1431–1491 CE)
was an emperor of the Vijayanagara Empire from the Saluva Dynasty.
 He patronised Kannada poet Kavi Linga.
 In 1452, he was given the title Maha Mandaleshwara of Chandragiri during the reign of
Mallikarjuna Raya. His father Saluva Gunda was the governor of Chandragiri.
 After the death of Virupaksha Raya II and arrival of Prauda Deva Raya as the new
monarch of Vijayanagar, the empire plunged into neglect and anarchy.
 Seeing that a military coup was the only hope to save the kingdom, he despatched the
son of Tuluva Isvara, Tuluva Narasa Nayaka to the imperial capital of Vijayanagara. The
incumbent king Prauda Raya fell, thus starting the rule of Saluva Narasimha.

65. With reference to Sufism in Indian history, consider the following statements: [2002]
1. Sheikh Ahmad Sarhandi was a contemporary of Ibrahim Lodi
2. Sheikh Nasiruddin Chirag-I-Dehlavi was a disciple of Sheikh Nizamuddin Auliya
3. Aurangzeb was a contemporary of Sheikh Salim Chisti
4. Qadiri order of Sufis was first introduced in India by Sheikh Niamutullah and Makhdum
Muhammad Jilani

Which of these statements are correct?

A. 1 and 2
B. 1 and 3

Contact : https://t.me/EN123upsc || Telegram Link : https://t.me/prelimbits


100

C. 2 and 3
D. 2 and 4

Ans. D

 Sheikh Ahmad Sarhandi of Naqshbandi order was a contemporary of Akbar and


Jahangir.
o Akbar (1556-1605) & Jahangir (1605-1627) || Ibrahim Lodi (1517-1526)
 The Qadiri order of Sufis was first introduced in India by Sheikh Nizamatullah (He died
in 1430 AD) and Makhdum (or Nasiruddin) Muhammad Jilani (died in 1517).
 Aurangzeb (1658-1707) & Salim Chishti (1478–1572)
 Nasiruddin Mahmud Chirag-Dehlavi ( c. 1274–1337) was a 14th-century mystic-poet and
a Sufi saint of the Chishti Order. He was a disciple of Sufi saint Nizamuddin Auliya, and later
his successor. He was the last important Sufi of the Chishti Order from Delhi.

66. Historian Barani refused to consider the state in India under Delhi Sultans as truly
Islamic because [2002]
A. majority of the population did not follow Islam
B. Muslim the ologists were often disregarded
C. Sultan supplemented the Muslim law by framing his own regulations
D. religious freedom was accorded to non-muslims

Ans. C

BARANI (1285–1357)
 The historian Ziauddin Barani (1285–1357) was a political thinker of the Delhi Sultanate
during Muhammad bin Tughlaq and Firoz Shah's reign.
 He was best known for composing the Taarikh-i- Firoz Shahi a work on medieval India.
 He refused to consider the state in India under Delhi Sultans as truly Islamic as the Sultan
supplemented the Muslim law by framing his own regulations.

67. [2002]
Assertion (A): Muhammad Bin Tughlaq left Delhi, and for two years lived in a camp called
Swarga-Dwari.
Reason (R): At that time, Delhi was ravaged by the plague and many people died.
A. Both A and R are individually true but R is the correct explanation of A
B. Both A and R are individually true but R is not a correct explanation of A
C. A is true but R is 1 false
D. A is false but R is true

Ans. A

 Like Allauddin Khalji, Muhammad bin Tughlaq also tried to bring changes in the

Contact : https://t.me/EN123upsc || Telegram Link : https://t.me/prelimbits


101

agrarian setup.
 Some historians point out that he made an over-assessment because of which many
peasants fled the region. But the states share remained half.
 A severe famine in this period worsened the situation. Muhammad bin Tughlaq tried to
provide relief to those affected and made efforts to improve and extend cultivation.
 The sultan left Delhi and was rendered in a camp called swargadwari near kanauj. He
also set up a separate department called diwan-i amir-i kohi whose function was to extend
cultivation by providing loans.

MOHAMMAD BIN TUGHLAQ / JAUNA KHAN (1325-1351)


 His reign marks the zenith of the Delhi Sultanate but also saw the beginning of
disintegration.

 He defeated Mongols.

 Appointed official on the basis of merit.

 Advanced secular policies. Applied justice to Ulema.

 During his reign, the famous traveller Ibn Batuta (author of Safarnama Rehla) came to
India in c.1334 CE and acted as a Qazi at Delhi for eight years.

 He was the only Delhi Sultan who had received a comprehensive literacy,
religious and philosophical education. (Most learned Man of D. Sul.)
 His ideas/decisions were ahead of his time hence he was not successful of a ruler.

 He Built fort of Adilabad and the city of Jahanpanah.

 During his reign 3 major kingdoms of South India emerged: Vijayanagar, Bahamani, and
Madurai.

 Imposed additional cesses called Abwabs on the peasants.

Muhamma Tughlaq‘s Experiments

 Transferred capital from Delhi to Devagiri (Daulatadab). Shifted back after 2 years
because of lack of water supply.
 Token Currency- Issued bronze coins at par with the value of the silver tanka coins. He
had also introduced the copper currency system. Later withdrew both.
 He launched Khorasan project to counter the threat of Chinses incursions – Failed.
 Quarachi expedition – launched in Kumaon hills to counter the threat of Chinses
incursions – failed.
 He set up Diwan -i- amir- kohi a separated department to extend cultivation by giving
loans to cultivators (takkavi loans) – Failed due to corrupt officials

Contact : https://t.me/EN123upsc || Telegram Link : https://t.me/prelimbits


102

68. The motive behind Shah Jahan's Balkh campaign was to [2002]
A. secure a friendly ruler in Balkh and Badakshan which bordered Kabul
B. conquer Samarqand and Fargh ana, the Mughal homelands
C. fix the Mughal frontier on the 'scientific line', the Amu Daria
D. expand the Mughal Empire beyond the sub-continent

Ans. A

The motive behind Shah Jahan's Balkh and Badakshan campaign in central Asia was to secure the
defence of NorthWest India.

SHAHJAHAN (1628-1658)
 Succeeded Jahangir ad ascended throne in 1628.

 Three years after accession, his beloved wife Mumtaj Mahal died in 1631.

 In the north-west, the campaign to seize Balkh from the Uzbegs was unsuccessful and
Qandahar was lost to the Safavids.

 His Deccan policy was more successful. He defeated the forces of Ahmadnagar and
annexed it. Both Bijapur and Golkonda signed a treaty with the emperor.

 He defeated Portuguese in 1632.

 The court, army and household moved from Agra to the newly completed imperial capital,
Shahjahanabad. It was a new addition to the old residential city of Delhi, with the Red Fort,
the Jama Masjid, Chandni Chowk etc.

 Salutation: Under Shah Jahan it was chahar taslim and zaminbos (kissing the ground).

 During his reign WAR OF SUCCESSION among his sons broke out.

o Battle of Dharmat, Battle of Samugarh, Battle of Khajwah, and Battle of Deorai


took place & Finally Aurangzeb emerged victorious.

o Battle of Dharmat →

 The battle of Dharmat was fought during the Mughal War of Succession
(1658–1659) by Aurangzeb against Jaswant Singh Rathore who was allied
with the Mughal prince Dara Shikoh [son of Shahjahan].

 The battle was fought on the open plain of Dharmat on the hot Summer day
of 15th April 1658 in which Aurangzeb won a decisive victory due to
advantage in artillery and tactics.

 His court historian Abdul Hameed / Hamid Lohiri wrote ―Badusha nama‖.

 Shah Jahan Nama is written by Inayat Khan. His son translated Bhagavat Gita and
Upanishads into Persian language.

Contact : https://t.me/EN123upsc || Telegram Link : https://t.me/prelimbits


103

Architecture

 Built Taj Mahal in 1632-33 to perpetuate memories of his wife, Mumtaj Mahal.

 Moti Masjid (entirely of white marble), Sheesh Mahal and Mussaman Burj at Agra.

 Red Fort with its Rang Mahal, Diwan-i-Am and Diwan-i-Khas at Delhi.

 Jama Masjid in Delhi (red stone), Shalimar Bagh in Lahore and city of
Shahjahanabad.

 The pietra dura method was used on a large scale in the Taj Mahal by Shah Jahan.

69. With reference to medieval Indian rulers, which one of the following statements is
correct? [2002]
A. Alauddin Khalji first set up a separate ariz's department.
B. Balban introduced the branding of horses in his military.
C. Muhammad Bin Tughlaq was succeeded by his uncle to the military.
D. Firoz Tughlaq set up a separate department of slaves.

Ans. D

 Firoz Tughlaq set up a separate department of slaves known as 'Diwan-i-Bandagan'.


 Mohd Bin Tughlaq was succeeded by his cousin (not uncle) Firoz Tughlaq.
 Alauddin Khalji introduced the branding system of horses in his military.
 Balban introduced DIwan-i-Ariz was related to the Department of Military. It was
headed by Ariz-i-Mumalik.

SLAVE DYNASTY (1206-1290 AD) → BALBAN (1266-1287)


 Also known as Ulugh khan. He was one of the main architects of the Delhi Sultanate.

 According to him, the Sultan was God‘s shadow on earth (Zil-i-Ilahi) and recipient of divine
grace (Nibyabat- i-Khudai).

 He broke the power of the Forty.

 Introduced Persian festival Nawrouz.

 Called himself Nasir-amir-ul-momin (Caliph‘s right-hand man).

Administration
 Separated Diwan-i-wizarat (Finance Department) from the Diwan-i-Arz (Military
Department).

 Administered justice with extreme impartiality.

 He followed policy of blood & iron.

 Excluded non-Turks from administration.

Contact : https://t.me/EN123upsc || Telegram Link : https://t.me/prelimbits


104

 Indian Muslims not given important posts.

 Appointed spies to monitor the activities of the nobles.

 Introduced sijada (prostration) and paibos (kissing the Sultan‘s feet) to prove his
superiority over the nobles

ALAUDDIN KHALJI (1296-1316 CE)


 Was appointed as Amir-i-Tuzuk (Master of Ceremonies) and Ariz-i-Mumalik (Minister of
Law) by Jalauddin Khalji.

 Jalauddin‘s policy of tolerance was reversed by Alauddin Khalji & awarded harsh
punishment who opposed him.

 To keep hold on nobles –He prohibited to hold festivals, marriage without permission
of him, banned wines & intoxicants to avoid social gathering, reorganized spy services
(Barids).

 He was first sultan who separated religion from politics. He proclaimed ―Kingship knows
no kinship‖.

 Barani wrote book ‗Tarikh-i-Firuz Shahi.


 Patronized poets like-Amir Khusrau and Mir Hasan Dehlv.

 Though Aladuddin was illiterate, he adopted the title of Sikander-i-Azam and gave Amir
Khusrau the title of Tuti-i-Hind (Parrot of India).

 Malik Kafur was his slave – general. He led a series of expeditions in the southern part of
India, against the Yadavas (1308), Kakatiyas (1310), Hoysalas (1311) etc.

 Amir Khusrau‘s Khazain-ul-Futuh speaks about Alauddin‘s conquests.

 In 1303, Allauddin defeated Raja Ratan Singh of Chittor, including Rani Padmini other
Rajput women performed Jauhar (self-immolation) which is written in the book
'Padmavath' by malik Muhammad Jayasi

Architecture

 Famous gateway known as Alai Darwaza, Hauz Khas, Mhal Hazaar Satoon, Jamait Khana
Mosque, Alai Minar and constructed a new capital at Siri.

Military Reforms

 1st sultan to have large permanent standing army & paid in cash from royal treasury.

 Innovated: Chehra and Dagh system.

o CHEHRA – detailed description of each soldier

o DAGH- Branding of horse

Contact : https://t.me/EN123upsc || Telegram Link : https://t.me/prelimbits


105

Land Revenue administration

 Revenue was half of the produce & was based on land under cultivation.

 First ruler to fix land revenue in cash. It enabled him to pay his soldiers in cash.

 1st Delhi Sultan to order for Measurement Of Land before fixing land revenue.

 Biswa was standard unit of measurement. Imposed one fifth of the produce and along
with it house tax (Grahi / ghari) and pasture tax (Chari / charai ).

 Abolished Kismat-i-Khuti (Headman‘s cess). Confiscated religious endowments and free


lands (Inam and Wakt).

 Mukadam & Khuts had to pay taxes. (class of rural intermediaries)

 He created post of Mustakraj / Mustakharaj to collect revenue.

 He brought area surrounding to Delhi directly under the state for revenue purpose. Thus,
Iqta system was not applied there

 Akbar introduced the Nasq System of assessment.

 This system of assessment was widely used in Akbar's time. It meant a rough calculation of
the amount payable by the peasant on the basis of past experience. The peasant was given
remission in the land revenue if crops failed on account of drought, floods, etc.

 Under Akbar, Dahsala System of land revenue existed.

Architecture

 In 1296, Alauddin constructed the Hauz-i-Alai.

 Alauddin built the Siri Fort & camped in Siri during the Mongol invasion in 1303.

 He built the Qasr-i-Hazar Situn palace at the Siri fort.

 Alauddin constructed the Alai Darwaza - entrance gate to Qutub Minar

Market Reforms

 Fixed cost of all commodities. For this purpose, he set up 4 separate markets:

o for Grains

o for cloth, Sugar, dried fruits & oils

o for horses, slaves & cattle

o for miscellaneous commodities.

 Each market were under the control of a high officer → "Shahna-i-Mandi".

 He also appointed Naib-I-Riyasat & Diwan-I-Riyasat to keep check on these markets

Contact : https://t.me/EN123upsc || Telegram Link : https://t.me/prelimbits


106

 He also received daily reports of markets from Barids (intelligence officer) and
Munhiyans (secret spies).
 Reasons for Market Reforms:

o To enjoy the support of citizens

o To sustain large army with low salaries.

FIROZ SHAH TUGHLAQ (1351-1388)


 Adopted policy of trying to appease the nobles, army, theologians and of asserting his
authority over only such areas which could be easily administered from the center.

 Appointed Khan-i-Jahan Maqbal, a Telugu Brahmin as Wazir or prime minister.

 He extended the principle of heredity to the army & nobility.

 Thus, the iqta system was not only revived, but also it was made hereditary.

 Malik Sarwar was prominent noble and had been wazir for some time. He asserted
independence and assumed title of Malik-us-Sharq (lord of the east).

 Malik ruled from Jaunpur, it was called Shiraz of the east. Malik Muhammad Jaisi author
of ―Padmavat‖ lived in Jaunpur.

 To appease theologians, Firoz took following decisions:

o Prohibited practice of Muslim women going out to worship.

o Gave concessions to theologians

o Made jizya a separate tax. Earlier it was part of land revenue. Only children,
women, disabled exempted.

o Erased wall paintings in his palace

 He constructed and improved several canals.

 He set up hospitals for poor called – Dar-ul-shifa.

 Established town of Hissar , Firozabad (Firoz Shah Kotla) & Jaunpur


 Set up new departments:

o Diwan -i-Khairat - Care of orphans and widows. [Department of Charity]

o Department for public work.

o Diwan -i-Bandagan – Department for slaves

 Introduced 2 new coins: Adha (50% Jital) and Bitch (23% Jital).

 He led two unsuccessful expeditions to Bengal. Bengal became free from the control of
Contact : https://t.me/EN123upsc || Telegram Link : https://t.me/prelimbits
107

Delhi Sultanate.

 He developed royal factories called karkhanas in which thousands of slaves were employed.

 Imposed four taxes sanctioned by Islamic kharaj (land tax), khams (1/5 of the looted
property during wars), Jizya (religious tax on the Hindus), and Zakat (2½per cent of the
income of the Muslims which was spent for the welfare of Muslim subjects and their
religion).

 He was first Sultan to impose Sharb (irrigation tax).

 Added last two storeys to Qutub Minar

 Ashokan pillars from Topara and Meerut were brought to Delhi.

 Patronised scholars like Barani (famous historian who wrote Tarikh-i-Firoz Shah and
Futwa-i-Jahandari) and Khwajja Abdul Malik Islami (who wrote Futah-Us- Sulatin).
He himself wrote the Futuhat-e-firozshahi.

70. Who among the following was the first Bhakti saint to use Hindi for the propagation of
his message? [2002]
A. Dadu
B. Kabir
C. Ramananda
D. Tulsidas

Ans. C

Ramananda was a devotional poet and saint of the Vaishnava sect in the region of Northern India.
He was the first saint to compose his works and propagate the message in Hindi in order to make
the knowledge accessible to the masses.

RAMANANDA (1360-1470)
 Lived in the first half of the 15th century, born in Allahabad, and was originally a follower
of Ramanuja.

 He preached his principles in Hindi at Benaras and Agra.

 His most outstanding contribution is the abolition of distinctions of caste among his
followers. He looked upon Ram and not Vishnu as the object of bhakti.

 He worshiped Ram and Sita and came to be identified as the founder of the Ram cult in
north India.

 His followers are called Ramanandis, like Tulsidas.

 He put emphasis on bhakti and avoided both gyana marg and karma marg.

 Gave rise to two schools of thought →


Contact : https://t.me/EN123upsc || Telegram Link : https://t.me/prelimbits
108

o Orthodox school – Represented by Nabhadas, Tulsidas

o Liberal – Represented by Kabir, Nanak, and others

 Other followers included →

o Raidasa – A cobbler whose songs are included in the Guru Granth Sahib

o Kabir – A weaver who preached that Ram, Rahim, and Allah are all the same

o Sena –A barber

o Sadhana – A butcher

o Dhanna -A farmer

o Naraharai - A goldsmith

o Pipa - A Rajput prince

71. [2001]
Assertion (A): The battle of Khanwa was certainly more decisive and significant than the
First Battle of Panipat
Reason (R): Rana Sanga, the Rajput hero, was certainly a more formidable adversary than
Ibrahim Lodi.
A. Both A and R are individually true but R is the correct explanation of A
B. Both A and R are individually true but R is not a correct explanation of A
C. A is true but R is false
D. A is false but R is true

Ans. A

Battle of Khanua (1527, Rana Sanga was defeated by Babar); First Battle of Panipat (1526, Ibrahim
Lodhi was defeated by Babur). Rana Sangha once defeated Ibrahim Lodhi so R explains A.

MEWAR
 Mewar or Udaipur Kingdom was originally called Medhpaat and over time, the name
Medhpath became Mewar. Interestingly, the rulers of Mewar used the title Maharana
(Prime Minister or Custodian) instead of the typical title “Maharaja” (King), as they
believed themselves to be merely custodians of the Hindu civilisation epitomised in the
temple of their lord Eklingaji (a manifestation of Lord Shiva), who was also called
Medhpateshwar (Lord of Medhpath).
 The rise of Mewar during the 15th century was an important factor in the political life
of north India.
 The power of the Chauhans in Rajputana came to an end with the conquest of
Ranthambhore by Alauddin Khalji.
 After being overrun by the armies of Alauddin Khalji, Mewar had become relatively

Contact : https://t.me/EN123upsc || Telegram Link : https://t.me/prelimbits


109

insignificant.
 Later in c.1335 CE, Rana Hammira (c.1314– 78 CE) established the Second Guhila
dynasty of Chittor and also became the progenitor of the Sisodia clan, a branch of the
Guhilot clan, to which every succeeding Maharana of Mewar has belonged.
 He was the first ruler who started the use of title Rana and also built the Annapoorna
Mata temple, which is located in the Chittorgarh Fort in Rajasthan.
 After the assassination of Rana Hammira‘s grandson, Maharana Mokal, his son Rana
Kumbha, ascended the throne of Mewar in c.1433 CE.
Rana Kumbha (1433–1468 CE)

 Rana Kumbha or Kumbhakarna Singh raised Mewar to the status of a power

 As discussed above, Rana Khumba and Mahmud Khalji fought against each other, and
interestingly, both sides claimed victory.

 Kumbha erected a Victory Tower at Chittor a mark of victory of his conquests. He


also consolidated the fortification of Chittor and constructed a road running through its
seven doors.

 He was a great patron of art and literature and a great musician himself (he was a great
veena player). He was acclaimed as Sangeet Shiromani and wrote an outstanding
treatise on indian music titled Sangeet-Raj as well as other works like Sangeet Mimansa,
Sangeet Ratnakar & Sudprabandh.

 He also patronised great scholars such as Mahesh and Atri, the composers of the
inscriptions of Victory Tower.

 He constructed the city of Basantpur. He built several inns, palaces, ponds, schools, and
temples.

 For the defence of the kingdom, he built five forts of Kumbhalgarh, Achalgarh, Maddan,
Kolana, and Vairat

Rana Sanga (c.1508–1528 CE)


 Originally called Sangram Singh, the grandson of Rana Kumbha was a ferocious and
brave Rajput ruler who fought several battles.

 He united the various factions of Rajput under his able leadership.

 After first consolidating his power at Mewar, Rana Sanga moved his army against the
internally troubled neighbouring kingdom of Malwa (as during this period Malwa was
disintegrating).

 The Battle of Khanwa was fought at Khanwa which is 60 km west of Agra on March
16, 1527.

o It was fought between the forces of Babur and the Rajput confederacy led by
Rana Sanga for suprermacy of Northern India. Sanga was helped by→
Contact : https://t.me/EN123upsc || Telegram Link : https://t.me/prelimbits
110

 Hasan Khan Mewati and the Afghan Mahmud Lodhi and Raja Medini Rai
of Alwar

o The battle was one of the most decisive battles as the defeat of Rana Sanga was a
watershed event in the Mughal conquest of northern India.

 In c.1528 CE, he again fought Babur at Battle of Chanderi to help Medini Rai who was
attacked by Babur. But, he fell sick at Kalpi and died in his camp.

72. Which of the following pairs is correctly matched? [2001]


A. Dewan-i-Bandagani – Tughlaq
B. Dewan-i-Mustakhraj – Balban
C. Dewan-i-Kohi – Alauddin Khilji
D. Dewan-i-Arz – Muhammad Tughlaq

Ans. A

 Dewan-i-Bandagani (Department of Slaves) was constituted by Firoz Tughlaq


 Dewan-i-Mustakhraj (Dept of Arrears) – Alauddin Khalji
 Dewan-i-Kohi (Dept of Agriculture) – Muhammad Bin Tughlaq
 Dewan-i-Arz (Dept of Military) - Balban.

MAJOR DEPARTMENTS IN THE CENTRAL ADMINISTRATION OF THE DELHI SULTANATE


Department Category of Department Function of the Department
Diwan-i-Risalat Department of appeals In charge of foreign affairs and dealing with
diplomatic correspondences
Diwan-i-arz Department of Military Responsible for maintaining the royal army and
recruiting troops. This department was
introduced by Balban
Diwan-i- Department of pensions In charge of pensions
Ishtiaq
Diwan-i- Department of arrears This was the department of revenue
Mustakhraj
Diwan-i-kohi Department of agriculture In charge of managing the agricultural activities
in the kingdom. This department was created by
Muhammad Bin Tughlaq
Diwan-i-insha Department of In charge of the royal correspondence and the
correspondence regulations set by the Sultans were the basis of
law
Diwan-i- Department of slaves In charge of slaves
Bandagan
Diwan-i-Qaza Department of justice Managing justice in the empire
Diwan-i- Department of charity In charge of charity in the kingdom
Khairat

Contact : https://t.me/EN123upsc || Telegram Link : https://t.me/prelimbits


111

73. Which among the following ports was called Babul Makka (Gate of Makka) during the
Mughal period? [2001]
A. Calicut
B. Broach
C. Cambay
D. Surat

Ans. D

 Surat has also been called the gate to Mecca because many pilgrim ships set sail from here.

74. Mongols under Chengiz Khan invaded India during the reign of [2001]
A. Balban
B. Firoz Tughlaq
C. Iltutmish
D. Muhammad Bin Tughlaq

Ans. C

FIRST MONGOL INVASION


 The first Mongol invasion of India took place in the reign of Sultan Shams-ud-din
lltutmish.
 In 1221, the Mongols, under Genghis Khan, appeared for the first time on the banks of
the river Indus.
o Genghis Khan made several incursions into the Indian subcontinent when he chased
the Jalal al Din, the last ruler of the Khwarezmian Empire, all the way to the
Indus river in 1221.
o The Khwarezmian Empire was destroyed by the Mongols the year before and
Genghis Khan had ordered anyone member of the ruling family to be tracked down
and killed.
o In their pursuit of Jalal, the Mongols sacked several cities in the Punjab region
but their incursions were limited towards the western banks of the Sindh river
at the time.

SHAMSUDDIN ILTUTMISH (1210-1236)


 Belonged to Ilbari tribe hence named Ilbari Dynasty.
 Aibak had appointed him as iqtadar of Gwalior.
 Real consolidator of Turkish rule in India and also Delhi.
 He Shifted capital from Lahore to Delhi.
 Iltmush defeated Quabacha, who had declared himself independent ruler of Multan &
seized Lahore & parts of Punjab.
 Introduced Arabic coinage into India. Silver tanka became a standard coin in medieval

Contact : https://t.me/EN123upsc || Telegram Link : https://t.me/prelimbits


112

India and also issued Copper jital.


 He Started Iqta System.
 Completed the construction of Qutub Minar which was started by Aibak.
 Patronized Minas-us Siraj (writer) author of Minaj-us- siraj.
 Created a new class of ruling elite of forty powerful military leaders, ―The Forty‖
(Turkan-i-chahalgani).
 He nominated Raziya as his successor. Thus, he initiated hereditary succession to Delhi
Sultanate.

75. Match List-I with List-II and select the correct answer using the codes given below the
lists: [2001]

Codes

a) A – 2; B – 3; C – 1; D – 4
b) A – 3; B – 2; C – 4; D – 1
c) A – 3; B – 2; C – 1; D – 4
d) A – 2; B – 3; C – 4; D – 1

Ans. D

These Bhakti saints were disciples of Saint Ramananda.

RAMANANDA (1360-1470)
 Lived in the first half of the 15th century, born in Allahabad, and was originally a follower
of Ramanuja.

 He preached his principles in Hindi at Benaras and Agra.

 His most outstanding contribution is the abolition of distinctions of caste among his
followers. He looked upon Ram and not Vishnu as the object of bhakti.

 He worshiped Ram and Sita and came to be identified as the founder of the Ram cult in
north India.

 His followers are called Ramanandis, like Tulsidas.

 He put emphasis on bhakti and avoided both gyana marg and karma marg.

 Gave rise to two schools of thought →

Contact : https://t.me/EN123upsc || Telegram Link : https://t.me/prelimbits


113

o Orthodox school – Represented by Nabhadas, Tulsidas

o Liberal – Represented by Kabir, Nanak, and others

 Other followers included →

o Raidasa – A cobbler whose songs are included in the Guru Granth Sahib

o Kabir – A weaver who preached that Ram, Rahim, and Allah are all the same

o Sena –A barber

o Sadhana – A butcher

o Dhanna -A farmer

o Naraharai - A goldsmith

o Pipa - A Rajput prince

76. In which one of the following cities is the Lingaraja Temple located? [2001]
A. Bhubaneswar
B. Bijapur
C. Kolkata
D. Shravananbelagola

Ans. A

LINGARAJA TEMPLE (ODISHA)


 Lingaraja Temple is a temple DEDICATED TO SHIVA.
 Built in 7th C. by King Jajati Keshari of Soma Vansh.
 It is built in RED STONE and is a classic example of KALINGA STYLE of architecture.
 It signifies the syncretisation of Shaivism and Vaishnavism sects in Odisha
 Bhubaneswar is called the Ekamra Kshetra as the deity of Lingaraja was originally under a
mango tree (Ekamra) as noted in Ekamra Purana, a 13th-century Sanskrit treatise.
 Located to the north of the temple is Bindusagar Lake

Contact : https://t.me/EN123upsc || Telegram Link : https://t.me/prelimbits


114

77. Which one of the following Muslim rulers was hailed as the 'Jagadguru' by his Muslim
subject because of his belief in secularism? [2000]
A. Hussain Shah
B. Zain-ul-Abidin
C. Ibrahim Adil Shah
D. Mahmud II

Ans. C

ADIL SHAHIS OF BIJAPUR (1490-1686 AD) → IBRAHIM ADIL SHAH II (1580 – 1627)

 Founder - Yusuf Adil Shah.

 Mohd. Adil Shah built the world‘s second largest dome (Gol Gumbaz of Bijapur). [1st is St
Peter's Basilica in Rome]
 Bijapur was conquered & annexed by Aurangzeb in 1687

Ibrahim Adil Shah II (1580 – 1627)

 Known for religious tolerance and was given title of Jagadguru Budshah owing to his
belief in secularism.

 He tried to bring in cultural harmony between the Shias and the Sunnis and between
Hindus and Muslims through music.

 He was a great lover of music. He wrote the book Kitab-E-Navras (Book of Nine
Rasas) in Dakhani.

o The fifty-nine songs and seventeen couplets of the book are written in Dakhni Urdu
and indicate the ragas in which they were to be sung. Ibrahim was deeply devoted
to music and was himself a dhrupad-singer and the player of a stringed lute-like
instrument called tambur.

o The first verse of the Kitab is an invocation to Saraswati, and the second verse
invokes Prophet Muhammad and the Sufi saint Gesu Daraz.

o Subsequent verses extol the quest for knowledge as the most important pursuit in
life.

o Several verses explore traditions of love-poetry, finding similes and metaphors to


describe the beloved; others speak of the beauty of music or describe ragas as
personifications.
Contact : https://t.me/EN123upsc || Telegram Link : https://t.me/prelimbits
115

 He maintained band of musicians was known as Lashkar-e-Nauras (Army of Nauras) as


they were paid by the government regularly.

78. Match List-I with List-II and select the correct answer using the codes given below the
lists:

Codes

a) A – 3; B – 2; C – 1; D – 4
b) A – 2; B – 3; C – 4; D – 1
c) A – 2; B – 3; C – 1; D – 4
d) A – 3; B – 2; C – 4; D – 1

Ans. B

IQTA SYSTEM

 The Iqtadari was a unique type of land distribution and the administrative system
that evolved during the sultanate of Iltutmish.

 Under this system, the entire empire was very evenly divided into several large and small
tracts of land, called the Iqtas.

 These plots of land were assigned to the various nobles, officers, and soldiers FOR the
purpose of easy and Flawless Administration and Revenue Collection.

 The Iqtas were transferable, i.e., the holders of Iqtas-Iqtadars-were transferred from one
region to another every three to four years.

 The holders of small Iqtas were individual troopers. They had no administrative
responsibilities.

 Muhammad of Ghuri in 1206 A.D. the able king was the first to introduce the Iqta
system in India, BUT it was lltutmish who gave it an institutional form.

 The Iqta system was abolished by Alauddin Khilji. However, during Firuz Shah
Tughlaq's reign, in the year 1351 A,D, it became hereditary

THE JAGIRDARI SYSTEM


 Iqta of the Sultanate period in a modified form became Jagir under the Mughals.

 The Jagirdari system was the system of assignment of revenue of a particular

Contact : https://t.me/EN123upsc || Telegram Link : https://t.me/prelimbits


116

territory to the nobles for their services to the state was an integral part of the
Mansabdari system.
There were various types of jagirs:
 Tankha Jagirs: Which were given in lieu of salaries and they were transferable every
three to four years.

 Mashrut Jagirs: Which were given on certain conditions

 Watan Jagirs: Which were assigned to zamindar or rajas in their local dominions. Watan
Jagirs were hereditary and nontransferable.

 Altamgha Jagirs: Which were given to Muslim nobles in their family towns or place of
birth.

Along with Jagirdars, there were zamindars who were the people who had hereditary rights
over the produce of the land and claimed a direct share in the peasants produce.
Zamindars also assisted the state and the jagirdar in the collection of land revenue. They had
their own armed forces, and generally lived in forts or grahis, which was both a place of refuge
and a status symbol.

It should be noted that the zamindar was not the owner of all the lands comprising
his zamindari. The peasants who actually cultivated the land could not be dispossessed as
long as they paid the land revenue. Thus, the zamindars and the peasants had their own
hereditary rights in land.

AMARAM (VIJAYANAGAR EMPIRE)


 In the Nayakar System, military chiefs were assigned certain pieces of land or territory
called amaram.
 These chiefs were known as Amar Nayaks, had revenue and administrative rights on their
lands.

MOKASA SYSTEM
 To give Jagirs to officers was the Mokasa system in the administration of Shivaji.
 Shivaji organised a disciplined and efficient army.
 The ordinary soldiers were paid in cash, but the chief and military commander were
paid through jagir grants.

79. Consider the following events: [2000]


1. Reign of Krishna Deva of Vijaynagara
2. Construction of Qutab Minar
3. The arrival of Portuguese in India
4. Death of Firoz Tughlaq

Correct chronological sequence of these events is:


Contact : https://t.me/EN123upsc || Telegram Link : https://t.me/prelimbits
117

A. 2, 4, 3, 1
B. 2, 4, 1, 3
C. 4, 2, 1, 3
D. 4, 2, 3, 1

Ans. A

 Reign of Krishna Deva of Vijaynagara – (1509 – 29)


 Construction of Qutab Minar (1199 – 1220)
 Arrival of Portuguese in India (1498)
 Death of Firoz Tughlaq (1388).

KRISHNA DEVA RAYA (1509–29 CE)

 The greatest of the Vijayanagar rulers

o Aka. Abhinava Bhoja, Andhra Pitamah, Andhra Bhoj (as he was great patron of
literature and art).

 He fought a series of war with the independent kingdoms (Deccan Sultanates) that came
up on the ruins of the Bahmani kingdom. The Muslim armies were decisively defeated in the
battle of Diwani.

 Then he invaded the Raichur Doab and completely shattered the Adil Shahi forces of
Bijapur and set free the three Bahmani princes who were imprisoned there. He thus
restored the Bahmani Sultanate to Muhammad Shah.

o Krishna Deva himself took the title of Yavanarajya sthapanacharya.

 He maintained friendly relations with the Portuguese.

o Albuquerque sent his ambassadors to the court of Krishna Deva Raya.

o Portuguese travellers Domingo Paes and Barbosa came to India during his reign.

 Eight eminent scholars (Ashta diggajas) of Telugu adorned his court :


o Allasani Peddanna (the greatest, who was also called Andhra kavita Pitamaha)

 his important works include

 Manucharitam & Harikathasaram

o Tenali Ramakrishna, author of Panduranga Mahamatyam

o Pingali Suranna, author of:

 Garuda Puranam

 Prabhavatee

 Pradyumnam

Contact : https://t.me/EN123upsc || Telegram Link : https://t.me/prelimbits


118

 Raghava

 Pandaveeyam

 Kalapurnodayamu

o Nandi Thimmana
o Madayyagari Mallana
o Dhurjati
o Ayyalaraju Ramambhadrudu
o Ramarajabhushanudu
 He himself authored

o Amukthamalyadha (Telugu)

o Jambavati Kalyanam and Ushaparinayam (Sanskrit)

 He built some fine stone temples such as Hazara Ramaswamy temples at Vijayanagar.

 The famous temple of Tirupati greatly developed during his period

 He also founded a suburban township near Vijayanagar called Nagalapuram after his
mother.

QUTB MINAR
 Qutb Minar consists of FIVE STORIES of red and grey sandstone
 The Qutb Minar, also spelled Qutub Minar and Qutab Minar, is a minaret and "victory
tower" that forms part of the Qutb complex, which lies at the site of Delhi‘s
oldest fortified city, Lal Kot, founded by the Tomar Rajputs
 It is a UNESCO World Heritage Site in the Mehrauli area of South Delhi, India.
 It is one of the most visited tourist spots in the city, mostly built between 1199
and 1220.
 It can be compared to the 62-metre all-brick Minaret of Jam in Afghanistan,
of c. 1190, which was constructed a decade or so before the probable start of
the Delhi tower. The surfaces of both are elaborately decorated with inscriptions
and geometric patterns.
 The Qutb Minar has a shaft that is fluted with "superb stalactite bracketing
under the balconies" at the top of each stage.
 The Qutb Minar was built over the ruins of the Lal Kot, the citadel of Dhillika. Qutub
Minar was begun after the Quwwat-ul-Islam Mosque, which was started around 1192
by Qutb-ud-din Aibak, first ruler of the Delhi Sultanate
 It is usually thought that the tower is named for Qutb-ud-din Aibak, who began it. It is
also possible that it is named after Khwaja Qutbuddin Bakhtiar Kaki a 13th-century sufi
saint, because Shamsuddin Iltutmish was a devotee of his.

The Patrons and Architects


Contact : https://t.me/EN123upsc || Telegram Link : https://t.me/prelimbits
119

 Qutb-ud-din Aibak, a deputy of Muhammad of Ghor, who founded the Delhi Sultanate
after Muhammad of Ghor's death, started construction of the Qutb Minar's first story in
1199.
 Aibak's successor and son-in-law Shamsuddin Iltutmish completed a further three
stories.
 After a lightning strike in 1369 damaged the then top story, the ruler at the time, Firuz
Shah Tughlaq, replaced the damaged story and added one more. [fifth and final]
 Sher Shah Suri also added an entrance while he was ruling and the Mughal emperor
Humayun was in exile.

80. Who among the following streamlined the Maratha administration after Sambhaji?
[2000]
A. Raja Ram
B. Balaji Vishwanath
C. Ganga Bai
D. Nanaji Deshmukh

Ans. B

Sambhaji (1680-89) was succeeded by Rajaram, Shivaji II, Tarabai, and Shahu Ji. Balaji Vishwanath
played a crucial role in the final victory of Shahu by winning over almost all the Maratha sardars to
the side of Shahu.

MARATHA EMPIRE IN 18TH CENTURY


 After the Mughals retreated from the Deccan, the Marathas grew as a confederacy of
military leaders called Maratha sardars. Originally, the Maratha sardars only held
temporary assignments of land revenue. But in practice, they tended to become hereditary
once they established themselves. But the Maratha sardars lacked unity and vision for
founding an all India empire. However, they successfully waged continuous struggle
against the Mughals till they were destroyed.
Relationship with the Mughals

 Marathas had a long history of suppression by the Mughals, particularly during the 27-
year war started by Aurangzeb in 1681.
 The Marathas believed in Hinduism and the religious intolerance practiced by the
Mughal emperors further provoked the Maratha resentment.
 In 1689 Aurangzeb defeated and killed Shivaji‘s son and successor Sambhaji and
captured his son Shahu.
 Aurangzeb had treated Shahu and his mother with great dignity hoping to arrive at a
political agreement with Shahu. However, the whole Maratha nation rose in arms against
the Mughals in what was to become a people‘s war.
 The younger son of Shivaji, Raja Ram carried on the struggle till his death in 1700 and
thereafter the struggle was carried on further by his wife Tarabai, acting as a regent for
Contact : https://t.me/EN123upsc || Telegram Link : https://t.me/prelimbits
120

his minor son Shivaji II.

 In 1707 after Aurangzeb‘s death, his successors released Shivaji‘s grandson


Shahu in the hope of throwing an apple of discord in Maharashtra.
Coronation of Shivaji

 As expected by the Mughals, Shahu‘s release divided the Marathas into two rival camps
o one led by Tarabai at Kolhapur and the
o other led by Shahu at Satara.
 Tarabai, Shahu‘s aunt, declared Shahu as an imposter and refused to recognize Shahu‘s
right to succession. The consequence was a long civil war in which Shahu ultimately
emerged victorious mainly with the help of a group of new independent sardars
and an able Chitpavan Brahmin from Konkan, Balaji Vishwanath.
 In this way, the civil war between Shahu and Tara Bai gave rise to a new system of Maratha
government under the leadership of Balaji Vishwanath, the Peshwa (Prime
Minister) of King Shahu.
o This change marked the beginning of the second period in Maratha history—the
period of Peshwa domination. It was in this period that the Maratha state
transformed into an empire.
FIRST PESHWA-BALAJI VISHWANATH (1713-20)
 King Shahu appointed him as the Peshwa or Prime Minister in November 1713. By virtue
of his diplomacy and statesmanship, Balaji Vishwanath turned the office of the Peshwa as
the functional head of the Maratha Empire while Raja Shahu became a mere figurehead.
 Help to Sayyid Brothers:
o In 1719, Balaji helped the Sayyid brothers remove Farrukhsiyar from the Mughal
throne and place a puppet emperor in Delhi.
o Moreover, as a reward, he secured for the Maratha king a Mughal sanad (imperial
order).
 Mughal Sanad of 1719
o As per the Sanad, the Marathas were allowed to collect chauth (literally: one-fourth
of revenues) and sardeshmukhi (an additional 10% of revenues) from the entire
Deccan (Aurangabad, Berar, Bidar, Bijapur, Hyderabad and Khandesh) and
Karnataka.
o In return for the above privileges, the Marathas were to place at the Emperor s
disposal a contingent of 15,000 troops, maintain peace and order in the Deccan and
pay an annual tribute of 10 lakh to the emperor.
o In this way, the Maratha king became the sardeshmukh of the entire Deccan and
Karnataka. The sanad is also known in history as ‗the Magna Carta of the Maratha
dominion‘.

Contact : https://t.me/EN123upsc || Telegram Link : https://t.me/prelimbits


121

 Foundation of Maratha Confederacy

o For the efficient collection of chauth and sardeshmukhi of the Deccan, Balaji
Vishwanath assigned separate areas to Maratha sardars, thus laying the foundation
of the Maratha Confederacy.
o This system of watans and saranjams (grants of land) was just another form of
Jagir system which Balaji found as best suited to bring peace from anarchy at that
time.
o However, it later turned into a major Maratha weakness as it made the King largely
dependent on the sardars for his finances.
o The sardars now began to establish their control in distant lands of Mughal
Empire where they gradually settled down as more or less autonomous chiefs.
In this way, the conquests of the Marathas outside their original kingdom were not
made directly by the Maratha king or the Peshwa, but by the sardars with their own
private parties. During these conquests, they often clashed with one another and
did not hesitate to join hands with the enemies, be they the Nizams, the Mughals or
the English.
Balaji Vishwanath has been rightly called ‗the second founder of the Maratha State‘. After his
death in April 1720, he was succeeded by his 20-year-old son Baji Rao I

Contact : https://t.me/EN123upsc || Telegram Link : https://t.me/prelimbits


122

81. Match List-I with List-II and select the correct answer using the codes given below the
list: [2000]

Codes

a) A – 1; B – 3; C – 2; D – 4
b) A – 1; B – 4; C – 2; D – 3
c) A – 3; B – 4; C – 1; D – 2
d) A – 2; B – 1; C – 3; D – 4

Ans. B)

 Under the Mahalwari system, revenue, the settlement was to be made by village or estates
with the landlords.
 In the Ryotwari system, a direct settlement was made between the government and the
ryot (cultivator).
 In the Zamindari system, the land is allotted to revenue farmers of Zamindars (rent
collectors).

82. Which one of the following pairs is not correctly matched? [1999]
A. Jahangir: William Hawkins
B. Akbar: Sir Thomas Roe
C. Shahjahan: Travernier
D. Aurangzeb: Manucci

Ans. B

 Ralph Fitch - He was the first Britisher to visit India during Akbar's period. He talked about
miserable conditions of common man.
 Sir Thomas Roe also visited the court of Jahangir. He was an ambassador of James – I, king
of England.
 Tavernier - He was a French diamond trader, who visited India 6 times during Shah Jahan
and Aurangzeb period. He visited Golconda and gave information about Qutb Shahis also.
 Francois Bernier - He was a French doctor, who worked in the military of Shah Jahan. He

Contact : https://t.me/EN123upsc || Telegram Link : https://t.me/prelimbits


123

said there was no middle class in India (indicating feudal society). He described Mughal
emperors as kings of beggars and barbarians.
 Peter Mundy - He was a Britisher, who visited India during the rule of Shah Jahan. He
talked about famines and revolts against the king. He also visited Gujarat and Deccan.
 Niccolao Manucci - He wasn an Italian doctor who worked in the artillery of Dara Shikoh.
He wrote a book called Storia Do Mogor (Story of Mughals). He permanently settled in
India and died and buried here.
o Storia Do Mogor : It is an important account of the time of the later reign of Shah
Jahan and of the reign of Aurangzeb. He also documented folk beliefs including
witchcraft.

83. The striking feature of the Jama Masjid in Kashmir completed by Zain-ul-Abdin
include(s): [1999]
1. turret
2. similarity with Buddhist pagodas
3. Persian style

Which of the above statements is/are correct?

A. 1 alone
B. 1, 2 and 3
C. 2 and 3
D. 1 and 3

Ans. B

JAMIA MASJID, SRINAGAR (STARTED IN 1394 CE AND COMPLETED IN 1402 CE)


 Jamia Masjid is a mosque in Srinagar, Jammu and Kashmir, India. Situated at Nowhatta in
the middle of the Old City, the Mosque was commissioned by Sultan Sikandar in 1394 CE
and completed in 1402 CE, at the behest of Mir Mohammad Hamadani, son of Mir
Sayyid Ali Hamadani (Persian Poet), and is regarded as one of the most important
mosques in Kashmir.

 Architecture →
o The Jamia Masjid is heavily influenced by Persian architecture, with similarities to

Contact : https://t.me/EN123upsc || Telegram Link : https://t.me/prelimbits


124

the Buddhist pagodas.


o The structure occupies a total area of 384 x 381 feet and is quadrangular in shape
with four turrets.

84.'The king was freed from his people and they from their king'. On whose death did
Badauni comment this? [1999]
A. Balban
B. Ala-ud-din Khalji
C. Muhammad-bin-Tughlaq
D. Firoz Shah Tughlaq

Ans. C

It was on his death , that Badauni commented that 'The king was freed from his people and they
from their king'. He had been a man of controversies and crisis. He faced attacks of Mongols,
dissension within his own support group, and rebellions from a very large and diverse population.
In an effort to adapt to his growing empire, he attempted to shift his capital from Delhi to
Daulatabad, which was supposed to be a more central location, but it was a disastrous decision
and was costly. His reign was full of problems and hardships.

MOHAMMAD BIN TUGHLAQ / JAUNA KHAN (1325-1351)


 His reign marks the zenith of the Delhi Sultanate but also saw the beginning of
disintegration.

 He defeated Mongols.

 Appointed official on the basis of merit.

 Advanced secular policies. Applied justice to Ulema.

 During his reign, the famous traveller Ibn Batuta (author of Safarnama Rehla) came to
India in c.1334 CE and acted as a Qazi at Delhi for eight years.

 He was the only Delhi Sultan who had received a comprehensive literacy, religious and
philosophical education. (Most learned Man of D. Sul.)

 His ideas/decisions were ahead of his time hence he was not successful of a ruler.

 He Built fort of Adilabad and the city of Jahanpanah.

 During his reign 3 major kingdoms of South India emerged: Vijayanagar, Bahamani, and
Madurai.

 Imposed additional cesses called Abwabs on the peasants.

Muhamma Tughlaq‘s Experiments

 Transferred capital from Delhi to Devagiri (Daulatadab). Shifted back after 2 years
because of lack of water supply.

Contact : https://t.me/EN123upsc || Telegram Link : https://t.me/prelimbits


125

 Token Currency- Issued bronze coins at par with the value of the silver tanka coins. He
had also introduced the copper currency system. Later withdrew both.
 He launched Khorasan project to counter the threat of Chinses incursions – Failed.
 Quarachi expedition – launched in Kumaon hills to counter the threat of Chinses
incursions – failed.
 He set up Diwan -i- amir- kohi a separated department to extend cultivation by giving
loans to cultivators (takkavi loans) – Failed due to corrupt officials

85. One consistent feature found in the history of southern India was the growth of small
regional kingdoms rather than large empires because of : [1999]
A. absence of minerals like iron
B. too many divisions in the social structure
C. absence of vast areas of fertile land
D. scarcity of manpower

Ans. C

86. [1999]
Assertion (A): During the time of Akbar, for every ten cavalrymen, the mansabdars had to
maintain twenty horses.
Reason (R): Horses had to be rested while on march and replacements' were necessary
in times of war.
A. Both A and R are true but R is the correct explanation of A
B. Both A and R are true but R is not a correct explanation of A
C. A is true but R is false
D. A is false but R is true

Ans. A

MANSABDARI SYSTEM
Mansabdars

 Mansabdars were officers in Mughal administration.


 Those nobles who joined Mughal service were enrolled as mansabdars.
 The term mansabdar refers to an individual who holds a mansab (rank).
 The Mansabdars were appointed to all civil and military posts.
 They were liable to be transferred from one branch of the administration (civil) to another
(military).
How were Mansabdars recruited?

 The Mughals enrolled people of all races and religions into government jobs.
 A person wishing to join the royal service had to petition through a noble, who
presented a Tajwiz to the emperor.

Contact : https://t.me/EN123upsc || Telegram Link : https://t.me/prelimbits


126

o Tajwiz was a petition presented by a nobleman to the emperor, recommending


that an applicant be recruited as mansabdar.
 If the applicant was found suitable a mansab (rank) was granted to him.
 Higher mansabs were given to princes and Rajput rulers who accepted the suzerainty of the
emperor.
What does the term ‗Mansab‘ denote?

 The term ‗Mansab‘ denotes the rank (position) of a Mughal military officer.
 Higher the Mansab, higher the salary, status, and position of the officer.
 Though in administrative records there were 66 grades of mansabdars, but in practice there
were only around 33 mansabs.
Mansab → Zat and Sawar

 Initially, a single number represented the rank, salary, and the size of the contingent of the
mansabdar.
 However later, the rank of mansabdar came to be denoted by two numbers – Zat and
Sawar.
 Example: A a mansabdar with 5000 Zat and 2000 Sawar.
Sub-divisions of each Mansab (Rank)

 The ‗Zat‘ fixed the rank in the army. The salary of a Mansabdar was based on his Zat.
 The ‗Sawar‘ refered to cavalary men Mansabdar had to maintain. Mansabdar also had to
keep horses ready.
 Zat vs Sawar
o Zat — Denote the rank in the administration
o Zat — Denote the Salary of the Mansabdar
o Sawar — Denote the number of cavalry men Mansabdar had to maintain.
 Note: Even if the Sawar rank was higher, the mansabdar‘s position in the official hierarchy
would not be affected. It will be decided only by the Zat rank.
 For example, a mansabdar with 5000 Zat and 2000 Sawar was higher in rank than a
Mansabdar of 4000 Zat and 3000 Sawar.
 However, there were exceptions to this rule particularly when the mansabdar was serving in
a difficult terrain.
 In addition to meeting his personal expenses, the mansabdar had to maintain out of his
salary a stipulated quota of horses, elephants, camels, mules and carts. A mansabdar
holding a rank of 5,000 had to maintain 340 horses, 100 elephants, 400 camels, 100
mules and 160 carts.
 Handsome salaries were paid to a mansabdar →
o A mansabdar with a rank of 5,000 got a salary of 30,000 rupees per month, one of
o 3,000 could get 17,000 rupees,
o while a mansabdar of 1,000 got 8,200 rupees.
 The horses were classified into six categories and the elephants into five.
Contact : https://t.me/EN123upsc || Telegram Link : https://t.me/prelimbits
127

 For every ten cavalry men, the mansabdar had to maintain twenty horses for horses that
had to be provided rest while on a march and replacements were necessary in times of
war.
 A record was kept of the description (‗huliya‘) of each horseman under a mansabdar and
branding (‗dag‘) of horses to prevent corruption.
 The troops raised by the emperor but not paid directly by the state and placed under the
charge of mansabadars were known as Dakhili
Military responsibilities of Mansabdars

 The Mansabdar was required to maintain a specified number of cavalrymen.


 The Mansabdar was required to maintain a specified number of horses.
 The mansabdar had to bring his cavalrymen for review and get them registered.
 The mansabdar had also get their horses branded.
Hierarchy within the Mansabdars

 Amir: Those mansabdars whose rank was 1000 or below were called Amir.
 Great Amir: Those mansabdars above 1,000 were called Amir-al Kabir (Great Amir).
 Amir of Amirs: Some great Amirs whose ranks were above 5,000 were also given the title
of Amir-al Umara (Amir of Amirs).
Salary of Mansabdars: In cash and land

 The Mansabdars were paid according to their ranks. They were paid a good amount of
money.
 Those Mansabdars, who were paid in cash, were called Naqdi.
 Those Mansabdars who were paid through land (Jagirs) were called Jagirdars.
 It is to be remembered that it is not land that was assigned but only the right to collect
revenue or income from the piece of land.
 No mansabdar could hold on to the said Jagir for a long term as they were liable for
transfer. Mansabdars were not supposed to accumulate their salaries and wealth. After
the death of a mansabdar, all his jagirs and wealth was confiscated. As a result,
Mansabders used to spend lavishly. In short, they had no option but to spoil their earning.
The Mansabdars‘ (Jagirdar‘s) lands were not hereditary!

 The Mansab‘s post or honour or dignity was not hereditary and it lapsed after the
death or dismissal of the Mansabdar. Mansabdar‘s property returned to the state after his
demise.
Who introduced Mansabdari in India?

 The Mansabdar appears to be a Central Asian institution. There is a view that this
institution came to India with Babur. However, during Babur‘s time, instead of the term
of Mansabdar, the term Wajahdar was used.
 It was under the regime of Akbar when Mansabdari system became the basis of military

Contact : https://t.me/EN123upsc || Telegram Link : https://t.me/prelimbits


128

and civil administration.


Did Mansabdars reside in their Jagirs (land allotted to them)?

 All mansabdars did not reside in their own jagirs but used servants to collect revenues
there while they themselves served in another part of the empire.
 Note: The Delhi Sultanate (The Khalji and the Tughlaq monarchs) too appointed military
commanders as governers of territories. These land were called the Iqta and the
landholders were called Iqtadars or the muqti. Most of the Muqti stayed in their
Iqta unlike Jagirdars.
Iqtadari vs Mansabdari

Iqtadari System Mansabdari


Iqtadari system was used by the Delhi Sultans Mansabdari was used by Mughal rulers.
Whole land of the Empire was divided into two Whole land belonged to the Emperor
parts – one which belonged to Iqtedars and the
other which belonged to the emperor
Itqadar was the officer in charge of the revenue Jagirdar had law and order responsibility in
collection and distribution addition to the revenue collection.

Most of the Muqti stayed in their Iqta They don‘t


Hereditary System was never hereditary
The number of Mansabdars during Mugal Rule

 Akbar maintained 1,803 Mansabdars, by the end of the reign of Aurangzeb, their number
rose to 14,499.
 In Akbar‘s reign, there were 29 mansabdars with a rank of 5,000 zat; by Aurangzeb‘s reign
the number of mansabdars with a zat of 5000 had increased to 79.
 The increase of the number of Mansabdars during the reign of Aurangzeb
led to the Jagirdari and agrarian crisis which led to the collapse of the
Mansabdari system.
The fall of Manasabdari System

 In Akbar‘s reign, the system worked near perfect. The revenue collected by the
Mansabdar from his jagirs (and transferred to the Emperor) was enough to pay his assigned
salary as well.
 These jagirs, in the initial days, were carefully assessed so that their revenues were
roughly equal to the salary of the mansabdar.
 However, in the later stage, there was a shortage of jagirs. Also the size of the jagirs
started to shrink.
 In the Aurengazeb era, the revenue collected by Mansabdars for the government was not
enough to pay the salary assigned to them.
Mansabdari System → Important Terms

Contact : https://t.me/EN123upsc || Telegram Link : https://t.me/prelimbits


129

 Mashrut = conditional rank, which means an increase of sawar rank for a short period.
 Tajwiz: Tajwiz was a petition presented by a nobleman to the emperor, recommending
that an applicant be recruited as mansabdar.
 Du-aspah and Sih-aspah: These were features added later to the Mansabdari system by
Jahangir. This is a system whereby the selected nobles could be allowed to
maintain a larger quota of troopers, without raising their Zat rank. The
system was popular as ‗du-aspah‘ (a trooper with two horses) or ‗sih-aspah‘ (a trooper
with three horses) system. As you can understand, this was related to the sawar rank.
Long story in Short (Summary)

 Mansab system was a grading system used by the Mughal rulers to fix the rank and salary
of a Mansabdar, who were basically royal officers.
 The mansabdars were nobles who acted as military commanders, high civil and military
officers, and provincial governors.
 There was no distinction between the civil and military departments. Both civil and military
officers held mansabs and were liable to be transferred from one branch of the
administration to another.
 The rank of a Mansabdar was determined by the number of horses and cavalrymen he
maintained.

87. The first writer to use Urdu as the medium of poetic expression was: [1999]
A. Amir Khusrau
B. Mirza Ghalib
C. Bahadur Shah Zafar
D. Faiz

Ans. A

He composed poetry in Arabic and Persian besides being the first writer to use Urdu as a medium
of poetic expression.

AMIR KHUSRAU (C.1252–1325 CE)


 Was the most famous Persian writer of this period.
 Patronized by Alauddin Khalji
 He created a new style of Persian poetry called Sabaq- i- Hind or the Indian style.
 He also wrote some Hindi verses.
 Amir‘s Khazain-ul-Futuh speaks about Alauddin‘s conquests.
 His famous work, the Tughlaq Nama, deals with the rise of Ghiyasuddin Tughlaq.
 Amir Khusrau introduced many new ragas → Ghora and Sanam.
 He also introduces:
o Qawwalis → Father of Qawali
o Sitar

Contact : https://t.me/EN123upsc || Telegram Link : https://t.me/prelimbits


130

o Ghazal
 Khusrau is sometimes referred to as the "voice of India" or "Parrot of India" (Tuti-e-
Hind), and has been called the "father of Urdu literature.

88. Match List-I with List-II and select the correct answer using the codes given below the
lists: [1998]
List I List II
A. 1556 1. Battle of Haldi Ghati
B. 1600 2. Nadir Shah's capture of Delhi
C. 1680 3. Death of Shivaji
D. 1739 4. Grant of Charter to East India Company
5. Accession of Akbar
Codes

A. A – 3; B – 4; C – 2; D – 1
B. A – 5; B – 4; C – 3; D – 2
C. A – 5; B – 2; C – 1; D – 4
D. A – 1; B – 5; C – 3; D – 2

Ans. B

 The Battle of Haldighati (1576 ) = Mughal Empire vs the forces of Mewar


 Grant of Charter of EIC = 1600
 Death of Shivaji = 1680
 Nadir Shah‘s Capture of Delhi = 1739

BATTLE OF HALDIGHATI (1576)


 The Battle of Haldighati was a battle fought on 18 June 1576 between the Mewar forces
led by Maharana Pratap, and the Mughal forces led by Man Singh I of Amber.
 The Mughals carried the day after inflicting significant casualties on Mewar forces,
although they failed to capture Pratap, who reluctantly retreated persuaded by his fellow
commanders.
 The siege of Chittorgarh in 1568 had led to the loss of the fertile eastern belt of Mewar
to the Mughals.
 However, the rest of the wooded and hilly kingdom was still under the control of the
Sisodias.
 Akbar was intent on securing a stable route to Gujarat through Mewar; when Pratap
Singh was crowned king (Rana) in 1572, Akbar sent a number of envoys
entreating the Rana to become a vassal like many other Rajput leaders in
the region. However, Pratap refused to enter into a treaty, which led to the battle.

NADIR SHAH (PERSIAN) INVASION [1738-1739]


 Emperor Nader Shah, the Shah of Iran (1736–47) and the founder of the Afsharid dynasty,
Contact : https://t.me/EN123upsc || Telegram Link : https://t.me/prelimbits
131

invaded Northern India, eventually attacking Delhi in March 1739


 Battle of Karnal (1739): Defeated Mughals.
 Captured Lahore and Emperor Mohammad Shah.
 Annexed areas west of Indus.
 Looted Peacock throne, Kohinoor.
 The devastation of Delhi has been famously lamented by the poets Mir and Sauda.
SHIVAJI (19 FEBRUARY 1630 – 3 APRIL 1680)
 Shivaji also referred to as Chhatrapati Shivaji Maharaj, was an Indian ruler and a member of
the Bhonsle Maratha clan.
 Shivaji carved out his own independent kingdom from the declining Adilshahi
sultanate of Bijapur which formed the genesis of the Maratha Empire.
 In 1674, he was formally crowned the Chhatrapati of his realm at Raigad Fort
 Over the course of his life, Shivaji engaged in both alliances and hostilities with the
Mughal Empire, the Sultanate of Golkonda, Sultanate of Bijapur and the European
colonial powers.
 Shivaji's military forces expanded the Maratha sphere of influence, capturing and building
forts, and forming a Maratha navy.
 Shivaji established a competent and progressive civil rule with well-structured
administrative organisations.
 He revived ancient Hindu political traditions, court conventions and promoted
the usage of the Marathi and Sanskrit languages, replacing Persian in court and
administration.
 Shivaji's legacy was to vary by observer and time, but nearly two centuries after his death,
he began to take on increased importance with the emergence of the Indian independence
movement, as many Indian nationalists elevated him as a proto-nationalist and hero of the
Hindus.
 Shivaji was succeeded by his son Sambhaji, who was captured by Aurangzeb's forces in
Deccan and executed while his son, Shahu was taken prisoner.
 Shivaji's dominion was divided into two sections →
o Mulk-i-qadim (ancient territory) or Swaraj (own kingdom), and
o an unspecified stretch of land that paid Chauth but was not subject to Shivaji's
authority.
 To fortify the administration, Shivaji abolished the Jagir system and began paying his
officers in cash. He provided land grants for schools and temples despite abolishing the
Jagirdari system. In this article, we will discuss Shivaji‘s Administration which will be
helpful for UPSC exam preparation.
 With the help of a disciplined military and a well-established administrative setup,
Shivaji implemented an efficient and progressive administration.
 Shivaji was well-known for his unconventional military tactics that took advantage of
factors such as geography and speed.

Contact : https://t.me/EN123upsc || Telegram Link : https://t.me/prelimbits


132

Administration under Shivaji

Maratha Empire under Shivaji extended to Maharashtra, Carnatic and Tamil Nadu.
 Provinces were divided into Parganas and Parganas were further divided into villages.
 To strengthen the administration Shivaji abolished the Jagir system and began giving
cash salary to his officers. Though he abolished Jagirdari but gave land grants for temples
and schools.
 In his rule hereditary occupation of post was not allowed.
 Shivaji did not encourage the Zamindari system.
ASHT PRADHAN were the main axis of his administration. Eight prominent officials were
collectively known as Asht Pradhan. They were-
1. Peshwa-He was the Prime minister of the king.
2. Amatya or Majumdar-He was Finance Minister.
3. Waq-i-Nawis-He worked as Home Minister.
4. Dabir or Samant-He looked after the work of the foreign department.
5. Sachiv-He conducted the official correspondence.
6. Pandit Rao-He was a religious officer.
7. Sar-i-Naubat or Senapati-He used to administer the army affairs.
8. Nyayadhish-He was the chief justice.
Judicial System

 Shivaji's judiciary was based upon the ancient Hindu laws. In the villages, the
Panchayats settled the disputes.
 The highest court was the king's 'Hazar Majils.'
 The Panchayats handled disputes between various parties in the communities, and the
village 'Patel' decided on criminal cases.
Military

 The regular army consisted of about 30000 to 40000 cavalry and they were given fixed
salaries. Shivaji set up the Maratha navy in 1659. The most famous Maratha admiral was
Kanhoji Angre (1669-1729).
 There were two main divisions of the army of Shivaji Maharaj- infantry and cavalry.
 In infantry there were officers such as Havaldar and Jumledar. In cavalry there
were Shiledars and Bargirs.
o Bargirs, equipped and paid by the state;
o Silahdars, maintained by the nobles.
 The ordinary soldiers were paid in cash, but the chief and military commander were
paid through jagir grants (Saranjam or Mokasa).

Contact : https://t.me/EN123upsc || Telegram Link : https://t.me/prelimbits


133

o A Saranjam is grant of land (initially non-hereditary) for maintenance of troops or for


military service
Revenue Administration

 Shivaji abolished the Jagirdari System in favour of the Ryotwari System, which eliminated
the need for middlemen between farmers and the state.
 Shivaji strictly supervised the Mirasdars, who had inherited inland rights.
 The revenue system was based on Malik Amber's Kathi system, in which every piece of
land was measured by Rod or Kathi.
 Shivaji implemented the Chauth and Sardeshmukhi taxes.
o Chauth was 1/4 of the total revenue from Non-Maratha territories and it was an
annual tax.
o Sardeshmukhi was realized by Shivaji, just 1/10 of total revenue on the basis that
legally he was the Pramukh (Sardeshmukh) or the head of all Deshmukhs.
AKBAR THE GREAT (25 OCTOBER 1542 – 27 OCTOBER 1605)
 Was the third Mughal emperor, who reigned from 1556 to 1605. Akbar succeeded his
father, Humayun, under a regent, Bairam Khan, who helped the young emperor expand
and consolidate Mughal domains in India.
 A strong personality and a successful general, Akbar gradually enlarged the Mughal Empire
to include much of the Indian subcontinent.
 His power and influence, however, extended over the entire subcontinent because of
Mughal military, political, cultural, and economic dominance.
 To unify the vast Mughal state, Akbar established a centralised system of administration
throughout his empire and adopted a policy of conciliating conquered rulers through
marriage and diplomacy.
 To preserve peace and order in a religiously and culturally diverse empire, he adopted
policies that won him the support of his non-Muslim subjects. Eschewing tribal bonds
and Islamic state identity, Akbar strove to unite far-flung lands of his realm through loyalty,
expressed through an Indo-Persian culture, to himself as an emperor.
 Mughal India developed a strong and stable economy, leading to commercial
expansion and greater patronage of culture. Akbar himself was a patron of art and
culture.
 He was fond of literature, and created a library of over 24,000 volumes written in Sanskrit,
Urdu, Persian, Greek, Latin, Arabic, and Kashmiri, staffed by many scholars, translators,
artists, calligraphers, scribes, bookbinders, and readers.
 He did much of the cataloging himself through three main groupings. Akbar also
established the library of Fatehpur Sikri exclusively for women, and he decreed that
schools for the education of both Muslims and Hindus should be established throughout
the realm.
 He also encouraged bookbinding to become a high art. Holy men of many faiths, poets,
architects, and artisans adorned his court from all over the world for study and
discussion.
 Akbar's courts at Delhi, Agra, and Fatehpur Sikri became centres of the arts, letters, and
Contact : https://t.me/EN123upsc || Telegram Link : https://t.me/prelimbits
134

learning.
 Timurid and Perso-Islamic culture began to merge and blend with indigenous Indian
elements, and a distinct Indo-Persian culture emerged characterized by Mughal style
arts, painting, and architecture.
 Disillusioned with orthodox Islam and perhaps hoping to bring about religious
unity within his empire, Akbar promulgated Din-i-Ilahi, a syncretic creed derived
mainly from Islam and Hinduism as well as some parts of Zoroastrianism and
Christianity.
 Akbar's reign significantly influenced the course of Indian history. During his rule, the
Mughal Empire tripled in size and wealth.
 He created a powerful military system and instituted effective political and social reforms.
By abolishing the sectarian tax (Zajiya) on non-Muslims and appointing them
to high civil and military posts, he was the first Mughal ruler to win the trust and
loyalty of the native subjects.
 He had Sanskrit literature translated, participated in native festivals, realising that a stable
empire depended on the co-operation and good-will of his subjects.
 Thus, the foundations for a multicultural empire under Mughal rule were laid during his
reign. Akbar was succeeded as emperor by his son, Prince Salim, later known as
Jahangir.

89. Consider the following: [1998]


1. Tughlaqabad fort
2. Lodi Garden
3. Qutab Minar
4. Fatehpur Sikri

The correct chronological order in which they were built is:

A. 3, 1, 4, 2
B. 3, 1, 2, 4
C. 1, 3, 2, 4
D. 1, 3, 4, 2

Ans. B

 Tughlaqabad Fort = 1321


 Fatehpur Sikri = 1571
 Qutab Minar = 1199 -1220
 Lodi Gardens = 1444

FATEHPUR SIKRI
 Fatehpur Sikri is a town in the Agra District of Uttar Pradesh.

Contact : https://t.me/EN123upsc || Telegram Link : https://t.me/prelimbits


135

 Situated 35.7 kilometres from the district headquarters of Agra, Fatehpur Sikri itself was
founded as the capital of Mughal Empire in 1571 by Emperor Akbar, serving this role
from 1571 to 1585, when Akbar abandoned it due to a campaign in Punjab and was later
completely abandoned in 1610.
 The name of the city is derived from the village called Sikri which occupied the spot before.
An Archaeological Survey of India (ASI) excavation from 1999 to 2000 indicated that
there was a habitation, temples and commercial centres here before Akbar built his
capital. The region was settled by Sungas following their expansion.
 It was controlled by Sikarwar Rajputs from the 7th to 16th century CE until the Battle
of Khanwa (1527).
 The khanqah of Sheikh Salim Chishti existed earlier at this place. (khanqah → building
designed specifically for gatherings of a Sufi brotherhood or tariqa and is a place for
spiritual practice and religious education.
 Akbar's son Jahangir was born in the village of Sikri to his favourite wife Mariam-uz-
Zamani in 1569 and in that year Akbar began construction of a religious compound to
commemorate the Sheikh who had predicted the birth.
 After Jahangir's second birthday, he began the construction of a walled city and
imperial palace here. The city came to be known as Fatehpur Sikri, the
"City of Victory", after Akbar's victorious Gujarat campaign in 1573.
 After occupying Agra in 1803, the East India Company established an administrative centre
here and it remained so until 1850. In 1815, the Marquess of Hastings ordered the repair of
monuments at Sikri.
 Fatehpur Sikri was awarded the status of UNESCO World Heritage Site in 1986.

TUGHLUQABAD FORT
 Tughluqabad Fort is a ruined fort in Delhi, built by Ghiyasuddin Tughluq, the founder of
the Tughlaq dynasty, of the Delhi Sultanate of India in 1321, as he established the third
historic city of Delhi, which was later abandoned in 1327.
 It lends its name to the nearby Tughluqabad residential-commercial area as well as the
Tughluqabad Institutional Area.

LODHI GARDENS
 Lodhi Gardens is a city park situated in the heart of New Delhi.
 Spread over 90 acres, the park houses the mausoleums of two Delhi Sultanate rulers,
namely Mohammed Shah and Sikandar Lodi, along with other architectural works of the
15th century by Lodis, who ruled parts of northern India and Punjab and Khyber
Pakhtunkhwa province of modern-day Pakistan, from 1451 to 1526. With the blend of
century-old architectural heritage and impeccable natural beauty
 s Lodhi Gardens, formerly known as Lady Willingdon Park, has its roots back in 1444 when
the tomb of Mohammed Shah, a Sayyid Dynasty ruler of the Delhi Sultanate, was
Contact : https://t.me/EN123upsc || Telegram Link : https://t.me/prelimbits
136

founded by his son and the last ruler of the dynasty, Ala-ud-din Alam Shah.
 Another significant tomb located in the area is that of Sikandar Lodi from the Lodi
Dynasty, which was erected in 1517 by his son Ibrahim Lodi.
 Several other structures were also built here during the rule of the Sayyids and Lodis. When
Emperor Akbar became the king of Delhi, he made use of the Lodhi Gardens area as an
observatory and stored records in a library built for this purpose.
 With time, when the British government were in power, they removed the villagers residing
around the tomb and turned the area into an attractive garden in 1936. The park was then
named Lady Willingdon Park, after the wife of Marquess of Willingdon, the then Governor-
General of India.
 When the country gained its independence in 1947, it was renamed Lodhi Gardens. Later, in
1968, the park was redesigned by JA Stein, an eminent architect, who was also involved with
many other buildings around the Lodhi Gardens complex.

90. Ahadis were those troopers who:


1. offered their services singly
2. did not attach themselves to any chief
3. had the emperor as their immediate colonel
4. attached themselves to Mirzas

Which of the above statements is/are correct?

A. 1, 3, and 4 are correct


B. 1, 2, and 3 are correct
C. 2 and 3 are correct
D. 1 and 4 are correct

Ans. B

MUGHAL ARMY
 The cavalry was the principal army of the Mughal army and the Mansabdar proved the
overwhelming proportion of it.
 In addition to the Mansabdars, the Mughal emperors used to entertain individual troopers,
called Ahadis.
 The Ahadis have been called gentlemen-troopers and received much higher salaries than
other troopers.
 They were highly trusted cops, being recruited directly by the emperors.
 In addition to the Ahadis, the emperor's maintained royal bodyguards (wala-shuhis) and
armed palace guards.

Contact : https://t.me/EN123upsc || Telegram Link : https://t.me/prelimbits


137

91. [1998]
Assertion (A): During the reign of Shahjahan, Dara Sikoh was sent on an expedition to
Balkh, Badakhshan, and Qandahar.
Reason (R): The expedition sent by Shahjahan to the Middle East was a marvelous
success.
A. Both A and R are true but R is the correct explanation of A
B. Both A and R are true but R is not a correct explanation of A
C. A is true but R is false
D. A is false but R is true

Ans. C

Expedition failed miserably.

BALKH CAMPAIGN OF SHAHJAHAN


 Nestled between the Amu Darya and the Hindu Kush mountains, in the very heart of Asia,
lies Balkh. Though now little more than a provincial town in Northern Afghanistan (450 km
from Kabul), for over 2,000 years it was among the great cities of Asia.
 Formerly called ‗Bactra‘, the capital of the territory of Bactria once ruled by the Persians, the
Indo-Greeks and the Kushanas, its centrality made it a key artery of the Silk Road. Trade
bestowed upon its immense material and cultural wealth, and some considered it to be the
mythical Shangri-La.
 The city was so prosperous in ancient and medieval times that the Arabs called it Umm-Al-
Belad—the ‗mother of all cities‘. Such affluence naturally attracted conquerors from all
corners of Eurasia: King Darius of the Achaemenids (6th to 5th BCE), Alexander of
Macedonia (4th BCE), the Great Khan Genghis (12th to 13th CE) and the feared Amir Timur
(14th to 15th CE).
 In the 17th century CE, Balkh was coveted and captured by Mughal and Rajput princes. It
was the only time, possibly in all of history, that an Indian army was to go on the offensive
beyond the Hindu Kush. This was the Balkh-Badakhshan campaign of 1646-1647 CE, which
would finally bring to end the Mughal dream of reconquering their homeland in the
Ferghana Valley (in present-day Uzbekistan), the region from where Babur (r. 1526-30 CE)—
the first Mughal Emperor and founder of the dynasty in India— hailed from.

Shah Jahan launched a military expedition

 In 1646-1647, Mughal Emperor Shah Jahan launched a military expedition to


conqueror Balkh and the neighbouring province of Badakhshan. This campaign was to
be the first step in the expansion of Mughal rule into Transoxiana, now Central Asia, where
the Ferghana valley lay.
 There were many reasons why Shah Jahan undertook such an ambitious campaign. The
Khanate of Bukhara—the kingdom holding overlordship over Balkh and Badakhshan—was
Contact : https://t.me/EN123upsc || Telegram Link : https://t.me/prelimbits
138

going through a period of internal strife, which made it an opportune moment to strike.
Moreover, the imperialistic inclinations of the Safavid Dynasty of Iran, which too was eyeing
a foothold in Central Asia, made Shah Jahan want to anticipate any incursions from them.
 However, behind all the obvious geopolitical manoeuvring, there was sheer
sentimentality. The lush valleys and venerable mountains of Transoxiana were the ancestral
homelands of the Mughals. It was in Balkh where their most eminent ancestor, Amir Timur,
had declared himself ‗Khan‘, and it was from the magnificent citadel of Samarkand (in
present-day Uzbekistan) that he governed his empire.
 Babur himself was born north of Balkh, in the fertile valley of Ferghana. He spent most
of his life wandering the Khanates of Central Asia and Afghanistan, squandering one
kingdom only to gain another. While the Mughals were Emperors of Hindustan and had
been so for more than a century by Shah Jahan‘s time, Transoxiana was still ubiquitous in
their imperial imaginings.
 The official chronicle of every Mughal Emperor up to Aurangzeb mentioned plans to
conquer Central Asia. Thus, in the late 1640s, when the situation in the Deccan had
greatly eased with the collapse of the Ahmadnagar Sultanate of the Nizam Shahi,
Shah Jahan had the military flexibility to undertake what was to be the only serious
attempt at regaining the lost homeland of the Mughals. This medieval irredentism
was going to cost dearly.
 An army of 50,000 horsemen and 10,000 infantry—including artillerymen, musketeers, and
sappers—was assembled in Kabul, the last great city at the western limits of the Mughal
Empire. Leading them was Shah Jahan‘s youngest son, Mirza Murad Bakhsh.

Read more here :

https://www.peepultree.world/livehistoryindia/story/religious-places-/balkh-campaign

92. Sultan of Delhi who is reputed to have built the biggest network of canals in India was:
[1998]
A. Iltutmish
B. Ghiyasuddin Tughlaq
C. Firoz Shah Tughlaq
D. Sikandar Lodi

Ans. C

 Firuz Shah Tughlaq was born in 1309 and became sultan of Delhi after the demise of his
cousin Muhammad-bin-Tughlaq.
 He was the third ruler of the Tughlaq dynasty that ruled over Delhi from 1320 to 1412 AD.
He was in power from 1351 to 1388 AD.
 The Britisher used to call him Father of Irrigation department because of many
gardens and Canals he built like:
o The canal connecting the Yamuna to Hissar city.
Contact : https://t.me/EN123upsc || Telegram Link : https://t.me/prelimbits
139

o The Satluj to the Ghaggar.


o Mandvi and Sirmour Hills to Hansi in Haryana.
o The Ghaggar to Firuzabad.
 He is known to establish four new towns, Firuzabad, Fatebabad, Jaunpur and Hissar.
 He was the one who started Imposition of Jaziya on the Brahmans.
 He died in 1388.

93. Fawazil in the Sultanate period meant: [1998]


A. extra payment to the nobles
B. revenue assigned in lieu of salary
C. excess amount paid to the exchequer by the Iqtadars
D. illegal exactions extracted from the peasants

Ans. C

Fawazil was a balance between the income and expenditure of Iqta-holders.

94. The loss of Qandahar was a big blow to the Mughal empire from the viewpoint of [1998]
A. natural resources
B. buffer territory
C. communication
D. strategic stronghold

Ans. D

MUGHAL–SAFAVID WAR
https://en.wikipedia.org/wiki/Mughal_sieges_of_Kandahar_(1649%E2%80%931653)

 During the Mughal–Safavid War of 1649–1653, the Mughal Empire laid siege to the city of
Kandahar in Afghanistan three times. All three sieges failed, and thus the Mughal
Empire was unable to retake control of Kandahar from the Safavids.
 The loss of Kandahar was seen as a major strategic loss for the Mughal Empire.
Furthermore, the Mughals saw the defeat as a blow to the empire's prestige,

Background

 In the mid 17th-century, rising tensions between the Safavid Empire and the Mughal Empire
led to a number of territorial disputes in Afghanistan. Control of Afghanistan was
centered around two key cities, Kabul and Kandahar; by the 1630s the Mughals were
in control of Kabul, while the Safavid's controlled Kandahar.
 A major development came in 1638 when the Safavid governor of Kandahar, Ali Mardan
Khan, betrayed the Safavids and gave control of the city over to the Mughals. This
event provoked further tensions between the two empires.
 In 1647, a Mughal attempt to conquer Badakhshan ended in failure. Seizing

Contact : https://t.me/EN123upsc || Telegram Link : https://t.me/prelimbits


140

on the weakness of its rival, the Safavid Empire invaded the Mughal-controlled parts of
Afghanistan in early 1649, beginning the Mughal–Safavid War. The Safavids won a major
victory in February of that year when Kandahar was captured after a two-month siege.

95. The member of Shivaji's Ashtapradhan who looked after foreign affairs were: [1998]
A. Peshwa
B. Sachiv
C. Pandit Rao
D. Sumant

Ans. D

ASHT PRADHAN
 Asht Pradhan were the main axis of his administration. Eight prominent officials were
collectively known as Asht Pradhan. They were-
o Peshwa-He was the Prime minister of the king.
o Amatya or Majumdar-He was Finance Minister.
o Waq-i-Nawis-He worked as Home Minister.
o Dabir or Samant-He looked after the work of the foreign department.
o Sachiv-He conducted the official correspondence.
o Pandit Rao-He was a religious officer.
o Sar-i-Naubat or Senapati-He used to administer the army affairs.
o Nyayadhish-He was the chief justice.
96. The medieval Indian writer who refers to the discovery of America is : [1997]
A. Malik Muhammad Jayasi
B. Amir Khusrau
C. Raskhan
D. Abul Fazl

Ans. D

ABU'L-FAZL
 Abu'l-Fazl ibn Mubarak, also known as Abul Fazl, Abu'l Fadl and Abu'l-Fadl 'Allami (14
January 1551 – 22 August 1602), was the grand vizier of the Mughal emperor Akbar, from
his appointment in 1579 until his death in 1602.
 He was the author of the Akbarnama, the official history of Akbar's reign in three
volumes, (the third volume is known as the Ain-i-Akbari) and a Persian translation of the
Bible.[3] He was also one of the Nine Jewels (Navaratnas) of Akbar's royal court and the
brother of Faizi, the poet laureate of Emperor Akbar.

MALIK MUHAMMAD JAYASI


 Malik Muhammad Jayasi (1477– 1542) was an Indian Sufi poet and pir.
 He wrote in the Awadhi language, and in the Persian Nastaʿlīq script.
Contact : https://t.me/EN123upsc || Telegram Link : https://t.me/prelimbits
141

 His best known work is the epic poem Padmavat (1540). A poem describing the story of
the historic siege of Chittor by Alauddin Khalji in 1303. In Padmavat, Alauddin attacks
Chittor after hearing of the beauty of Queen Padmavati, the wife of king Ratansen.
 His other important works include Akhrawat and Akhiri Kalaam. He also wrote
Kanhavat, based on Krishna.[1]

SYED IBRAHIM KHAN / RASKHAN (1548-1628)


 Was an Indian Sufi Muslim poet who became a devotee of the Hindu deity Krishna.
 He was either born in Pihani (Hardoi)[1] or Amroha, in modern-day Uttar Pradesh, India. His
original name was Saiyad Ibrahim and Raskhan was his takhallus (pen name) in Hindi.
 In his early years, he became a follower of Krishna, learned the bhakti marga from
Vitthalanatha and began living in Vrindavan, where he spent the rest of his life.
 He accepted Krishna as the supreme god (Svayam Bhagavan) and became a Vaishnava. He
died in 1628 AD. His samadhi is at Mahaban, about six miles east of Mathura.
 Poetry
o Raskhan is widely acknowledged as a great poet, having dedicated most of his
creations to Lord.
o Rachnavali is the collection of Raskhan's poetry. His creations describe the beauty of
not only Lord Krishna but also his relations with his beloved Radha. His poetry is in
the form of Doha, Padawali and Savayya.
o Raskhan's Khariboli writings are numerous, the five most important being the Sujana
Raskhana, the Premavatika, the Danalila, the Astayama and a collection of Padas
(rhymed couplets). Of these the most well-known is the Premavatika ("The
Forest of Love").

97. Which one of the following pairs of composers in different languages and their works
on the Mahabharata theme is correctly matched? [1997]
A. Sarladasa–Bengali
B. Kasirama–Oriya
C. Tikkana–Marathi
D. Pampa–Kannada

Ans. D

ADIKAVI PAMPA
 Pampa (c. 10th century), called by the honorific Ādikavi ("First Poet") was a Kannada-
language Jain poet whose works reflected his philosophical beliefs.
 A court poet of Vemulavada Chalukya king Arikesari II, he was a feudatory of the
Rashtrakuta dynasty king Krishna III.
 Pampa is best known for his epics Vikramārjuna Vijaya or Pampa Bharata,
and the Ādi purāṇa, both written in the champu style around c.939.
Contact : https://t.me/EN123upsc || Telegram Link : https://t.me/prelimbits
142

 These works served as the model for all future champu works in Kannada.

The works of Jain writers Pampa, Sri Ponna and Ranna, collectively called the "Three
gems of Kannada literature", heralded the 10th century era of medieval Kannada literature
KASHIRAM DAS (16TH CENTURY)
 Important poet in medieval Bengali literature.
 His Bengali re-telling of the Mahābhārata, known as Kāśīdāsī Môhābhārôt, is a popular
and influential version of the Mahābhārata legend in Bengal.

SARALA DASA (15TH-CENTURY)


 Was a 15th-century poet and scholar of Odia literature.
 Best known for three Odia books — Mahabharata, Vilanka Ramayana and Chandi
Purana — he was the first scholar to write in Odia and his revered as the Adi Kabi (First
Poet) of Odia literature.
 As an originator of Odia literature, his work has formed an enduring source of information
for succeeding generations.

TIKKANA
 Tikkana (or Tikkana Somayaji) (1205–1288) was a 13th century Telugu poet.
 Born into a Telugu-speaking Niyogi Brahmin family during the golden age of the Kakatiya
dynasty, he was the second poet of the "Trinity of Poets (Kavi Trayam)" that
translated Mahabharata into Telugu.
 Nannaya Bhattaraka, the first, translated two and a half chapters of Mahabharata.
 Tikkana translated the final 15 chapters, but did not undertake translating the half-
finished Aranya Parvamu.
 The Telugu people remained without this last translation for more than a century, until it
was translated by Errana.
 Tikkana is also called Tikkana Somayaji, as he completed the Somayaga. Tikkana's titles were
Kavibrahma and Ubhaya Kavi Mitrudu.

https://en.wikipedia.org/wiki/Kavitrayam

98. Head of the military department under the recognized central machinery of
administration during Akbar's reign was: [1997]
A. Diwan
B. Mir Bakshi
C. Mir Saman
D. Bakshi

Ans. B

Contact : https://t.me/EN123upsc || Telegram Link : https://t.me/prelimbits


143

MIR BAKSHI
 The head of the military administration was Mir Bakshi who was also considered as head
of the nobility.

 The Mir Bakshi was also the head of the intelligence and information agencies of the
empire. Barids (intelligence officers) and Waqia-navis (news reporters) were posted to
all the parts of the empire.

 Their reports were presented to the emperor at the court through Mir Bakshi.

99. After consolidating his power, Balban assumed the grand title of [1997]
A. Tute-Hind
B. Kaisr-I-Hind
C. Zil-I-Ilahi
D. Din-I-Ilahi

Ans. C

BALBAN (1266-1287)
 Also known as Ulugh khan. He was one of the main architects of the Delhi Sultanate.

 According to him, the Sultan was God‘s shadow on earth (Zil-i-Ilahi) and recipient of
divine grace (Nibyabat- i-Khudai).

 He broke the power of the Forty.

 Introduced Persian festival Nawrouz.

 Called himself Nasir-amir-ul-momin (Caliph‘s right-hand man).

Administration
 Separated Diwan-i-wizarat (Finance Department) from the Diwan-i-Arz (Military
Department).

 Administered justice with extreme impartiality.

 He followed policy of blood & iron.

 Excluded non-Turks from administration.

 Indian Muslims not given important posts.

 Appointed spies to monitor the activities of the nobles.

 Introduced sijada (prostration) and paibos (kissing the Sultan‘s feet) to prove his
superiority over the nobles

 He also introduced Persian festival of Nauroz

Contact : https://t.me/EN123upsc || Telegram Link : https://t.me/prelimbits


144

100. Prem Vatika, poems on the life of Krishna, were composed by: [1996]
A. Bihari
B. Surdas
C. Raskhan
D. Kabir

Ans. C

SYED IBRAHIM KHAN / RASKHAN (1548-1628)


 Was an Indian Sufi Muslim poet who became a devotee of the Hindu deity Krishna.
 He was either born in Pihani (Hardoi) or Amroha, in modern-day Uttar Pradesh, India. His
original name was Saiyad Ibrahim and Raskhan was his takhallus (pen name) in Hindi.
 In his early years, he became a follower of Krishna, learned the bhakti marga from
Vitthalanatha and began living in Vrindavan, where he spent the rest of his life.
 He accepted Krishna as the supreme god (Svayam Bhagavan) and became a Vaishnava. He
died in 1628 AD. His samadhi is at Mahaban, about six miles east of Mathura.
 Poetry
o Raskhan is widely acknowledged as a great poet, having dedicated most of his
creations to Lord.
o Rachnavali is the collection of Raskhan's poetry. His creations describe the beauty of
not only Lord Krishna but also his relations with his beloved Radha. His poetry is in
the form of Doha, Padawali and Savayya.
o Raskhan's Khariboli writings are numerous, the five most important being the Sujana
Raskhana, the Premavatika, the Danalila, the Astayama and a collection of Padas
(rhymed couplets). Of these the most well-known is the Premavatika ("The
Forest of Love").

101. Which of the following pairs is correctly matched? [1996]


A. Guru Amar Das–Miri and Piri
B. Guru Arjun Dev–Adi Granth
C. Guru Ram Das–Dal Khalsa
D. Guru Gobind Singh– Manji

Ans. B

 Guru Amar Das = Manji (was a Sikh religious administrative unit)


 Guru Hargobind = The concept of "The Mir and the Pir" (temporal power and spiritual
authority) was started by the sixth Guru of Sikhism, Guru Hargobind, on June 12, 1606.
 Guru Arjun Dev = Adi Granth
 Guru Gobind Singh = Dal Khalsa

Contact : https://t.me/EN123upsc || Telegram Link : https://t.me/prelimbits


145

THE SIKHS

 Sikhism was founded by Guru Nanak in the Punjab region in the 15th century. The Sikh
Panth was as old as the Mughal Empire.
 When Guru Nanak began to preach his message of devotion and equality in Punjab in the
15th century, Babur was founding the Mughal Empire.
 ‗Sikh‘ literally means learner or disciple. In the course of time, the new cult took the form
of a religious movement and spread quickly among the Jat peasantry and other lower
castes in Punjab.
 Guru Nanak‘s religious movement was peaceful and secular.
 Nanak was succeeded by nine Gurus; all the ten Sikh Gurus emphasised on simplicity of
religion and freedom from bigotry.

o Guru Nanak : founder of Sikhism.


o Guru Angad : developed the Gurumukhi script, introduced the langar (free
kitchen) and tradition of mall akhara for the youth.
o Guru Amar Das : composed the Anand Sahib, introduced the simple Anand
Karaj marriage and abolished sati among the Sikhs.
o Guru Ram Das : laid the foundation of the holy city of Amritsar (earlier known
as Ramdaspur and Guru-ka-Chak) and initiated the construction of the Golden
Temple.
o Guru Arjan Dev : compiled the Adi Granth and completed the construction of
the Golden Temple (It was during Ranjit Singh‘s reign that the shrine of the temple
was richly decorated with marble and gold plates and came to be known as the
Golden Temple).
o Guru Hargobind : created the Sikh marital art called Gatka, trained the Sikhs in
military art and war tactics, built the Akai Takhat, and was known as the ‗soldier
saint‘.
o Guru Har Rai : he maintained the large army of Sikh soldiers that the sixth Sikh
Guru had amassed, yet avoided military conflict, and was known as the ‗tender-
hearted guru‘.
o Guru Har Krishan : was the youngest of all gurus, installed as guru at the age of
five.
o Guru Teg Bahadur : he refused to convert to Islam and was beheaded in Delhi
on the orders of Mughal Emperor Aurangzeb.
o Guru Gobind Singh : transformed the Sikhs into a military force by
establishing the body of the Khalsa in 1699.
102. In medieval India, the Mansabdari system was introduced for: [1996]
A. making recruitment to the army
B. facilitating revenue collection
C. ensuring religious harmony
D. effecting clean administration
Contact : https://t.me/EN123upsc || Telegram Link : https://t.me/prelimbits
146

Ans. D

Refer Q. 84

MANSABDARI SYSTEM
 Mansabdars were officers in Mughal administration.
 Those nobles who joined Mughal service were enrolled as mansabdars.
 The term mansabdar refers to an individual who holds a mansab (rank).
 The Mansabdars were appointed to all civil and military posts.
 They were liable to be transferred from one branch of the administration (civil) to another
(military).

103. Mughal painting reached its zenith under [1996]


A. Humayun
B. Akbar
C. Jahangir
D. Shahjahan

Ans. C

JAHANGIR → PAINTING
 Jahangir was fascinated with art and architecture. In his autobiography, the Jahangirnama,
Jahangir recorded events that occurred during his reign, descriptions of flora and fauna
that he encountered, and other aspects of daily life, and commissioned court painters
such as Ustad Mansur to paint detailed pieces that would accompany his vivid prose.
 For example, in 1619, he put pen to paper in awe of a royal falcon delivered to his court
from the ruler of Iran: ―What can I write of the beauty of this bird's colour? It had black
markings, and every feather on its wings, back, and sides was extremely beautiful,‖ and then
recorded his command that Ustad Mansur paint a portrait of it after it perished.
 Jahangir bound and displayed much of the art that he commissioned in
elaborate albums of hundreds of images, sometimes organized around a
theme such as zoology.
 Jahangir took his connoisseurship of art very seriously. He also preserved paintings from
Emperor Akbar's period.
 An excellent example of this is the painting done by Ustad Mansur of Musician Naubat
Khan, son in law of legendary Tansen.
 In addition to their aesthetic qualities, paintings created under his reign were closely
catalogued, dated and even signed, providing scholars with fairly accurate ideas as to
when and in what context many of the pieces were created.
 Jahangir himself was far from modest in his autobiography when he stated his prowess at
being able to determine the artist of any portrait by simply looking at a painting.

Contact : https://t.me/EN123upsc || Telegram Link : https://t.me/prelimbits


147

 Ustad Mansur, Abdul Hassan, Farukh Beg, Murad, Madhav were the famous painters in
Jahangir‘s court. During Jahangir's reign, the tradition of miniature paintings
developed, which is famous even today.

104. The Sufi saint who maintained that devotional music was one way of coming close
to God was; [1996]
A. Muin-ud-din Chisti
B. Baba Farid
C. Saiyid Muhammad Gesudaraz
D. Shah Alam Bukhari

Ans. A

Muin-ud-din Chishti

 The Sufi saint who maintained that devotional music was one way of coming close to God
was Muin-ud-din Chishti. He was born in Afghanistan in 1156.He began his religious career
at the age of 13. He distributed his inheritance among the poor and adopted the simple life
of an itinerant Sufi fakir.
 Khwaja Sahib and his disciples settled in Ajmer at the beginning of the thirteenth century.
Withdrawing into a life of meditation and fasting, he preached a message of renunciation,
affirming that personal experience of God was attainable to anyone who relinquished their
ties to the world.

105. Nastaliq was: [1996]


A. a Persian script used in medieval India
B. a raga composed by Tansen
C. a cess levied by the Mughal rulers
D. a manual of code of conduct for the Ulemas

Ans. A

 It was a persian script during the Mughal period.

106. The term 'Apabhramsa' was used in medieval Sanskrit texts to denote: [1996]
A. outcastes among the Rajputs
B. deviations from Vedic rituals
C. early forms of some of the modern Indian language
D. non-Sanskrit verse meters

Ans. C

APABHRAMS
 Before the rise of modern language, Apabhrams was the most vibrant language of

Contact : https://t.me/EN123upsc || Telegram Link : https://t.me/prelimbits


148

colloquial and literary works in north India.


 Between the late Middle and the early Modern Indo-Aryan languages, spanning the
period between the 6th and 13th centuries CE.
 Apabhramsa in Sanskrit literally means "corrupt" or "non-grammatical language", that
which deviates from the norm of Sanskrit grammar.
 The only known example of an Apabhramsa work by a Muslim is the Sandesarasaka of
Abdur Rahman of Multan, possibly written around 1000 CE.

Writers and poets

 Mahakavi Swyambhudev (8th century CE)


o Ritthanemichariu
o Pauma-Chariu
 Mahakavi Pushpadant (10th century)
o Mahapuran
o Naykumarchariu
o Jasaharchariu
 Hemachandra (12th century)
 Abdul Rehman (13th century) – Multani poet who penned an epic romance in Apabhraṃśa.
 Padmanabha who wrote Kanhadade Prabandha (15th century)

Contact : https://t.me/EN123upsc || Telegram Link : https://t.me/prelimbits

You might also like